dr. robert a. durham, phd, pe dr. marcus o. durham, phd, pe … nutshell master.pdf · dr. robert...

325
ELECTRICAL ENGINEERING in a Nutshell Dr. Robert A. Durham, PhD, PE Dr. Marcus O. Durham, PhD, PE A Quick Reference The book is designed with four purposes: A Study Guide to provide assistance with preparation for professional examinations. A Reference for investigating those occasional problems outside your regular specialty. A Curriculum for professional classes for maintaining your license. A Synopsis for university students in electrical engineering and other technical disciplines.

Upload: dangliem

Post on 25-Jul-2018

217 views

Category:

Documents


0 download

TRANSCRIPT

Page 1: Dr. Robert A. Durham, PhD, PE Dr. Marcus O. Durham, PhD, PE … Nutshell Master.pdf · Dr. Robert A. Durham, PhD, PE . Dr. Marcus O. Durham, PhD, PE . A Quick Reference . The book

ELECTRICAL ENGINEERING in a Nutshell

Dr. Robert A. Durham, PhD, PE

Dr. Marcus O. Durham, PhD, PE

A Quick Reference

The book is designed with four purposes:

A Study Guide to provide assistance with preparation for

professional examinations.

A Reference for investigating those occasional problems outside your

regular specialty.

A Curriculum for professional classes for maintaining your license.

A Synopsis for university students in electrical engineering and other

technical disciplines.

Robert.Durham
Typewritten Text
Page 2: Dr. Robert A. Durham, PhD, PE Dr. Marcus O. Durham, PhD, PE … Nutshell Master.pdf · Dr. Robert A. Durham, PhD, PE . Dr. Marcus O. Durham, PhD, PE . A Quick Reference . The book

Electrical Engineering

in a Nutshell

A Quick Reference

Contact:

THEWAY Corp.

P.O. Box 33124

Tulsa, OK 74153

www.ThewayCorp.com

[email protected]

Cover Design:

Cover photo:

Printed in United States of America

First printing: March 2006

Library of Congress Control Number

ISBN: 978-0-9719324-7-?

Copyright 2006 by Robert A. Durham

All rights reserved under International Copyright Law. Contents and/or cover may not be

reproduced in whole or in part in any form without the express written consent of the Publisher.

Page 3: Dr. Robert A. Durham, PhD, PE Dr. Marcus O. Durham, PhD, PE … Nutshell Master.pdf · Dr. Robert A. Durham, PhD, PE . Dr. Marcus O. Durham, PhD, PE . A Quick Reference . The book

Contents Chapter 1 - Circuit Elements and Analysis ................................................................................................. 1-1

1.1 Introduction ............................................................................................................................... 1-1

1.2 Circuit Element Description & Construction .............................................................................. 1-3

1.2.1 Resistor ..................................................................................................................................... 1-3

1.2.2 Inductor ..................................................................................................................................... 1-3

1.3 Circuit Basics .............................................................................................................................. 1-5

1.3.1 Element Manipulation .............................................................................................................. 1-5

1.3.2 Element Combinations .............................................................................................................. 1-6

1.3.3 Sources ...................................................................................................................................... 1-7

1.3.4 Circuit Laws ............................................................................................................................... 1-8

1.4 Circuit Analysis ........................................................................................................................... 1-9

1.4.1 Loop Current Method ............................................................................................................. 1-10

1.4.2 Node Voltage Method ............................................................................................................. 1-12

1.4.3 Equivalent Circuit Method ...................................................................................................... 1-13

1.5 Single Source and Impedance .................................................................................................. 1-14

1.6 Steady State AC Analysis .......................................................................................................... 1-18

1.7 Electric Fields – Electrostatics .................................................................................................. 1-20

1.8 Magnetic Fields ........................................................................................................................ 1-21

1.9 Maxwell ................................................................................................................................... 1-24

Chapter 1 Problems ............................................................................................................................. 1-25

Chapter 2 Waveforms ................................................................................................................................ 2-1

2.1 Introduction ..................................................................................................................................... 2-1

2.2 Waveforms .................................................................................................................................... 2-1

2.3 Transients ......................................................................................................................................... 2-2

2.3.1 First Order Transients ............................................................................................................... 2-2

2.3.2 RL Circuits .................................................................................................................................. 2-4

2.3.3 RC Circuits ..................................................................................................................................... 2-5

2.4 LaPlace ............................................................................................................................................. 2-6

2.5 LaPlace Operational Rules ................................................................................................................ 2-8

2.5.1 Partial Fraction Expansion ......................................................................................................... 2-9

2.5.2 Alternate Approach ................................................................................................................. 2-10

Page 4: Dr. Robert A. Durham, PhD, PE Dr. Marcus O. Durham, PhD, PE … Nutshell Master.pdf · Dr. Robert A. Durham, PhD, PE . Dr. Marcus O. Durham, PhD, PE . A Quick Reference . The book

2.6 Fourier Series ................................................................................................................................. 2-11

2.7 Signals - Modulation ...................................................................................................................... 2-15

2.7.1 Modulation Types ................................................................................................................... 2-15

2.7.2 Amplitude Modulation (AM) ................................................................................................... 2-17

2.7.3 Angle Modulation ................................................................................................................... 2-19

2.7.4 Frequency Modulation (FM) ................................................................................................... 2-20

2.7.5 Phase Modulation ................................................................................................................... 2-21

2.7.6 Sampled Messages .................................................................................................................. 2-22

2.7.7 Digital - Pulse Modulation ....................................................................................................... 2-22

2.8 Signal transmission ........................................................................................................................ 2-23

2.8.1 dBm ......................................................................................................................................... 2-23

2.8.2 Noise ....................................................................................................................................... 2-23

2.8.3 Propagation - Transmit ........................................................................................................... 2-25

2.8.4 Reflections ............................................................................................................................... 2-26

2.9 RLC System Response .................................................................................................................... 2-28

2.9.1 RLC Equations .......................................................................................................................... 2-28

2.9.2 System Response .................................................................................................................... 2-29

2.9.3 Characteristic Transfer ............................................................................................................ 2-29

2.9.4 Resonance ............................................................................................................................... 2-29

2.9.5 Series Parallel Duality .............................................................................................................. 2-32

2.9.6 First Order ............................................................................................................................... 2-32

Chapter 2 Problems ............................................................................................................................. 2-33

Chapter 2 Waveforms ................................................................................................................................ 2-1

2.1 Introduction ..................................................................................................................................... 2-1

2.2 Waveforms .................................................................................................................................... 2-1

2.3 Transients ......................................................................................................................................... 2-2

2.3.1 First Order Transients ............................................................................................................... 2-2

2.3.2 RL Circuits .................................................................................................................................. 2-4

2.3.3 RC Circuits ..................................................................................................................................... 2-5

2.4 LaPlace ............................................................................................................................................. 2-6

2.5 LaPlace Operational Rules ................................................................................................................ 2-8

2.5.1 Partial Fraction Expansion ......................................................................................................... 2-9

Page 5: Dr. Robert A. Durham, PhD, PE Dr. Marcus O. Durham, PhD, PE … Nutshell Master.pdf · Dr. Robert A. Durham, PhD, PE . Dr. Marcus O. Durham, PhD, PE . A Quick Reference . The book

2.5.2 Alternate Approach ................................................................................................................. 2-10

2.6 Fourier Series ................................................................................................................................. 2-11

2.7 Signals - Modulation ...................................................................................................................... 2-15

2.7.1 Modulation Types ................................................................................................................... 2-15

2.7.2 Amplitude Modulation (AM) ................................................................................................... 2-17

2.7.3 Angle Modulation ................................................................................................................... 2-19

2.7.4 Frequency Modulation (FM) ................................................................................................... 2-20

2.7.5 Phase Modulation ................................................................................................................... 2-21

2.7.6 Sampled Messages .................................................................................................................. 2-22

2.7.7 Digital - Pulse Modulation ....................................................................................................... 2-22

2.8 Signal transmission ........................................................................................................................ 2-23

2.8.1 dBm ......................................................................................................................................... 2-23

2.8.2 Noise ....................................................................................................................................... 2-23

2.8.3 Propagation - Transmit ........................................................................................................... 2-25

2.8.4 Reflections ............................................................................................................................... 2-26

2.9 RLC System Response .................................................................................................................... 2-28

2.9.1 RLC Equations .......................................................................................................................... 2-28

2.9.2 System Response .................................................................................................................... 2-29

2.9.3 Characteristic Transfer ............................................................................................................ 2-29

2.9.4 Resonance ............................................................................................................................... 2-29

2.9.5 Series Parallel Duality .............................................................................................................. 2-32

2.9.6 First Order ............................................................................................................................... 2-32

Chapter 2 Problems ............................................................................................................................. 2-33

Chapter 1 - Circuit Elements and Analysis ................................................................................................. 1-1

1.1 Introduction ............................................................................................................................... 1-1

1.2 Circuit Element Description & Construction .............................................................................. 1-3

1.2.1 Resistor ..................................................................................................................................... 1-3

1.2.2 Inductor ..................................................................................................................................... 1-3

1.3 Circuit Basics .............................................................................................................................. 1-5

1.3.1 Element Manipulation .............................................................................................................. 1-5

1.3.2 Element Combinations .............................................................................................................. 1-6

1.3.3 Sources ...................................................................................................................................... 1-7

Page 6: Dr. Robert A. Durham, PhD, PE Dr. Marcus O. Durham, PhD, PE … Nutshell Master.pdf · Dr. Robert A. Durham, PhD, PE . Dr. Marcus O. Durham, PhD, PE . A Quick Reference . The book

1.3.4 Circuit Laws ............................................................................................................................... 1-8

1.4 Circuit Analysis ........................................................................................................................... 1-9

1.4.1 Loop Current Method ............................................................................................................. 1-10

1.4.2 Node Voltage Method ............................................................................................................. 1-12

1.4.3 Equivalent Circuit Method ...................................................................................................... 1-13

1.5 Single Source and Impedance .................................................................................................. 1-14

1.6 Steady State AC Analysis .......................................................................................................... 1-18

1.7 Electric Fields – Electrostatics .................................................................................................. 1-20

1.8 Magnetic Fields ........................................................................................................................ 1-21

1.9 Maxwell ................................................................................................................................... 1-24

Chapter 1 Problems ............................................................................................................................. 1-25

Chapter 3 - Power Analysis ........................................................................................................................ 3-1

3.1 Introduction ..................................................................................................................................... 3-1

3.2 Power Definitions ............................................................................................................................. 3-2

3.2.1 – Z,R,X,θ; S,P,Qpf Conversions .................................................................................................. 3-4

3.3 Three-Phase AC ................................................................................................................................ 3-5

3.3.1 Y-∆ Relationships ....................................................................................................................... 3-6

3.3.2 Phase Sequence ........................................................................................................................ 3-7

3.4 Power Transfer across a Reactance ................................................................................................. 3-8

3.5 Power One-Line ................................................................................................................................ 3-9

3.6 Power Problem Plan ....................................................................................................................... 3-10

3.7 Mechanical Power .......................................................................................................................... 3-11

3.8 Electric Machinery ......................................................................................................................... 3-13

3.8.1 Basics ....................................................................................................................................... 3-13

3.8.2 Machine Models ..................................................................................................................... 3-14

3.8.3 Machine Tests ......................................................................................................................... 3-18

3.8.4 Rotating Machines .................................................................................................................. 3-19

3.8.5 Power Conversion ................................................................................................................... 3-21

3.8.6 Losses ...................................................................................................................................... 3-22

3.8.7 Performance............................................................................................................................ 3-23

3.9 Transformer ................................................................................................................................... 3-24

3.9.1 Transformer Tests ................................................................................................................... 3-25

Page 7: Dr. Robert A. Durham, PhD, PE Dr. Marcus O. Durham, PhD, PE … Nutshell Master.pdf · Dr. Robert A. Durham, PhD, PE . Dr. Marcus O. Durham, PhD, PE . A Quick Reference . The book

3.9.2 Transformer Turns .................................................................................................................. 3-28

3.10 DC Machines ................................................................................................................................ 3-31

3.11 AC Machines - Synchronous ......................................................................................................... 3-33

3.11.1 Synchronous Machine ........................................................................................................... 3-33

3.11.2 Induction Machine .................................................................................................................... 3-35

3.12 Transmission Lines ....................................................................................................................... 3-37

3.13 Per Unit Notation ......................................................................................................................... 3-39

3.14 Short Circuit Considerations ........................................................................................................ 3-41

3.14.1 Introduction .......................................................................................................................... 3-41

3.14.2 Fault Analysis ........................................................................................................................ 3-43

3.14.3 Symmetrical Components ..................................................................................................... 3-44

3.14.4 Ratings & Reactances ............................................................................................................ 3-45

3.14.5 Short Circuit Study ................................................................................................................ 3-46

3.14.6 Unbalanced Faults ................................................................................................................. 3-48

3.14.7 Faults with Rotating Machines .............................................................................................. 3-49

3.14.8 Fault Illustrations .................................................................................................................. 3-50

Chapter 3 Problems ............................................................................................................................. 3-55

Chapter 4 - Electronics ............................................................................................................................... 4-1

4.1 Introduction ..................................................................................................................................... 4-1

4.1.1 Solid State Device Characteristics ............................................................................................. 4-2

4.2 Boundary Conditions ........................................................................................................................ 4-4

4.3 Diodes & Rectifiers ........................................................................................................................... 4-6

4.3.1 Diodes ....................................................................................................................................... 4-6

4.3.2 Rectifiers & Clippers .................................................................................................................. 4-7

4.4 Operational Amplifiers ..................................................................................................................... 4-8

4.5 Transistors ...................................................................................................................................... 4-10

4.5.1 Bias vs. Small Signal ................................................................................................................. 4-10

4.5.2 Transistor Mathematical Relationships .................................................................................. 4-13

4.5.3 General Two-port Models ....................................................................................................... 4-15

4.5.4 BJT Transistor Models ............................................................................................................. 4-15

4.5.5 FET Transistor Models ............................................................................................................. 4-29

4.6 State Space ..................................................................................................................................... 4-35

Page 8: Dr. Robert A. Durham, PhD, PE Dr. Marcus O. Durham, PhD, PE … Nutshell Master.pdf · Dr. Robert A. Durham, PhD, PE . Dr. Marcus O. Durham, PhD, PE . A Quick Reference . The book

4.6.1 The 6-Minute Approach .......................................................................................................... 4-35

4.6.2 Description .............................................................................................................................. 4-37

Chapter 4 Problems ............................................................................................................................. 4-42

Chapter 5 - Controls ................................................................................................................................... 5-1

5.1 Introduction ..................................................................................................................................... 5-1

5.2 Controls Basics ................................................................................................................................. 5-2

5.2.1 Introduction .............................................................................................................................. 5-2

5.2.2 Block Diagrams .......................................................................................................................... 5-2

5.2.3 Steady State Errors .................................................................................................................... 5-4

5.2.4 Time Response .......................................................................................................................... 5-5

5.3 Routh-Hurwitz Criteria ..................................................................................................................... 5-6

5.3.1 Introduction .............................................................................................................................. 5-6

5.3.2 Rules .......................................................................................................................................... 5-6

5.3.3 Routh-Hurwitz – Special Cases .................................................................................................. 5-6

5.4 Root Locus ..................................................................................................................................... 5-11

5.4.1 Introduction ............................................................................................................................ 5-11

5.4.2 Object of Root Locus ............................................................................................................... 5-12

5.4.3 Rules for Root Locus Construction .......................................................................................... 5-13

5.5 Frequency Response Plots ............................................................................................................. 5-18

5.5.1 Introduction ............................................................................................................................ 5-18

5.5.2 Bode Plots – Basic rules .......................................................................................................... 5-19

5.5.3 – Phase Margin and Gain Margin ........................................................................................... 5-22

5.5.3 Polar - Nyquist Plot. ................................................................................................................ 5-23

5.6 Analog Filters ................................................................................................................................. 5-25

Chapter 5 Problems ............................................................................................................................. 5-30

Problem 5-1 ...................................................................................................................................... 5-30

Chapter 6 Digital ........................................................................................................................................ 6-1

6.1 Introduction ..................................................................................................................................... 6-1

6.2 Binary (Digital) Systems ................................................................................................................... 6-1

6.2.1 Number Systems ....................................................................................................................... 6-1

6.2.2 Binary System Wiring ................................................................................................................ 6-4

6.2.3 The Huntington Postulates ....................................................................................................... 6-6

Page 9: Dr. Robert A. Durham, PhD, PE Dr. Marcus O. Durham, PhD, PE … Nutshell Master.pdf · Dr. Robert A. Durham, PhD, PE . Dr. Marcus O. Durham, PhD, PE . A Quick Reference . The book

6.2.4 Basic Digital Gates ..................................................................................................................... 6-7

6.3 Karnaugh Maps ................................................................................................................................ 6-8

6.3.1 Construction / Simplification of Karnaugh Maps: ..................................................................... 6-9

6.4 Design w/ Multiplexer .................................................................................................................... 6-11

6.5 Decoder .......................................................................................................................................... 6-14

6.5.1 74156 DECODER ...................................................................................................................... 6-15

6.6 Flip Flops / Latch ............................................................................................................................ 6-16

6.6.1 General – Flip Flop Types ........................................................................................................ 6-17

6.6.2 Counter ................................................................................................................................... 6-18

6.6.3 Sequence Detector (Random) ................................................................................................. 6-23

Chapter 7 - Economics – Time Value of Money ......................................................................................... 7-1

7.1 Introduction ..................................................................................................................................... 7-1

7.2 Time and Interest ............................................................................................................................. 7-2

7.2.1 Uniform series ........................................................................................................................... 7-3

7.2.2 Gradient .................................................................................................................................... 7-5

7.2.3 Nominal Interest or APR ........................................................................................................... 7-6

7.2.4 Perpetual and Rule of 72........................................................................................................... 7-7

7.3 Rate of Return .............................................................................................................................. 7-8

7.3.1 Incremental Analysis ................................................................................................................. 7-9

7.3.2 Payback ................................................................................................................................... 7-10

7.3.3 Benefit Cost Ratio ................................................................................................................... 7-11

7.3.4 Tax Implications ...................................................................................................................... 7-12

7.4 Table of Terminology ..................................................................................................................... 7-13

7.5 Commentary .................................................................................................................................. 7-13

7.6 Review ............................................................................................................................................ 7-13

Chapter 7 Problems ............................................................................................................................. 7-15

Bibliography ......................................................................................................................................... 7-16

Chapter 8 Business Ethics .......................................................................................................................... 8-1

240.15 Rules of Professional Conduct ................................................................................................... 8-1

Chapter 9 Codes and Standards ................................................................................................................. 9-1

9.1 Introduction ..................................................................................................................................... 9-1

9.2 NEC Synopsis .................................................................................................................................... 9-1

Page 10: Dr. Robert A. Durham, PhD, PE Dr. Marcus O. Durham, PhD, PE … Nutshell Master.pdf · Dr. Robert A. Durham, PhD, PE . Dr. Marcus O. Durham, PhD, PE . A Quick Reference . The book

General: Article 100 ........................................................................................................................... 9-1

Identification of grounded conductors: Article 200........................................................................... 9-1

Branch circuits: Article 210 ................................................................................................................ 9-1

Feeders: Article 215 ........................................................................................................................... 9-1

Branch-circuit, feeder, and service calculations: Article 220 ............................................................. 9-2

Overcurrent protection: Article 240 .................................................................................................. 9-2

Grounding: Article 250 ....................................................................................................................... 9-2

Conductors for general wiring: Table 310.16 ff ................................................................................. 9-2

Motors: Article 430 ............................................................................................................................ 9-2

Tables: Chapter 9 ............................................................................................................................... 9-2

Examples: Annex D ............................................................................................................................. 9-3

9.3 Motor Installation Tables ................................................................................................................. 9-4

NEC 240.6(A) Standard Ampere Ratings for Fuses & Circuit Breakers ........................................... 9-4

Table 310.16 Allowable Ampacities of Insulated Conductors Rated 0 Through 2000 Volts, 60°C

Through 90°C (140°F Through 194°F), Not More Than Three Current-Carrying Conductors in

Raceway, Cable, or Earth (Directly Buried), Based on Ambient Temperature of 30°C (86°F) ........... 9-5

Table 310.15(B)(2)(a) Adjustment Factors for More Than Three Current-Carrying Conductors in a

Raceway or Cable ............................................................................................................................... 9-6

Table 310.15(B)(6) Conductor Types and Sizes for 120/240-Volt, 3-Wire, Single-Phase Dwelling

Services and Feeders. ......................................................................................................................... 9-6

Table 430.7(B) Locked-Rotor Indicating Code Letters .................................................................... 9-8

Table 430.52 Maximum Rating or Setting of Motor Branch-Circuit Short-Circuit and Ground-Fault

Protective Devices .............................................................................................................................. 9-8

Table 430.91 Motor Controller Enclosure Selection ...................................................................... 9-9

Table 430.248 Full-Load Currents in Amperes, Single-Phase Alternating-Current Motors ......... 9-11

Table 430.250 Full-Load Current, Three-Phase Alternating-Current Motors .............................. 9-12

Table 430.251(A) Conversion Table of Single-Phase Locked- Rotor Currents for Selection of

Disconnecting Means and Controllers as Determined from Horsepower and Voltage Rating ....... 9-13

Table 430.251(B) Conversion Table of Polyphase Design B, C, and D Maximum Locked-Rotor

Currents for Selection of Disconnecting Means and Controllers as Determined from Horsepower

and Voltage Rating and Design Letter.............................................................................................. 9-14

Table 8 Conductor Properties ...................................................................................................... 9-15

Table 9 Alternating-Current Resistance and Reactance for 600-Volt Cables, 3-Phase, 60 Hz, 75°C

(167°F) — Three Single Conductors in Conduit ............................................................................... 9-16

Page 11: Dr. Robert A. Durham, PhD, PE Dr. Marcus O. Durham, PhD, PE … Nutshell Master.pdf · Dr. Robert A. Durham, PhD, PE . Dr. Marcus O. Durham, PhD, PE . A Quick Reference . The book

Table C.4 Maximum Number of Conductors or Fixture Wires in Intermediate Metal Conduit (IMC)

(Based on Table 1, Chapter 9) .......................................................................................................... 9-18

NEMA Controller Size for Motors, Transformers, & Capacitors ...................................................... 9-21

NEMA Table 11 Typical Characteristics and Applications of Fixed Frequency Small and Medium AC

Squirrel-Cage Induction Motors ....................................................................................................... 9-23

NEMA MOTOR DIMENSIONS ........................................................................................................... 9-24

NEMA MOTOR DIMENSIONS – 2 ...................................................................................................... 9-25

Chapter 9 Codes and Standards ................................................................................................................. 9-1

9.1 Introduction ..................................................................................................................................... 9-1

9.2 NEC Synopsis .................................................................................................................................... 9-1

General: Article 100 ........................................................................................................................... 9-1

Identification of grounded conductors: Article 200........................................................................... 9-1

Branch circuits: Article 210 ................................................................................................................ 9-1

Feeders: Article 215 ........................................................................................................................... 9-1

Branch-circuit, feeder, and service calculations: Article 220 ............................................................. 9-2

Overcurrent protection: Article 240 .................................................................................................. 9-2

Grounding: Article 250 ....................................................................................................................... 9-2

Conductors for general wiring: Table 310.16 ff ................................................................................. 9-2

Motors: Article 430 ............................................................................................................................ 9-2

Tables: Chapter 9 ............................................................................................................................... 9-2

Examples: Annex D ............................................................................................................................. 9-3

9.3 Motor Installation Tables ................................................................................................................. 9-4

NEC 240.6(A) Standard Ampere Ratings for Fuses & Circuit Breakers ........................................... 9-4

Table 310.16 Allowable Ampacities of Insulated Conductors Rated 0 Through 2000 Volts, 60°C

Through 90°C (140°F Through 194°F), Not More Than Three Current-Carrying Conductors in

Raceway, Cable, or Earth (Directly Buried), Based on Ambient Temperature of 30°C (86°F) ........... 9-5

Table 310.15(B)(2)(a) Adjustment Factors for More Than Three Current-Carrying Conductors in a

Raceway or Cable ............................................................................................................................... 9-6

Table 310.15(B)(6) Conductor Types and Sizes for 120/240-Volt, 3-Wire, Single-Phase Dwelling

Services and Feeders. ......................................................................................................................... 9-6

Table 430.7(B) Locked-Rotor Indicating Code Letters .................................................................... 9-8

Table 430.52 Maximum Rating or Setting of Motor Branch-Circuit Short-Circuit and Ground-Fault

Protective Devices .............................................................................................................................. 9-8

Page 12: Dr. Robert A. Durham, PhD, PE Dr. Marcus O. Durham, PhD, PE … Nutshell Master.pdf · Dr. Robert A. Durham, PhD, PE . Dr. Marcus O. Durham, PhD, PE . A Quick Reference . The book

Table 430.91 Motor Controller Enclosure Selection ...................................................................... 9-9

Table 430.248 Full-Load Currents in Amperes, Single-Phase Alternating-Current Motors ......... 9-11

Table 430.250 Full-Load Current, Three-Phase Alternating-Current Motors .............................. 9-12

Table 430.251(A) Conversion Table of Single-Phase Locked- Rotor Currents for Selection of

Disconnecting Means and Controllers as Determined from Horsepower and Voltage Rating ....... 9-13

Table 430.251(B) Conversion Table of Polyphase Design B, C, and D Maximum Locked-Rotor

Currents for Selection of Disconnecting Means and Controllers as Determined from Horsepower

and Voltage Rating and Design Letter.............................................................................................. 9-14

Table 8 Conductor Properties ...................................................................................................... 9-15

Table 9 Alternating-Current Resistance and Reactance for 600-Volt Cables, 3-Phase, 60 Hz, 75°C

(167°F) — Three Single Conductors in Conduit ............................................................................... 9-16

Table C.4 Maximum Number of Conductors or Fixture Wires in Intermediate Metal Conduit (IMC)

(Based on Table 1, Chapter 9) .......................................................................................................... 9-18

NEMA Controller Size for Motors, Transformers, & Capacitors ...................................................... 9-21

NEMA Table 11 Typical Characteristics and Applications of Fixed Frequency Small and Medium AC

Squirrel-Cage Induction Motors ....................................................................................................... 9-23

NEMA MOTOR DIMENSIONS ........................................................................................................... 9-24

NEMA MOTOR DIMENSIONS – 2 ...................................................................................................... 9-25

Page 13: Dr. Robert A. Durham, PhD, PE Dr. Marcus O. Durham, PhD, PE … Nutshell Master.pdf · Dr. Robert A. Durham, PhD, PE . Dr. Marcus O. Durham, PhD, PE . A Quick Reference . The book

Page | 1-1

Chapter 1 - Circuit Elements and Analysis

1.1 Introduction The book is an outline and review of electrical engineering. The material is designed with four

purposes.

A Study Guide to provide assistance with preparation for professional examinations.

A Reference for investigating those occasional problems outside your regular specialty.

A Curriculum for professional classes for maintaining your license.

A Synopsis for university students in electrical engineering and other technical disciplines.

A generic electrical system covers equipment from a generator or power supply through controls to

a motor or load. In some problems, the system is analyzed as a whole. In small signal analysis, models

are employed.

Electrical systems always convert an available energy source to electrical energy. The electricity is

then conveniently transferred to a load which converts the electrical energy back to another energy

form.

Electrical systems, as all physical systems, operate based on the Trinity Principle which states: Any

item than can be uniquely identified can be further explained by three components. A corollary states:

Two of the three components that identify a system are similar and project into one plane, while the

third component is dissimilar and operates in an orthogonal plane.

The necessary terms for an electrical system can be identified using this grouping of three

quantities.

In a system, only 3 things are measured: voltage (V), current (I), and time (t)

i = through = flow rate = Amps = dq

dt

v = across = potential = Volts = 2 1v v

t = time

GENERATOR METER TRANSFORMER CONTROLLER MOTOR

Page 14: Dr. Robert A. Durham, PhD, PE Dr. Marcus O. Durham, PhD, PE … Nutshell Master.pdf · Dr. Robert A. Durham, PhD, PE . Dr. Marcus O. Durham, PhD, PE . A Quick Reference . The book

Page | 1-2

From these three measurements, only three things can be calculated.

Z = impedance = ratio = Ohms (Ω) V

I

S = power = product = Watts (w) *VI =dE

dt

= angle = time diff = degrees (seconds) = 2 1t t

Page 15: Dr. Robert A. Durham, PhD, PE Dr. Marcus O. Durham, PhD, PE … Nutshell Master.pdf · Dr. Robert A. Durham, PhD, PE . Dr. Marcus O. Durham, PhD, PE . A Quick Reference . The book

Page | 1-3

1.2 Circuit Element Description & Construction Impedance has only three linear (passive) elements available in a circuit: resistor, inductor, and

capacitor

1.2.1 Resistor

A resistor converts electrical energy into mechanical energy in the form of heat.

Resistance is measured in ohms (Ω). The voltage across a resistor, VR, is the product

of the resistance and the current through the resistor.

RV Ri

A resistor is simply a length of conductive material. The resistance of a piece of material is given by

T

lR

A

Where

T - resistivity of construction material

l is length of material, A is cross-sectional area

The resistivity of the construction material, T , is

temperature dependent.

20 201 20T T

1.2.2 Inductor

An inductor converts electrical energy into magnetic energy. The inductor stores

energy in a magnetic field. The inductance is measured in Henry (H). Voltage across an

inductor is the product of the inductance and the derivative of the current through the

inductor

L

diV L

dt

The energy stored in an inductor comes from

212LdW Vdq W Li

Because of the magnetic fields, the current in an inductor cannot change

instantly.

At its most basic form, an inductor is a coil of wire wrapped around a

closed magnetic path. The inductance created is calculated from

2 2N A NL

l

R

n-# of

turns

l – average path length

820 1.8 10cu 3

20 3.9 10cu

620 1.08 10nichrome 5

20 17 10nichrome

R

+ VR - i

+ VL -i

L

Page 16: Dr. Robert A. Durham, PhD, PE Dr. Marcus O. Durham, PhD, PE … Nutshell Master.pdf · Dr. Robert A. Durham, PhD, PE . Dr. Marcus O. Durham, PhD, PE . A Quick Reference . The book

Page | 1-4

Where

μ – permeability of magnetic path

r o

74 10 /o henry m

1.2.3 Capacitor

A Capacitor stores electrical energy in an electric field. The capacitance is measured in Farads (F).

The voltage across a capacitor is

1 1cv dq i dt

C C

The current through a capacitor is found from

cdvi C

dt

The amount of energy stored in a capacitor is shown.

212L cdW Vdq CVdv W CV

Because of the electric field, the voltage across a capacitor (vc) cannot change instantaneously.

A capacitor is constructed of two conductors, separated by some dielectric material. The

capacitance generated is a product of the permittivity and the size of the conductors.

AC

l

Where

A – cross-sectional area of conductors

l – separation of conductors

ε – permittivity of dielectric material

The permittivity of the dielectric material is referenced to the permittivity of free space

o r

Where

o - permittivity of free space - 9110

36 ; r - dielectric constant (relative permittivity)

(copper) 2r

(amorphous steel) 2000r

(laminated steel) 6000r

( ) 5 10r glass

( ) 16r germanium

(titaniumdioxide) 86 173r

+ VC -i

C

Page 17: Dr. Robert A. Durham, PhD, PE Dr. Marcus O. Durham, PhD, PE … Nutshell Master.pdf · Dr. Robert A. Durham, PhD, PE . Dr. Marcus O. Durham, PhD, PE . A Quick Reference . The book

Page | 1-5

1.3 Circuit Basics

1.3.1 Element Manipulation

Because of circuit combinations, reciprocal values of elements often arise. Where impedance (Z) is

opposition to current flow, the reciprocals of impedance indicate the ease of current flow. Impedance is

made up of two components, the resistance (real portion) and the reactance, which is the combination

of inductance and capacitance (imaginary portion). Impedance is the sum of the resistance, and the

reactance*j.

Z R jX

Admittance (Y) is the reciprocal of impedance, and is measured in mhos ( ) 1

YZ

Conductance (G) is the reciprocal of resistance: 1

GR

Susceptance (GB) is the reciprocal of reactance: 1

BGX

Elastance is the reciprocal of capacitance

Reluctance ( ) is the reciprocal of inductance: 1

L

1

1

1

1

B

Y mhoZ

G mhoR

G siemens SX

amp turns

L weber

Impedance is the sum of resistance and reactance: Z R jX

Admittance is the sum of conductance and susceptance: BY G jG

Page 18: Dr. Robert A. Durham, PhD, PE Dr. Marcus O. Durham, PhD, PE … Nutshell Master.pdf · Dr. Robert A. Durham, PhD, PE . Dr. Marcus O. Durham, PhD, PE . A Quick Reference . The book

Page | 1-6

1.3.2 Element Combinations

The elements defined in the previous topics can be combined into combinations of impedances. The

configuration of the combinations determines the mathematical treatment of the impedances (Z). The

bases of all combinations are the series and parallel connections.

Series Impedances are added,

1 2 3totalZ Z Z Z

For resistors, the impedance is equal to the resistance, RZ R

1 2 3R totalZ R R R

The impedance of inductors is frequency dependant and is proportional to the inductance, LZ L .

Series inductors at the same frequency are treated the same as resistances.

1 2 3( )L totalZ L L L

Capacitors are a different animal. The impedance of capacitors is frequency dependant, but is

inversely proportional to the capacitance, 1

CZC

. For this reason, when capacitors are in series, the

series resistance is the sum of the elastances, or 1

C

1 2 3

1 1 1C totalZ

C C C

To calculate the impedance of elements in parallel, sum the reciprocals of the impedances, or the

admittance of each element. This gives the admittance of the entire circuit. This

is the reciprocal of the impedance of the circuit.

1 2 3

1 1 1 1total

total

YZ Z Z Z

The special case of two parallel impedances can be found by taking the product of the impedances

and dividing by the sum of the impedances.

1 2

1 2

total

Z ZZ

Z Z

For resistors, the admittance of the circuit is the sum of the conductances of the individual

elements.

1 2 3

1 2 3

1 1 1 1

R total

G G GZ R R R

For inductors, the admittance of the circuit is proportional to the sum of the reluctance of the

individual inductors.

Z1 Z2 Z3

Z1 Z2 Z3

Page 19: Dr. Robert A. Durham, PhD, PE Dr. Marcus O. Durham, PhD, PE … Nutshell Master.pdf · Dr. Robert A. Durham, PhD, PE . Dr. Marcus O. Durham, PhD, PE . A Quick Reference . The book

Page | 1-7

1 2 3

1 1 1 1

L totalZ L L L

Again, capacitors are a special case. For capacitors at the same frequency, the admittance is

proportional to the sum of the capacitances.

1 2 3

1

C total

C C CZ

1.3.3 Sources

There are is a lower and upper limit on impedance. A short circuit is near zero impedance. An open

circuit is near infinite impedance. The source is the limit on voltage or current.

A short Circuit is defined as v=0 for any i

i

v=0

An open Circuit is defined as i=0 for any v

i=0

v+

A voltage source maintains v=Vo for any i

+v0

i

Current sources maintain i=Io for any v

I0

DC or constant sources are represented by

DCor

AC sources where cosov V t are

represented by

AC

Page 20: Dr. Robert A. Durham, PhD, PE Dr. Marcus O. Durham, PhD, PE … Nutshell Master.pdf · Dr. Robert A. Durham, PhD, PE . Dr. Marcus O. Durham, PhD, PE . A Quick Reference . The book

Page | 1-8

1.3.4 Circuit Laws

Two circuit laws are used in all circuit analysis. These circuit laws adhere to the Conservation of

Energy – There is nothing new under the sun; or, more traditionally, the sum of the energy in a closed

system is zero

0E

pqE

t

pv

t

qi

t

Kirchhoff’s Current Law (KCL) relates to currents entering a node. KCL states that the sum of

currents entering a node is equal to zero (0).

i1

i2

i3

i4

1 2 3 40 0ni i i i i

By convention, if current enters a node, it is considered negative. Current leaving a node is

considered positive.

Kirchhoff’s Voltage Law (KVL) relates to voltages in a circuit loop. KVL states that the sum of voltages

in a loop is equal to zero (0)

1 20 0n R Lv v v v v

1 2

diV V iR L

dt

By convention, if current goes into “+” of voltage source, then the

voltage is considered positive. If current goes into “–“ of voltage

source, then the voltage is considered negative.

V1

RL

V2

i

VLVR

Page 21: Dr. Robert A. Durham, PhD, PE Dr. Marcus O. Durham, PhD, PE … Nutshell Master.pdf · Dr. Robert A. Durham, PhD, PE . Dr. Marcus O. Durham, PhD, PE . A Quick Reference . The book

Page | 1-9

1.4 Circuit Analysis Five rules can aid in any circuit analysis.

Assume a consistent group of currents & voltages for each element (R, L, C)

Conservation: Write equations using Kirchhoff’s Law (KCL or KVL)

Use element definitions (Ohm’s Law)

Combine equations in terms of unknowns

Solve simultaneously using elimination or Cramer’s rule

Circuit analysis generally has a source (external forcing function) and elements (opposition). The

“answer” to a circuits analysis problem is voltage & current across an element, or a derivative such as

power or energy.

Three methods of solving circuit analysis

Loop current

Node voltage

Equivalent circuits

Page 22: Dr. Robert A. Durham, PhD, PE Dr. Marcus O. Durham, PhD, PE … Nutshell Master.pdf · Dr. Robert A. Durham, PhD, PE . Dr. Marcus O. Durham, PhD, PE . A Quick Reference . The book

Page | 1-10

1.4.1 Loop Current Method

Assign loops – “window pane” method

Assume voltage polarity (+ into Z)

Write KVL around loop

Substitute Ohm’s law (element) for the voltage across the Z

Solve for unknowns

Example

6V

10Ω

8Vi1 i2

Find the current in the 10Ω resistor

KVL Left Loop

6 2 10 0VV V V KVL left loop

2 12V i Ohm’s law

10 1 210( )V i i

1 1 26 2 10( ) 0i i i Substitute

KVL Right Loop

8 10 4 0VV V V KVL right loop

4 24V i Ohm’s law

10 1 210( )V i i

2 1 28 10( ) 4 0i i i Substitute

1 2

1 2

12 10 6

10 14 8

i i

i i

Solve equations simultaneously

Page 23: Dr. Robert A. Durham, PhD, PE Dr. Marcus O. Durham, PhD, PE … Nutshell Master.pdf · Dr. Robert A. Durham, PhD, PE . Dr. Marcus O. Durham, PhD, PE . A Quick Reference . The book

Page | 1-11

Cramer’s Rule

If solving for in – substitute RHS for in in numerator

Use matrix of coefficients in denominator

Manipulate using cross-multiplication

1

2

6 10

8 14 (6)(14) ( 10)( 8) 40.059

12 10 (12)(14) ( 10)( 10) 68

10 14

0.529

i A

i A

Now calculate the desired value.

10 1 2 0.588i i i A

Page 24: Dr. Robert A. Durham, PhD, PE Dr. Marcus O. Durham, PhD, PE … Nutshell Master.pdf · Dr. Robert A. Durham, PhD, PE . Dr. Marcus O. Durham, PhD, PE . A Quick Reference . The book

Page | 1-12

1.4.2 Node Voltage Method

Assign node voltage @ each connection (Ground = ref = 0V)

Assume branch current direction (e.g. always leaves node)

Write KCL @ each node

Substitute elements (Ohm’s law) for current

Solve

Example

6V

10Ω

8V

a cb

Find the current in the 10Ω resistor

KCL @ node b

Assume all current leaves node

2 10 4 0i i i KCL node b

2

6

2 2

b a bV V Vi

Ohm’s Law

10

4

0

10

8

4 4

b

b c b

Vi

V V Vi

6 0 80

2 10 4

b b bV V V

Substitute

60 10 40 5 2 0b b bV V V Solve equation

17 100

5.88

b

b

V

V v

Now calculate desired value

10

0 5.880.588

10 10

bV vi A

Page 25: Dr. Robert A. Durham, PhD, PE Dr. Marcus O. Durham, PhD, PE … Nutshell Master.pdf · Dr. Robert A. Durham, PhD, PE . Dr. Marcus O. Durham, PhD, PE . A Quick Reference . The book

Page | 1-13

1.4.3 Equivalent Circuit Method

Three types of equivalent circuit analysis can be used to reduce problem complexity. These are

combining series impedance, combining parallel impedance, and using ratios of impedance to divide

current or voltage.

For series impedances, use KVL - the current is the same, so sum

voltages in the path.

1 2

1

2

1 1

1 2

1 1 2

1 2

( )

R R

R

R

eq

eq

v v v

v i R

v i R

v i R R iR

R R R

For parallel impedances, use KCL – the voltage is the same,

so sum the current.

1 2

1

1

2

2

1 2

1 2

1 2 1 2

1 1( )

R

1 1 1

eq

eq

eq

i i i

vi

R

vi

R

vi v

R R

R RR

R R R R R

Voltage or current dividers are used to determine voltage across or current through a circuit

element.

A current divider is used in parallel circuits with the same voltage applied on the branches

21

1 2

( )Ropposite

i i iR R

A voltage divider is used in series circuits with same current in elements

1

adjacentv v

R1 R2

i1

R1

R2

i

i1

i2

Page 26: Dr. Robert A. Durham, PhD, PE Dr. Marcus O. Durham, PhD, PE … Nutshell Master.pdf · Dr. Robert A. Durham, PhD, PE . Dr. Marcus O. Durham, PhD, PE . A Quick Reference . The book

Page | 1-14

1.5 Single Source and Impedance Thevenin & Norton equivalents are used to simplify any circuit to an equivalent impedance and a

single source (driver). These are calculated by taking the sources to the limit. L is manipulated the same

as R, C in series is manipulated like R in parallel. The following rules help in finding the Thevenin or

Norton equivalent circuits.

Find Zeq

Replace independent V sources by short

Replace independent I sources by open

Calculate resistance – use series / parallel rules

If the voltage supplied at the node and the current into the node are known, then V

ZI

Find Vth

Open circuit terminals

Leave sources active

Calculate the voltage across the open terminal. This will be the voltage across an impedance.

Find Is

Short circuit terminals

Leave sources active

Calculate the current through the shorted terminal. This will be the current that shorts (bypasses) an

impedance

Page 27: Dr. Robert A. Durham, PhD, PE Dr. Marcus O. Durham, PhD, PE … Nutshell Master.pdf · Dr. Robert A. Durham, PhD, PE . Dr. Marcus O. Durham, PhD, PE . A Quick Reference . The book

Page | 1-15

Illustration

R1

R2

1

2

v

Replace voltages

R1

R2

1

2

1 2

1 2

eq

R RR

R R

Vth = voltage across R2 (Open terminals and calculate v)

R1

1

2

VTH

v1

Series circuit with voltage divider

2

1 2

th

RV v

R R

Alternatively

1 2

22

1 2

th

vi

R R

vRV iR

R R

Isc=I through terminals → R2=∞, it is bypassed, and Isc= I in series circuit

Page 28: Dr. Robert A. Durham, PhD, PE Dr. Marcus O. Durham, PhD, PE … Nutshell Master.pdf · Dr. Robert A. Durham, PhD, PE . Dr. Marcus O. Durham, PhD, PE . A Quick Reference . The book

Page | 1-16

R1

1

2

ISC

v1

1

SC

vI I

R

Check

2

1 2 1 2

1 2

1

THeq

SC

Rv

R RV R RR

vI R R

R

Page 29: Dr. Robert A. Durham, PhD, PE Dr. Marcus O. Durham, PhD, PE … Nutshell Master.pdf · Dr. Robert A. Durham, PhD, PE . Dr. Marcus O. Durham, PhD, PE . A Quick Reference . The book

Page | 1-17

Example

6v

10Ω

8v

Equivalent to

6v

10Ω

8v

Convert

2Ω 10Ω4Ω3A 2A

Simplify

4/3 Ω 10Ω3A+2A=5A

i

2 / /4

2* 4 4

2 4 3

THR

Current Divider for 10Ω

4

35 0.5884

103

i A A

Page 30: Dr. Robert A. Durham, PhD, PE Dr. Marcus O. Durham, PhD, PE … Nutshell Master.pdf · Dr. Robert A. Durham, PhD, PE . Dr. Marcus O. Durham, PhD, PE . A Quick Reference . The book

Page | 1-18

1.6 Steady State AC Analysis Steady state AC analysis is used when the source is operating at a constant frequency. Because of

inductors and capacitors, an angle change or phase shift is created in the voltage and current drops

associated with the impedance.

Frequency ( f ) is related to the time it takes for a waveform to repeat. It is the number of

repetitions or cycles in a second (Hz).

1

2f

T

where

ω = angular frequency in radians per second

On many power systems the frequency is fixed or constant for normal, or steady

state operations. The phasor transform can be applied to provide a concise phasor

notation at the fixed frequency.

P *Vpeak cos (ωt + θ)+ = VRMSθ

2peak RMSV V

where

θ = phase shift between voltage and current cause by L and C.

Because of the phase shift, complex numbers are often used to manipulate the mathematics.

Complex numbers involve both a real and imaginary part. The imaginary part is defined as multiplied by

“j” which is referred to as the imaginary unit.

;

Euler’s formula allows conversion of rectangular coordinates to polar. It gives a representation of

the angle impact on the sinusoids.

cos sinje j

cos2

j je e

sin2

j je e

j

Time varying voltage (AC) is defined based on the magnitude and angles described above.

21j

j e

21 j

j ej

V

θ

Page 31: Dr. Robert A. Durham, PhD, PE Dr. Marcus O. Durham, PhD, PE … Nutshell Master.pdf · Dr. Robert A. Durham, PhD, PE . Dr. Marcus O. Durham, PhD, PE . A Quick Reference . The book

Page | 1-19

( 2 )j j te RMSv R V e e

2 cos( )v V t

Impedance is the phasor representation of resistors, inductors, and capacitors.

Z R jX

V IZ

For a resistor

RV IR

RZ R

For an Inductor

LV j LI

LZ j L

2LZ j fL

For a Capacitor

C

IV

j C

1CZ

j C

2

C

jZ

fC

The combination of resistor, inductor, and capacitor can be shown in a phasor of voltages. In series

circuits, the current will be the same in all elements, so the impedance is proportional to the voltage. In

a parallel circuit the voltage is the same across all elements, so the impedance is proportional to the

current.

VC VR

VL V

I

R

+ VR - i

+ VL -i

L

+ VC -i

C

+ VL -

iLR C

+ VR - + VC -

+V

Page 32: Dr. Robert A. Durham, PhD, PE Dr. Marcus O. Durham, PhD, PE … Nutshell Master.pdf · Dr. Robert A. Durham, PhD, PE . Dr. Marcus O. Durham, PhD, PE . A Quick Reference . The book

Page | 1-20

1.7 Electric Fields – Electrostatics

W(Energy) Fs Vq NI energy conversion

(Electricintensity) /F V

V meterq s

E (closed loop) 2s r

2coulomb(electric density)

qE

A m D

q = charge 1 electron = 1.6021x10-19 Coulomb

Permittivity is the dielectric or charge insulation material property.

r o

9136 10

o

Force on charge 2 due to charge 1

1 2

24

q qF

r

Electric field intensity at point 2 due to point charge at point 1.

1

24

qF

q r E

Radial electric field due to line charge on z-axis

( / )

2

L C m

r

E

coulombcharge density -

meterL

Electric field due to sheet charge in x-y plane

2( / )

2

s C m

E

2

coulombsheet charge density -

meters

Energy is dependent on the electric field, which can be converted to voltage.

W = q E l = ½ ε E2

W = qV = ½ CV2

Page 33: Dr. Robert A. Durham, PhD, PE Dr. Marcus O. Durham, PhD, PE … Nutshell Master.pdf · Dr. Robert A. Durham, PhD, PE . Dr. Marcus O. Durham, PhD, PE . A Quick Reference . The book

Page | 1-21

1.8 Magnetic Fields

The fundamental relationships for magnetic devices are listed.

(magneto-motive force) mmf = amp-turns voltageNI H dl F R=

F(Magneticintensity) /

NIAmp meter

s H

(closed loop) 2 ( )s r S dL

2

NI

RH

2weber(field density) H

A m

B

2

(reluctance)l N

A L R

Permeability is the magnetic property of material.

r o

74 10 /o henry m

(copper) 2r

(amorphous steel) 2000r (laminated steel) 6000r

Force on pole 2 due to pole 1

1 2

24F

r

(fluxlinkage) N LI

The magnetic circuit for a machine includes the current and turns, the ferromagnetic metal, and air

gaps.

Cross sectional area of air is ~5% more than steel.

The magnetization curve shown below is a non-linear relationship for the magnetic circuit, and is

typical of all magnetic circuits. It is used to show the conversion between representations of magnetic

energy.

i

N

l

Air gap

lg

A – cross

sectional area

Page 34: Dr. Robert A. Durham, PhD, PE Dr. Marcus O. Durham, PhD, PE … Nutshell Master.pdf · Dr. Robert A. Durham, PhD, PE . Dr. Marcus O. Durham, PhD, PE . A Quick Reference . The book

Page | 1-22

The first portion of the curve has a physical anomaly near zero. The portion less than 5000 is in the

unsaturated region. There is an approximate proportional change in the vertical axis as the horizontal

changes. About 5000 is called the knee. That is the transition region. The top portion of the curve above

about 5000 is the saturated region. There is very little change in the vertical parameter as the horizontal

is increased.

The values are strictly representative. Different material alloys will yield other range of values.

Nevertheless, the general shape and form can be used for a variety of problems.

The curve then can represent a number of different relationships. Some of the more common are

shown in the table below.

X name unit Y name Units Function name unit

F mmf A-turns flux Weber R reluctance

H intensity A-turns/m B density Wb/m2 µ permeability H/m

I field I Amps V terminal Volts synchronous

F field mmf A-turns a Internal gen Volts dc

The location and direction of a magnetic field are determined by the configuration of the

conductor. Note that the intensity and density are related by the permeability μ. Therefore,

either can be determined from these equations.

Magnetic flux lines are continuous about a source, and perpendicular at all points to source,

in a parallel plane

2

I

rH

A magnetic field is produced by straight conductor carrying

current I

0

50

100

150

200

250

300

350

0 2000 4000 6000 8000 10000 12000

I

r

θ1 θ2

r

B

I

Page 35: Dr. Robert A. Durham, PhD, PE Dr. Marcus O. Durham, PhD, PE … Nutshell Master.pdf · Dr. Robert A. Durham, PhD, PE . Dr. Marcus O. Durham, PhD, PE . A Quick Reference . The book

Page | 1-23

Ffield direction follows the right hand rule.

( )F i l B

Thumb – B

First Finger – F

Middle Finger – I, l

Cross product vector directions are determined by the curl of

the right hand.

The motor relationship comes from a current flow through a

field.

The generator relationship comes from a wire moving

through a field.

Energy is dependent on the magnetic field, which can be

converted to current.

W = H l φ = ½ μ D2

W = Iφ = ½ LI2

1 2(sin sin )4

I

r

B

( )F i l B

(voltage)e v l B ,

B

I

F

Page 36: Dr. Robert A. Durham, PhD, PE Dr. Marcus O. Durham, PhD, PE … Nutshell Master.pdf · Dr. Robert A. Durham, PhD, PE . Dr. Marcus O. Durham, PhD, PE . A Quick Reference . The book

Page | 1-24

1.9 Maxwell

Maxwell’s Equations are a summary of all that is electrical and magnetic in a Calculus form. Although

they are not easily used, they do provide a mathematically clever summary. The integral form is tedious,

but the point form supplies definitions.

0

BEt

DH Jt

D

E

The electric-magnetic energy equation contains all the information in one equation. [1] Note that all

the definitions are in the point equation. In addition, all the field information is in the distributed

equation. W is energy and V is volume in these equations.

z yqW

t

z y ys t y

r y

q b d sW

t V

*1+ “Applications Engineering Approach to Maxwell and Other Mathematically Intense Problems“,

Marcus O. Durham, Robert A. Durham, and Karen D. Durham, Institute of Electrical and Electronics

Engineers PCIC, September 2002.

Page 37: Dr. Robert A. Durham, PhD, PE Dr. Marcus O. Durham, PhD, PE … Nutshell Master.pdf · Dr. Robert A. Durham, PhD, PE . Dr. Marcus O. Durham, PhD, PE . A Quick Reference . The book

Page | 1-25

Chapter 1 Problems

Practice Problem 1-1 (Old Style)

SITUATION:

The circuit shown in the figure below is the pi representation of a transmission line.

REQUIREMENTS:

Write a set of nodal equations for the circuit the “Ref” as the reference or common terminal.

Solve the equations of requirement (a) above for Vo if 100 0inV .

Vin

Ri=50Ω

-j50Ω

j100Ω

-j50Ω 50Ω

Ref

+

Vo

-

SOLUTION:

Simple circuits problem

a) Nodal equations use current – convert voltage sources to current sources.

50 -j50 -j50 50

a b

Vout

inV

50

j100

I1 I2 I4 I5

I3

1

0

50

avI

2

0

50

avI

j

3100

a bv vI

j

4

0

50

bvI

j

5

0

50

bvI

Page 38: Dr. Robert A. Durham, PhD, PE Dr. Marcus O. Durham, PhD, PE … Nutshell Master.pdf · Dr. Robert A. Durham, PhD, PE . Dr. Marcus O. Durham, PhD, PE . A Quick Reference . The book

Page | 1-26

KCL @ a KCL @ b

1 2 350

50 50 50 100 100

(2 1) 1 2

in

in a a a b

a b in

VI I I

V v v v v

j j j

j v j v V

3 4 5 0

0100 100 50 50

1 (2 1) 0

a b b b

a b

I I I

v v v v

j j j

j v j v

b outv v

Solve using Cramer’s Rule – Substitute RHS for vb

2 1 2

1 0 2 2 90

2 1 1 4 4 1 1 4 2 45

1 2 1

in

in inout

j V

j j V Vv

j j j

j j

0.3535 135*out inv V

b) For Vin=1000

100 0*0.3535 135 35.35 135outv

Page 39: Dr. Robert A. Durham, PhD, PE Dr. Marcus O. Durham, PhD, PE … Nutshell Master.pdf · Dr. Robert A. Durham, PhD, PE . Dr. Marcus O. Durham, PhD, PE . A Quick Reference . The book

Page | 1-27

Practice Problem 1-2 (Old Style)

SITUATION:

A starting circuit is needed that will limit the starting current in a dc motor to two and one half time

(2.5pu) the normal full load current.

The switches S1 and S2 in the starting circuit shown below are to close sequentially when the current

has dropped to normal full load current (1pu)

Both switches are open when the main breaker SM is closed.

Sm

EA

RA R1 R2

S1 S2

Vt

SOLUTION:

On Closing of Sm

1 2

1 2

2.5 0.4t tA A

A A

V VI pu R R R pu

R R R I

With Vt=1.0,

1 2

10.4

2.5

tT A

A

VR R R R pu

I pu

t At a A t t

A

V EV E I R R

I

When IA drops to 1.0 pu

1(0.4 ) 0.4 1 0.4 0.6a A tVt E pu E V pu

Close S1 and IA raises to 2.5pu, at that instance EA = 0.6pu

2

1 0.60.16

2.5

t aT A

A

V ER R R pu

I

When IA again drops to 1.0 pu

Page 40: Dr. Robert A. Durham, PhD, PE Dr. Marcus O. Durham, PhD, PE … Nutshell Master.pdf · Dr. Robert A. Durham, PhD, PE . Dr. Marcus O. Durham, PhD, PE . A Quick Reference . The book

Page | 1-28

1 1*0.16 0.84A t A TE V I R pu

Close S2 and IA raises to 2.5pu, at that instance EA = 0.84pu

1 0.84

0.0642.5

t aT A

A

V ER R pu

I

When IA again drops to 1.0 pu

1 1*0.064 0.936A t A TE V I R pu

2 2

2 1 1

0.064

0.16 0.096

0.4 0.24

A

A

A

R pu

R R pu R pu

R R R pu R pu

Signals that are encountered can be a constant, direct current (DC); they can be repetitive,

alternating current (AC); or they can be short term, transients. The circuit elements respond differently

to each type signal. This chapter will address waveforms and tools to analyze their impact on systems

performance. The time domain signal response or solution contains all the components.

Page 41: Dr. Robert A. Durham, PhD, PE Dr. Marcus O. Durham, PhD, PE … Nutshell Master.pdf · Dr. Robert A. Durham, PhD, PE . Dr. Marcus O. Durham, PhD, PE . A Quick Reference . The book

Page | 2-1

Chapter 2 Waveforms

2.1 Introduction Signals that are encountered can be a constant, direct current (DC); they can be repetitive,

alternating current (AC); or they can be short term, transients. The circuit elements respond differently

to each type signal. This chapter will address waveforms and tools to analyze their impact on systems

performance. The time domain signal response or solution contains all the components.

2.2 Waveforms By far, the sinusoid is the most common

repetitive waveform in electrical systems. It is the

physical result due to the rotational motion of

machines in a magnetic field.

The waveform definitions follow.

cosovoltage v V t

1( )frequency f hertz

T

2 ( / sec)f radians

0

10for sinusoid

T

DCaveragevalue V dtT

2

0

1

2

T

oRMS

Veffectivevalue V v dt

T

For multiple waveforms, use superposition. For effective or root mean square (RMS), this is square

root of the sum of the squares.

Generally, AC values of V & I are given in RMS. The frequency is assumed constant.

For a 100Volt, 60Hz voltage waveform

2 2

1 2 ...RMS RMS RMSV V V

100 2 cos(2 60 )v t

100 2oV

141OV volts

Vo = peak

Vrms = effective

v(V)

Time

T (sec)

Page 42: Dr. Robert A. Durham, PhD, PE Dr. Marcus O. Durham, PhD, PE … Nutshell Master.pdf · Dr. Robert A. Durham, PhD, PE . Dr. Marcus O. Durham, PhD, PE . A Quick Reference . The book

Page | 2-2

2.3 Transients Transients are waveforms that exist for a short period of time. Waveforms are determined by the

circuit elements. Since there are only three elements, the most complex circuit is a second order. The

characteristic solution for a systems circuit is the time varying equation that describes the exponential

decay after a signal is applied. The variable, y, can represent either current or voltage.

( ) ( ) cost

y t F I F e t

where

F = final value (t=∞)

I = Initial Value (t=0)

τ = time constant

2.3.1 First Order Transients

First order systems are very common, since they are the model of a simple system. First order

systems have a resistor and either a capacitor or an inductor.

First Order Circuits

RC or RL

Form:

div L Ri

dt

dv vi C

dt R

Characteristic Solution

Response to a step input (DC)

( ) ( )t

y t F I F e

1LC

Page 43: Dr. Robert A. Durham, PhD, PE Dr. Marcus O. Durham, PhD, PE … Nutshell Master.pdf · Dr. Robert A. Durham, PhD, PE . Dr. Marcus O. Durham, PhD, PE . A Quick Reference . The book

Page | 2-3

For Capacitor: dv

i Cdt

Voltage does not change instantaneously

Open circuit under DC conditions

Capacitor discharges to ( ) 0CV

Initial voltage = source voltage

For Inductor: di

v Ldt

Current does not change instantaneously

Short circuit under DC conditions

Inductor dissipates to ( )LI I

Initial current = source current

Process:

Find τ

-1

time constant= or /

1 time for exponent to be e

RC L R

e

use equivalent circuit w/o source to get RC or RL (Thevenin Impedance)

deactivate all the sources and replace with internal Z

reduce to single equivalent RC or RL

Find y(0)

use circuit (KVL) w/ element as source

Find y(final)

use circuit (KVL) w/ element as limit

Plot: Initial slope = F I

Transfer function = response

excitation=

output

input

Page 44: Dr. Robert A. Durham, PhD, PE Dr. Marcus O. Durham, PhD, PE … Nutshell Master.pdf · Dr. Robert A. Durham, PhD, PE . Dr. Marcus O. Durham, PhD, PE . A Quick Reference . The book

Page | 2-4

2.3.2 RL Circuits

Standard calculus form

o

diV L Ri

dt

Inductor is short circuit in final state.

0( ) 0i L

( ) of

Vi L

R

L

R

General solution

( ) ( )t

y t F I F e

Current solution

(0 )

t

o oV Vi e

R R

(1 )

Rt

o LV

i eR

Stability reached after 3 – 5 time constants

i

Sec

oV

R

diL

dt

L

R

VOR

t=0

i

L

Page 45: Dr. Robert A. Durham, PhD, PE Dr. Marcus O. Durham, PhD, PE … Nutshell Master.pdf · Dr. Robert A. Durham, PhD, PE . Dr. Marcus O. Durham, PhD, PE . A Quick Reference . The book

Page | 2-5

2.3.3 RC Circuits Standard calculus form

dv v

i Cdt R

(calculus form)

Capacitor is open circuit in final state.

i, vc cannot change instantaneously

0( ) ov C V

( ) 0fv C

RC

General solution

( ) ( )t

y t F I F e

Voltage solution

( )t

F I Fv V V V e

t

c ov V e

sec

Vc

Vo

RC

vo

R

Ct=0 i

Page 46: Dr. Robert A. Durham, PhD, PE Dr. Marcus O. Durham, PhD, PE … Nutshell Master.pdf · Dr. Robert A. Durham, PhD, PE . Dr. Marcus O. Durham, PhD, PE . A Quick Reference . The book

Page | 2-6

2.4 LaPlace A standard waveform is defined in terms of time and frequency. A mathematical transform is often

used to provide a different mathematical tool. The phasor representation is one transform that applies

to steady state alternating circuits. LaPlace transforms are used for many manipulations of the inductor

and capacitor elements. The function can be transformed from time to the s domain, which represents a

stationary and rotational component.

s j

The most used transform pairs are illustrated.

( )f t ( )f t ( )F s ( )mI s

1 1

s

t 2

1

s

2

te 1

s

( ) tf t e

10.37

e

1

( )f t t

( ) 1f t

Page 47: Dr. Robert A. Durham, PhD, PE Dr. Marcus O. Durham, PhD, PE … Nutshell Master.pdf · Dr. Robert A. Durham, PhD, PE . Dr. Marcus O. Durham, PhD, PE . A Quick Reference . The book

Page | 2-7

tte 2

1

( )s

2

sin t 2 2s

j

j

cos t 2 2

s

s

j

j

sinte t

2 2( )s

j

j

coste t

2 2( )

s

s

j

j

2

1

coste t

sinte t

2

1

-2

-1

0

1

2

2

sin t

-2

-1

0

1

2sin t

2

( ) tf t te 1

e

1

Page 48: Dr. Robert A. Durham, PhD, PE Dr. Marcus O. Durham, PhD, PE … Nutshell Master.pdf · Dr. Robert A. Durham, PhD, PE . Dr. Marcus O. Durham, PhD, PE . A Quick Reference . The book

Page | 2-8

2.5 LaPlace Operational Rules The mathematical manipulation of the time function and the LaPlace transform follows defined

rules.

1 2 1 2( ) ( ) ( ) ( )f t f t F s F s

( ) ( )a f t a F s

( )( ) ( ) (0)

d f tf t sF s f

dt

2( ) ( ) (0) (0)f t s F s sf f

0

1( ) ( )

tf t dt F s

s

Page 49: Dr. Robert A. Durham, PhD, PE Dr. Marcus O. Durham, PhD, PE … Nutshell Master.pdf · Dr. Robert A. Durham, PhD, PE . Dr. Marcus O. Durham, PhD, PE . A Quick Reference . The book

Page | 2-9

2.5.1 Partial Fraction Expansion

1

1 2 1 2

( ) ( ) 2( )

( ) ( )( ) ( ) ( )

KN s N s KF s

D s s p s p s p s p

1

11 1 2 3

1 2 1 3

( )( ) ( )

( )( )s p

N pK s p F s jK jK

p p p p

1 21 2( ) p t p tf t K e K e

Page 50: Dr. Robert A. Durham, PhD, PE Dr. Marcus O. Durham, PhD, PE … Nutshell Master.pdf · Dr. Robert A. Durham, PhD, PE . Dr. Marcus O. Durham, PhD, PE . A Quick Reference . The book

Page | 2-10

2.5.2 Alternate Approach

1 2( ), ( )F s F s are from the table, known transforms

1 1 2 2( ) ( ) ( )F s K F s K F s

Right hand side to common denominator, equate numerators, solve for K1, K2…

1 1 2 2( ) ( ) ( )f t K f t K f t

Impedance in s-domain

Impedance V

ZI

RZ R LZ sL 1

CZsC

Admittance 1 I

YZ V

1

RYR

1

LYsL

CY sC

Page 51: Dr. Robert A. Durham, PhD, PE Dr. Marcus O. Durham, PhD, PE … Nutshell Master.pdf · Dr. Robert A. Durham, PhD, PE . Dr. Marcus O. Durham, PhD, PE . A Quick Reference . The book

Page | 2-11

2.6 Fourier Series Any alternating waveform can be represented by the summation of a fundamental sine wave and its

multiples called harmonics. This summation is called a Fourier series.

0 1 1 2 2sin(1 ) sin(2 ) sin( )n ny Y Y t Y t Y n t

The term y is the instantaneous value at any time. It can be either current or voltage.

The Y0 term is the constant offset, average, or DC component. The Y terms are the maximum

amplitude for each of the harmonic frequencies. The angular frequency ω is 2πf. The phase shift angle

represents the time delay between the reference voltage waveform and the current. The n subscript

and coefficient of frequency indicates the harmonic number.

The time domain is a plot of the Y amplitude versus time for the curve. The frequency spectrum is a

plot of harmonic amplitude versus harmonic frequency number.

An odd function is created with the sum of the odd harmonics. A sine wave is the basic example. If

the waveform has the pattern of a fundamental sine wave, then it is odd.

( ) ( )y t y t

An even function is created with the sum of the even harmonics. A cosine is the basic example. If the

waveform has the pattern of a fundamental cosine wave, then it is even.

( ) ( )y t y t

A function that contains both even and odd harmonics will have spikes. A pulse and sawtooth are

examples.

Page 52: Dr. Robert A. Durham, PhD, PE Dr. Marcus O. Durham, PhD, PE … Nutshell Master.pdf · Dr. Robert A. Durham, PhD, PE . Dr. Marcus O. Durham, PhD, PE . A Quick Reference . The book

Page | 2-12

Page 53: Dr. Robert A. Durham, PhD, PE Dr. Marcus O. Durham, PhD, PE … Nutshell Master.pdf · Dr. Robert A. Durham, PhD, PE . Dr. Marcus O. Durham, PhD, PE . A Quick Reference . The book

Page | 2-13

The very definition of Fourier series indicates the series can take several forms. A cosine is an

orthogonal shift to a sine wave. As a result, a common representation is to use cosine terms for the even

harmonics and sine terms for the odd. Then the even harmonics become odd coefficients for the cosine

terms. Although this is a common representation, it is not as easy to visualize or to obtain a spectrum as

the simple sinusoidal form.

The Fourier series can be decomposed into the sum of even and odd parts.

( ) ( ) ( )e of t f t f t

The even part can be represented by the Fourier series

1

( ) cos( )2

oe n

n

af t a n t

Page 54: Dr. Robert A. Durham, PhD, PE Dr. Marcus O. Durham, PhD, PE … Nutshell Master.pdf · Dr. Robert A. Durham, PhD, PE . Dr. Marcus O. Durham, PhD, PE . A Quick Reference . The book

Page | 2-14

The odd part can be represented by the Fourier series

1

( ) sin( )o n

n

f t b n t

The coefficients are similar.

0

1( )

T

oa f t dtT

0

0

2( )cos( )

T

na f t n t dtT

0

0

2( )sin( )

T

nb f t n t dtT

Page 55: Dr. Robert A. Durham, PhD, PE Dr. Marcus O. Durham, PhD, PE … Nutshell Master.pdf · Dr. Robert A. Durham, PhD, PE . Dr. Marcus O. Durham, PhD, PE . A Quick Reference . The book

Page | 2-15

2.7 Signals - Modulation Modulation is the process of combining two signals into one combined waveform. The combination

can be through an adder or multiplier. The mathematical result looks very much like the Fourier series.

Modulation is similar to wrapping a paper note around a rock and tossing the combination. The

carrier wave, or rock, provides a vehicle for passing the information. The information is on the paper

note.

In its basic form, the carrier is a single waveform.

( ) sin(2 )c c cx t A f t

Ac = amplitude, f = frequency, and θ is the phase shift. Therefore only the amplitude, frequency, or

phase can be changed or modulated.

The message, information, or baseband has a similar form. Usually the message has a fixed or 0

phase shift.

( ) sin(2 )m m mm t A f t

2.7.1 Modulation Types

There are numerous variations to the types of modulation.

There are three analog modulation techniques based on the variables in the waveform.

Amplitude modulation (AM)

Frequency modulation (FM)

Phase modulation (PM)

Special variations of these techniques have unique characteristics that affect bandwidth and power.

Angle modulation includes both frequency and phase modulation, since they are operated

on by the sinusoid.

Double sideband modulation (DSB) is AM with the carrier removed.

Single-sideband modulation (SSB) is DSB with one of the sidebands removed.

There are three fundamental sampling or digital modulation techniques.

Pulse amplitude modulation (PAM)

Pulse frequency modulation (PFM)

Pulse phase modulation (PPM)

Variations of these techniques result in a variety of keying processes. The process of modulation and

demodulation is called a modem.

Pulse code modulation includes both frequency and phase.

Page 56: Dr. Robert A. Durham, PhD, PE Dr. Marcus O. Durham, PhD, PE … Nutshell Master.pdf · Dr. Robert A. Durham, PhD, PE . Dr. Marcus O. Durham, PhD, PE . A Quick Reference . The book

Page | 2-16

Amplitude shift key modulation (ASK)

Frequency shift key modulation (FSK)

Binary-phase shift key modulation (BPSK)

Quadrature-phase shift key modulation (QPSK)

Quadrature amplitude modulation (QAM)

Page 57: Dr. Robert A. Durham, PhD, PE Dr. Marcus O. Durham, PhD, PE … Nutshell Master.pdf · Dr. Robert A. Durham, PhD, PE . Dr. Marcus O. Durham, PhD, PE . A Quick Reference . The book

Page | 2-17

2.7.2 Amplitude Modulation (AM)

Amplitude modulation mixes the information or message with the carrier amplitude. The general

form of amplitude modulation is to add a function of the message to the carrier amplitude.

( ) [ ( )]sin(2 )c a cy t A k m t f t

For a single waveform, ka is unity.

1ak

The amplitude varies with the carrier and the signal. The expanded form illustrates the three

components, carrier + lower sideband - upper sideband.

( ) [ ( )]sin(2 )c cy t A m t f t

( ) [ sin(2 ]sin(2 )c m c cy t A A f t f t

1 1( ) sin(2 ) cos(2 2 ) cos(2 2 )

2 2c c m c m m c my t A f t A f f t A f f t

The modulation index is the depth of the variation around the original level of the carrier, Ac. When

multiplied by 100, it is the percent modulation.

mam

c c

AAm

A A

The power in an AM signal is the sum of the power in the carrier and the power in the signal.

22( )(( ( )) 1)

2

c m

cave

P P P

Akm t

AM signals can be demodulated with an envelope detector or a synchronous demodulator.

A double sideband (DSB) signal would contain the upper and lower sideband information but would

not have the carrier. DSB signals can be demodulated with a synchronous demodulator. A Costas loop is

a common technique.

Single sideband (SSB) can be either the lower or upper sideband information only without the

carrier or the other sideband. AM signals can be demodulated with a synchronous demodulator or by

carrier reinsertion and envelope detector.

Page 58: Dr. Robert A. Durham, PhD, PE Dr. Marcus O. Durham, PhD, PE … Nutshell Master.pdf · Dr. Robert A. Durham, PhD, PE . Dr. Marcus O. Durham, PhD, PE . A Quick Reference . The book

Page | 2-18

The bandwidth has a lower frequency of fc-fm, center frequency fc, and an upper frequency of

fc+fm.

h lBW f f

Page 59: Dr. Robert A. Durham, PhD, PE Dr. Marcus O. Durham, PhD, PE … Nutshell Master.pdf · Dr. Robert A. Durham, PhD, PE . Dr. Marcus O. Durham, PhD, PE . A Quick Reference . The book

Page | 2-19

2.7.3 Angle Modulation

Angle modulation mixes the signal as a component of the carrier sinusoid which includes the

frequency and phase terms. In essence the signal becomes the phase term.

( ) sin(2 )c c cx t A f t

( ) sin[2 ( )]c cy t A f t m t

( ) sin(2 )cos( ( )) cos(2 )sin( ( ))]c c c cy t A f t m t A f t m t

( ) sin(2 )cos( sin(2 )) cos(2 )sin( sin(2 ))]c c m m c c m my t A f t A f t A f t A f t

This is obviously a very complex function with numerous frequency components. There are infinite

sidebands to the signal. However, the amplitude of most deteriorates quickly. Frequency modulation

and phase modulation each use select components of this waveform.

The phase deviation or shift is a function of the message or information. As discussed earlier, it is

assumed that the message phase shift is zero. The function, kp, is the phase modulation index.

( ) ( )pt k m t

The instantaneous phase is the carrier angle added to the signal. This is the angle within the carrier

wave sin term.

( ) 2 ( )i ct f t t

The instantaneous frequency is the change of instantaneous phase with time. The instantaneous

frequency is the carrier frequency plus the frequency deviation.

2 ( )

i i

c

c

d

dt

df t

dt

( )

i i

c

c

df

dt

df t

dt

f f

The frequency deviation is the change in the phase, which is the change in the message with time.

( ) ( )d d

t km tdt dt

The message bandwidth is the frequency of modulation, fm.

m mBW f

The bandwidth of an FM & PM signal is approximated using Carson’s rule.

2( )

2( 1)

y m

fm m

BW f f

m f

Page 60: Dr. Robert A. Durham, PhD, PE Dr. Marcus O. Durham, PhD, PE … Nutshell Master.pdf · Dr. Robert A. Durham, PhD, PE . Dr. Marcus O. Durham, PhD, PE . A Quick Reference . The book

Page | 2-20

2.7.4 Frequency Modulation (FM)

Frequency modulation mixes the information or message with the carrier frequency. The amplitude

is constant. The result is the carrier varies above and below its idle or normal frequency, fc. As the

voltage amplitude of the modulating signal increases in the positive direction from A to B, the frequency

of the carrier is increased in proportion to the modulating voltage.

Frequency modulation is adding the carrier frequency and a function of the message.

( ) ( )i c ff t f k m t

The modulation index or factor is the maximum deviation in frequency, Δf, divided by the

modulation frequency. When multiplied by 100, it is the percent modulation.

fm

m

fm

f

The frequency modulator constant is the frequency deviation divided by the amplitude of the

modulating or message signal.

f

m

fk

A

Page 61: Dr. Robert A. Durham, PhD, PE Dr. Marcus O. Durham, PhD, PE … Nutshell Master.pdf · Dr. Robert A. Durham, PhD, PE . Dr. Marcus O. Durham, PhD, PE . A Quick Reference . The book

Page | 2-21

2.7.5 Phase Modulation

Phase modulation is another component of angular modulation that is sometimes referred to

indirect FM. Note that the phase is part of the sinusoid. Here, the amount of the carrier frequency shift

is proportional to both the amplitude and frequency of the modulating signal. The phase of the carrier is

changed by the change in amplitude of the modulating signal. The modulated carrier wave is lagging the

carrier wave when the modulating frequency is positive.

Phase modulation is manipulation of the angle of the carrier and a function of the signal.

( ) 2 ( )i c pt f t k m t

The modulation index is the peak phase variation.

( )pm

m

fm t

f

The phase modulation constant depends on both the frequency and amplitude. It is the ratio of the

phase deviation to the message.

( )

( )p

tk

m t

Page 62: Dr. Robert A. Durham, PhD, PE Dr. Marcus O. Durham, PhD, PE … Nutshell Master.pdf · Dr. Robert A. Durham, PhD, PE . Dr. Marcus O. Durham, PhD, PE . A Quick Reference . The book

Page | 2-22

2.7.6 Sampled Messages

A message, m(t) can be recreated from uniformly spaced samples. The sampling frequency, called

the Nyquist frequency fN, must be at least twice as fast as the highest frequency being recreated.

1

2N

s

f fT

2.7.7 Digital - Pulse Modulation

Pulse or digital modulation is frequently used to transmit sampled messages. Analog to digital

conversion is a two step process. First, sampling changes the analog source to a series of discrete values,

called sample. Second, quantization, converts each sample to a number. The number of quantization

levels, q, is the two power of the number of bits.

2nq

The bandwidth required is inversely proportional to the inverse of twice the pulse length or

duration, T. This is called the Shannon bandwidth when the Dimensionality, D is included. For minimum

bandwidth, D=1.

2S

DBW

T

The message bandwidth, W, and the number of bits determine the minimum modulated bandwidth,

BW.

22 logBW nW W q

Page 63: Dr. Robert A. Durham, PhD, PE Dr. Marcus O. Durham, PhD, PE … Nutshell Master.pdf · Dr. Robert A. Durham, PhD, PE . Dr. Marcus O. Durham, PhD, PE . A Quick Reference . The book

Page | 2-23

2.8 Signal transmission

2.8.1 dBm

Signal power can be measured in watts. However, comparison values and small signals are

measured in decibels.

1010log ( )signal

ref

Pdb

P

When the reference is on milliwatt, then the decibels are reference as dBm.

1010log ( )1

signalPdBm

mW

As a result a 1 milliwatt signal is 0dbM.

0 1 dBm mW

For amplitude measurements (Amps & Volts)

1020logsignal

noise

AdB

A

2.8.2 Noise

Noise is a random or background signal that may interfere with the message or information. Signal-

to-noise ratio is an indication of the power ratio between the desired information and the background

noise. The symbols are SNR or S/N.

/signal

noise

PS N

P

Often the expression is in terms of decibels (dB).

10 10/ ( ) 10log ( ) 20log ( )signal signal

noise noise

P AS N db

P A

In a digital signal, the number of bits in each value determines the SNR. Noise in a digital signal is

dependent on the conversion process. The dynamic range is an expression of the SNR.

10/ ( ) ( ) 20log (2 )nS N db DR db where n = # of bits

White noise creates a thermal noise power, P, in watts that is dependent on the bandwidth, Δf in

Hertz and temperature, T in degrees Kelvin. This is also the thermal noise that will be created by

electron activity in a resistor and is called Johnson noise.

Page 64: Dr. Robert A. Durham, PhD, PE Dr. Marcus O. Durham, PhD, PE … Nutshell Master.pdf · Dr. Robert A. Durham, PhD, PE . Dr. Marcus O. Durham, PhD, PE . A Quick Reference . The book

Page | 2-24

-23

K

1.3806503 × 10 Boltzmann's constant

273.15

T B K

B

OC

P K T f

JK

K

T T

For current or voltage across the resistor the power has the standard relationships.

22V

P I RR

Thermal noise at room temperature is dependent on the bandwidth. The units are decibels.

( ) 174 10log( )P dB f

The total noise figure for a series of transfer functions or amplifiers is based on the ratio of the noise

figure for each stage, F, to the gain ratio of each stage, G. The noise figure and gain must be converted

to the power ratio from dB.

31 2

1 1 2

11...

1T

FF FF

G G G

Page 65: Dr. Robert A. Durham, PhD, PE Dr. Marcus O. Durham, PhD, PE … Nutshell Master.pdf · Dr. Robert A. Durham, PhD, PE . Dr. Marcus O. Durham, PhD, PE . A Quick Reference . The book

Page | 2-25

2.8.3 Propagation - Transmit

The velocity of propagation of a wave is the distance the wave will travel in one time period. If the

distance is one wavelength, λ, then the velocity is the ratio of the wavelength to the frequency.

p

du

t f

In free space, the propagation velocity is the speed of light.

82.99 792 458 x 10c

The velocity of a wave on a transmission line is simply the ratio of the distance to the time it takes

for the wave to propagate. For a reflected wave, the distance is twice the length because of the trip

length and back.

p

du

t

Transmission of waves involves the power density in Watts per square meter. It is the ratio of the

power transmitted to the orthogonal area that the waveform strikes. A spherical shape is the normal

pattern of an omni-directional wave.

transmitteddensity 24

range from antenna, radius of sphere

xP PP

A R

R

Antennas can direct power in specific directions. The gain of the antenna is the radiation intensity in

a particular direction divided by the power that would be radiated from an omni-directional or isotropic

antenna.

Effective radiated power

Isotropic rated powerG

Power is dissipated as a waveform propagates. The attenuation or loss in free space depends on the

velocity of light. In other mediums, the velocity of propagation should be used. The loss is dB, distance is

m, and frequency is Hz.

420log

/fs

dP

c f

Characteristic impedance is the opposition in a circuit that connected to the output terminals of a

line will cause the line to appear infinitely long. It is the electric and magnetic property of the material

that impacts the velocity of propagation.

0

1p

p

Z uu

The electric property is permittivity in Farads per meter, Fd/m. It is a factor of the free air.

Page 66: Dr. Robert A. Durham, PhD, PE Dr. Marcus O. Durham, PhD, PE … Nutshell Master.pdf · Dr. Robert A. Durham, PhD, PE . Dr. Marcus O. Durham, PhD, PE . A Quick Reference . The book

Page | 2-26

0

-120 8.854x10 Fd/m

r

The magnetic property is permeability in Henries per meter, Hy/m.

0

70 4 x10 Hy/m

r

From these three preceding concepts, the impedance of free space air is calculated.

0 377Z

Because of the definitions of inductance and capacitance in relation to permeability and

permittivity, characteristic impedance can be found in terms of circuit elements.

0

LZ

C

2.8.4 Reflections

Maximum power transfer occurs when the load is equal to the source or characteristic impedance.

When a discontinuity occurs on a line or a load is connected that does not match the characteristic

impedance, the waveform will be reflected and oppose the message signal. The reflection coefficient

describes both the magnitude and phase shift of the reflection. The coefficient is the ratio of the

complex forward voltage to the complex reverse wave voltage.

0

0

fL

L r

VZ Z

Z Z V

Standing wave ratio is the maximum power over the minimum power transferred. SWR is

dependent on the reflection coefficient.

1

1SWR

Voltage SWR is the maximum voltage over the minimum voltage nodes. VSWR only contains the

magnitude of reflection coefficient.

max

min

1

1

VVSWR

V

The reflection coefficient has the following range of values.

Γ = − 1: maximum negative reflection, line is short-circuited,

Γ = 0: no reflection, when the line is perfectly matched,

Γ = + 1: maximum positive reflection, line is open-circuited.

At the maximum nodes the waves interfere positively and add. At the minimum nodes, the waves

are colliding and subtract.

Page 67: Dr. Robert A. Durham, PhD, PE Dr. Marcus O. Durham, PhD, PE … Nutshell Master.pdf · Dr. Robert A. Durham, PhD, PE . Dr. Marcus O. Durham, PhD, PE . A Quick Reference . The book

Page | 2-27

max

min

(1 )

(1 )

f r f

f r f

V V V V

V V V V

Transmission line properties are defined in terms of propagation constant. Propagation constant is

inversely proportional to the wavelength. The distance is measured from the load.

2

00

0

( )

( )

tan( )

tan( )

j d j d

j d j d

Lin

L

V d V e V e

I d I e I e

Z jZ dZ Z

Z jZ d

Page 68: Dr. Robert A. Durham, PhD, PE Dr. Marcus O. Durham, PhD, PE … Nutshell Master.pdf · Dr. Robert A. Durham, PhD, PE . Dr. Marcus O. Durham, PhD, PE . A Quick Reference . The book

Page | 2-28

R L C

2.9 RLC System Response

2.9.1 RLC Equations

The three elements, RLC can be arranged in series or its dual parallel. This is a second order system.

The analysis of the circuit can be made in many domains. Typically the time domain is the starting point.

However, the Calculus required makes the mathematic interpretation difficult. For that reason

numerous transforms are use d. The math of the transforms will not be developed, but the

correspondence is apparent from the table. The duality of the circuits is intriguing.

Function Series Parallel

Reference Same current through all elements Same voltage across all elements

Diagram

Fundamental 2

2

1( )

d q dqv t L R q

dt Cdt

2

2

1 1( )

d di t C

R dt Ldt

Time 1( )

div t L Ri idt

dt C

1( )

dvi t C Rv vdt

dt C

LaPlace 1( ) ( ) ( )V s Ls R I s

Cs

1 1( ) ( ) ( )I s Cs V s

R Ls

Sinusoidal Steady State

1( ) ( ) ( )V j j L R I j

j C

1 1( ) ( ) ( )I j j C V j

R j L

Several observations can be made about the relationships.

ds j

dt

1 1dt

j s

'dq

q idt

'd

vdt

R L C

Page 69: Dr. Robert A. Durham, PhD, PE Dr. Marcus O. Durham, PhD, PE … Nutshell Master.pdf · Dr. Robert A. Durham, PhD, PE . Dr. Marcus O. Durham, PhD, PE . A Quick Reference . The book

Page | 2-29

2.9.2 System Response

The system response is the solution to the second order equation.

Time constant is the time it takes for a signal to settle so that the exponential decay.

time constant L

RCR

2.9.3 Characteristic Transfer

Transfer functions are often used as a model for a system.

Function Series Parallel

Transfer function

( )( )

( )

I sY s

V s

( )( )

( )

V sX s

I s

Characteristic 1( )

1Y s

Ls RCs

1

( )1 1

Z s

CsR Ls

Standard form

2

/( )

1

s LY s

Rs s

L LC

2

/( )

1

s CZ s

ss

RC LC

Resonance 2 2

0

/( )

s LY s

s s

2 20

/( )

s CZ s

s s

2.9.4 Resonance

Frequency is inversely related to time. Angular frequency is one complete revolution of cycle of the

frequency.

2 f

Resonance is a very significant concept that may be a boon or ban to electrical systems. Resonance

is the frequency where the magnetic (or inductor) energy equals the electric (or capacitor) energy.

0

1

LC

Since the energies are balanced, it flows from one to the other resulting in a sinusoidal frequency.

The natural frequency is the oscillation determined by the physical properties. Resonant frequency is a

created oscillation that matches the natural frequency. Resonance is the frequency at which the input

impedance is purely real or resistive.

The frequency response has a roll-off on either side. The transition is called the cut-off frequency.

20 cL cH

Page 70: Dr. Robert A. Durham, PhD, PE Dr. Marcus O. Durham, PhD, PE … Nutshell Master.pdf · Dr. Robert A. Durham, PhD, PE . Dr. Marcus O. Durham, PhD, PE . A Quick Reference . The book

Page | 2-30

Bandwidth, Δω, is the range between the upper and lower cut-off

frequencies. The bandwidth is also called the pass band or bandpass.

cH cL

02

cL

02

cH

Quality factor or selectivity is the sharpness of the peak at

resonance.

0Q

oLQ

R

Damping is the effect of resistance on the rate that a signal is stabilized to steady state. Undamped

implies that there is no resistance, R=0. The damping coefficient is dependent on the natural frequency

and is inversely proportional to twice the quality factor. Some authors use the symbol alpha, α, rather

than zeta, ς. Note this is also the real term of the LaPlace, σ.

0 actual damping

2 critical damping2 /

R

Q L C

The range of values for the camping coefficient reflects how quickly the waveform will settle and

whether it will overshoot. Under-damping results in oscillations or ringing, over-damping results in a

slow exponential approach to stability, critical-damping is the transition between oscillations and

exponential.

1 under-damped = oscillation

1 critical-damped = transition

1 overdamped = exponential

The relationship between the various factors can be described in terms of the quality factor.

0 0

2Q

Damped resonance, ωd, is a shift from the resonant frequency caused by the damping.

2 2 20d

The root of the characteristic equation has the real part as damping coefficient and the imaginary

part as the damped resonance. For the second order, there are two roots.

Page 71: Dr. Robert A. Durham, PhD, PE Dr. Marcus O. Durham, PhD, PE … Nutshell Master.pdf · Dr. Robert A. Durham, PhD, PE . Dr. Marcus O. Durham, PhD, PE . A Quick Reference . The book

Page | 2-31

1,2 ds j

Page 72: Dr. Robert A. Durham, PhD, PE Dr. Marcus O. Durham, PhD, PE … Nutshell Master.pdf · Dr. Robert A. Durham, PhD, PE . Dr. Marcus O. Durham, PhD, PE . A Quick Reference . The book

Page | 2-32

2.9.5 Series Parallel Duality

Comparison of the standard form and the resonance equation reveal the duality of impedance and

admittance. The symmetry of the duality resolves to a reciprocal form at resonance.

Function Series Parallel

Quality factor XQ

R

RQ

X

Quality factor 1 LQ

R C

CQ R

L

2.9.6 First Order

A first order system has a resistor and either a capacitor or inductor. Therefore, there is no

oscillation. However, there is still a cut-off frequency that is the inverse of the time constant.

1 1( ) 0 cR j

C RC Time Constant = RC

2( ) ( ) 0 c

RL j R j

L Time Constant =

L

R

End of chapter

Page 73: Dr. Robert A. Durham, PhD, PE Dr. Marcus O. Durham, PhD, PE … Nutshell Master.pdf · Dr. Robert A. Durham, PhD, PE . Dr. Marcus O. Durham, PhD, PE . A Quick Reference . The book

Page | 2-33

Chapter 2 Problems Problem 2-1

Consider the circuit shown below. R1 and R2 are 5Ω

resistors. R3 is a 10Ω resistor and R4 is a 15Ω resistor.

Z1 is a 20μF capacitor, and V1 is a 120V source. The

time constant of the circuit is most nearly

(A) 85 μS

(B) 138 μS

(C) 550 μS

(D) 400 μS

SOLUTION:

Redraw the circuit to make it easier to see

The resistances can be combined to

determine the equivalent resistance of the

circuit.

4 3 1 2( / / )eqR R R R R

15 10 (5 / /5 )eqR

The time constant of a RC circuit is

27.5 20 550eq eqR C F S

The answer is (C)

25 2.5 27.5eqR

Z1R1

R3

R2

R4

Z1

R1 R3

R2

R4

AC

Page 74: Dr. Robert A. Durham, PhD, PE Dr. Marcus O. Durham, PhD, PE … Nutshell Master.pdf · Dr. Robert A. Durham, PhD, PE . Dr. Marcus O. Durham, PhD, PE . A Quick Reference . The book

Page | 2-34

Problem 2-2

Consider the circuit shown in the problem above, and recreated below. R1 and R2 are 15Ω resistors.

R3 is a 20Ω resistor and R4 is a 15Ω resistor. Z1 is a 20mH capacitor, and V1 is a 120V, 60Hz source. The

switch has been closed for a significant period of time. The voltage across the inductor is most nearly.

R1=15Ω

R2=15Ω

R3+R4

20Ω + 15Ω=35Ω120V

60Hz

Z1=j7.54Ω

I

1

A

(A) 25.455°

(B) 10.580°

(C) 50.7-60°

(D) 61.890°

SOLUTION

Impedance of the Inductor Z1

1 2 60(20 ) 7.54Z j mH j

Redraw with all impedances

The answer is (B)

R1=15Ω

R2=15Ω

R3+R4

20Ω + 15Ω=35Ω120V

60Hz

Z1=j7.54Ω

I

1

A

3 4 1 35 7.54AR R R Z j

15 // 10.6 0.664B AR R j

1

120( ) 120 (10.6 0.664 )49.73 1.822

( ) (25.6 0.664 )

BA

B

R V jV j

R R j

1

1

1

( ) (49.73 1.822 )( 7.54 )1.83 10.31 10.5 79.94

( ) (35 7.54 )

AZ

A

V Z j V jV j V V

R Z j

Page 75: Dr. Robert A. Durham, PhD, PE Dr. Marcus O. Durham, PhD, PE … Nutshell Master.pdf · Dr. Robert A. Durham, PhD, PE . Dr. Marcus O. Durham, PhD, PE . A Quick Reference . The book

Page | 2-35

Problem 2-3

What is the time constant of the figure shown?

0.2μF

3MΩ

12V

SOLUTION:

The time constant of an RC circuit is

6 63 10 0.2 10

0.6 seconds

RC

Page 76: Dr. Robert A. Durham, PhD, PE Dr. Marcus O. Durham, PhD, PE … Nutshell Master.pdf · Dr. Robert A. Durham, PhD, PE . Dr. Marcus O. Durham, PhD, PE . A Quick Reference . The book

Page | 2-36

Problem 2-4

In the figure below, the switch has been open for a significant period of time and is closed at t=0.

What is the current in the capacitor at t=0+?

0.3μF

6MΩ

12V

SOLUTION:

The capacitor, at t=0+, acts as a short circuit. The current through the capacitor then is determined

by the voltage and the resistance

6

60

122 10

6 10c t

V Vi A

Z

Page 77: Dr. Robert A. Durham, PhD, PE Dr. Marcus O. Durham, PhD, PE … Nutshell Master.pdf · Dr. Robert A. Durham, PhD, PE . Dr. Marcus O. Durham, PhD, PE . A Quick Reference . The book

Page | 2-37

Problem 2-5

In the figure below, the switch has been open for a significant period of time, and is then closed at

t=0. What is the current through the two capacitors at t=0+?

10V

75Ω

0.001F

0.001F 500Ω 200Ω

SOLUTION:

If the switch is opened for a significant period of time the capacitor on top of the circuit is charged

to 10V, and the capacitor in the middle of the circuit is discharged to 0V. At t=0+, the capacitors are

modeled as voltage sources with the charged voltages. The equivalent circuit is shown below

10V

75Ω

500Ω 200Ω0V

10V

is

iA iB

i2

The voltage across the 500Ω resistor is 0V, so iB=0A.

KVL on the left loop is

10 (75 ) 0 0

100.133

75

s

s

V i V

Vi A

KVL on the right loop is

2

2

10 0 (200 ) 0

100.05

200

V V i

Vi A

KCL

Page 78: Dr. Robert A. Durham, PhD, PE Dr. Marcus O. Durham, PhD, PE … Nutshell Master.pdf · Dr. Robert A. Durham, PhD, PE . Dr. Marcus O. Durham, PhD, PE . A Quick Reference . The book

Page | 2-38

2 0

0.133 0 0.05 0

0.133 0.05 0.083

s A B

A

A

i i i i

A i A

i A

The current through the top capacitor is i2=0.05A

The current through the middle capacitor is iA = 0.083A

Page 79: Dr. Robert A. Durham, PhD, PE Dr. Marcus O. Durham, PhD, PE … Nutshell Master.pdf · Dr. Robert A. Durham, PhD, PE . Dr. Marcus O. Durham, PhD, PE . A Quick Reference . The book

Page | 2-39

Problem 2-6

In the figure below, the switch has been open for a significant period of time. The switch is closed at

t=0. Find the current through the resistor at t=0+, and at t=1.25s. Find the energy in the inductor at

t=1.25s.

50V

20Ω

8H

SOLUTION:

The current in an inductor cannot change instantaneously, so

(0 ) 0Li A

The general solution for a first order RL circuit is

( ) 1

Rt

LV

i t eR

20 1.25

850

(2) 120

2.39

s

HV

i e

A

The energy in the inductor is found using

212LW Li

212(8 )(2.39) 22.85LW H J

Page 80: Dr. Robert A. Durham, PhD, PE Dr. Marcus O. Durham, PhD, PE … Nutshell Master.pdf · Dr. Robert A. Durham, PhD, PE . Dr. Marcus O. Durham, PhD, PE . A Quick Reference . The book

Page | 2-40

Problem 2-7

A carrier wave of 12 MHz is amplitude modulated by an audio signal of 1.5 kHz. What are the upper

and lower limits of the resulting modulated signals bandwidth?

SOLUTION:

612 10cf Hz

31.5 10Mf Hz

Lower sideband frequency - 6 312 10 1.5 10 11,998,500C Mf f Hz Hz

Upper sideband frequency - 6 312 10 1.5 10 12,001,500C Mf f Hz Hz

Page 81: Dr. Robert A. Durham, PhD, PE Dr. Marcus O. Durham, PhD, PE … Nutshell Master.pdf · Dr. Robert A. Durham, PhD, PE . Dr. Marcus O. Durham, PhD, PE . A Quick Reference . The book

Page | 2-41

Problem 2-8

A 110 MHz carrier is frequency modulated by a 65kHz information signal. The information signal has

1V amplitude, and a frequency modulator constant of 100Hz/V. What is the bandwidth?

SOLUTION:

Carson’s Rule

2( )f mBW ak f

32 1 100 65 10

130,200 130

HzBW V Hz

V

Hz kHz

Page 82: Dr. Robert A. Durham, PhD, PE Dr. Marcus O. Durham, PhD, PE … Nutshell Master.pdf · Dr. Robert A. Durham, PhD, PE . Dr. Marcus O. Durham, PhD, PE . A Quick Reference . The book

Page | 3-1

Chapter 3 - Power Analysis

3.1 Introduction Power analysis looks at the energy conversion segment of electrical systems. Machines can be

modeled as a Thevenin equivalent voltage and impedance with a magnetizing circuit consisting of an

inductor with its resistance. Three types of problems are encountered.

Model parameters and losses require the complete model using circuit theory.

Transients and load flow use the Thevenin equivalent.

Steady state uses the terminal conditions with complex power.

The time domain signal representation contains all the components of the responses including DC,

transient, and sinusoidal.

( ) cost

y t F I F e t

Page 83: Dr. Robert A. Durham, PhD, PE Dr. Marcus O. Durham, PhD, PE … Nutshell Master.pdf · Dr. Robert A. Durham, PhD, PE . Dr. Marcus O. Durham, PhD, PE . A Quick Reference . The book

Page | 3-2

3.2 Power Definitions Use per unit calculations if multiple voltages are used in the problem (i.e. transformers).

Use the following equations to obtain the desired quantity.

Complex apparent power – volt-amps, VA

* (cos sin )S VI P jQ S j

Real Power – watts, W

P VI pf S pf vi

resistance only, Q=0

Reactive Power – volt-amp reactive, VAR

1sin(cos ) sinQ VI pf VI

Q > 0 for inductive load, lagging pf

Q < 0 for capacitive load, leading pf

Power Factor

*

cosP vi

pfS VI

pf=1 when or L CZ R jX jX

X/R

tan LX

R

Convert Hp to real power (KW).

? 0.746 1

* *Hp kW

PHp eff

Alternately, convert Hp to apparent power.

? 0.746 1 1

* * *Hp kW

SHp eff pf

The angle has several relationships. It is also the time delay between the voltage and current

crossing the axis (going through zero value). The conversion is

2π radians = 360 degrees = 1 cycle

1cos 2V I Z S pf ft

Page 84: Dr. Robert A. Durham, PhD, PE Dr. Marcus O. Durham, PhD, PE … Nutshell Master.pdf · Dr. Robert A. Durham, PhD, PE . Dr. Marcus O. Durham, PhD, PE . A Quick Reference . The book

Page | 3-3

2

tf

The relationship between the power terms is shown graphically below.

The relationship of the power terms is often illustrated with complex triangles.

Impedance is the ratio of voltage and current (Ohm’s Law)

2 2( )V

Z R jX R XI

Complex Numbers are easiest to manipulate in the following manner:

Add & Subtract P’s and Q’s

Multiply & Divide: Convert P & Q to S &, multiply (divide) magnitudes, add (subtract) angles

Phasor rotation is used to explain the relationship between the lines of a three phase power system

kW

kVAR kVA

θ

I

Vref θ

X

R

Z

Impedance Triangle

Q

P

S

Power Triangle

Page 85: Dr. Robert A. Durham, PhD, PE Dr. Marcus O. Durham, PhD, PE … Nutshell Master.pdf · Dr. Robert A. Durham, PhD, PE . Dr. Marcus O. Durham, PhD, PE . A Quick Reference . The book

Page | 3-4

3.2.1 – Z,R,X,θ; S,P,Qpf Conversions

Often, when performing power calculations, all of the components of a problem are not known. Out

of the four components to any problem, only two must be known to calculate all four. The base

presumptions are the equations discussed above.

Before conversion, move power factor to angle.

1cos ; cos ( )pf pf

For Impedances

If know Z R X θ

R, X 2 2Z R X R R X X 1tanX

R

R, θ cosZ R R R tanX R

R,Z Z Z R R 2 2X Z R 1cosR

Z

Z, θ Z Z cosR Z sinX Z

Z, X Z Z 2 2R Z X X X 1sinX

Z

X, θ

sin

XZ

tan

XR

X X

For Power

If know S P Q θ

P, Q 2 2S P Q P P Q Q 1tan

Q

P

P, θ cosS P P P tanQ P

P, S S S P P 2 2Q S P 1cosP

S

S, θ S S cosP S sinQ S

S, Q S S 2 2P S Q Q Q 1sinQ

S

Q, θ

sin

QS

tan

QP

Q Q

Page 86: Dr. Robert A. Durham, PhD, PE Dr. Marcus O. Durham, PhD, PE … Nutshell Master.pdf · Dr. Robert A. Durham, PhD, PE . Dr. Marcus O. Durham, PhD, PE . A Quick Reference . The book

Page | 3-5

3.3 Three-Phase AC Three-phase indicates 3 lines, 3 connections, 3 sources and 3 loads

When balanced, a single relationship for the phase values can be

related. Phase voltages are the voltage difference between each phase

and neutral.

The relationship between the lines and phases are simple vector

calculations. The reference voltage starts with an angle of zero. The line

voltage is the difference in the corresponding phase voltages.

3 32 2

0 120

(1 0) ( 0.5 ) (1.5 )

3 30

ab an bn p p

P p P

p

V V V V V

V j V j V j

V

The other two line voltages are simply shifted by 120°.

3 30 120

3 30 240

bc p

ca p

V V

V V

The relationships are visually shown in a phasor diagram.

The delta connection is shifted by 120° between phases.

From these considerations, the following relationships hold,

whether delta or wye.

303

303

N A B C

LLLN

LP

I I I I

VV

VV

Specific conditions impact the significance of the

equations.

ab bc

for , 0

Phase lags 30 behind line

is 30 behind V , and 90 ahead of V

N

an

I

V

A

C B

Line, IA

Neutral

0P anV V

120P bnV V 240P cnV V

Line, IB

Line, IC

Van

Vab

Vbn

Vcn

Vbc

Vca

30o

30o

30o

Line, IA

0L abV V

120L bc

V V

240L caV V

Line, IB

Line, IC

A

BC

Page 87: Dr. Robert A. Durham, PhD, PE Dr. Marcus O. Durham, PhD, PE … Nutshell Master.pdf · Dr. Robert A. Durham, PhD, PE . Dr. Marcus O. Durham, PhD, PE . A Quick Reference . The book

Page | 3-6

3.3.1 Y-∆ Relationships

For a three-phase system, there are only two possible connections, wye and delta. The magnitude of

the voltage and current changes by a factor of 3 .

Wye Delta

Power can be determined using phase or line values.

1 Power cosP PV I

3 Power 3 cos easier for wyeP PV I

3 Power 3 cos delta or wyeL LV I

Apparent power contains both real and reactive components. Real (Watts) is dependent on

resistance and reactive (VARS) is dependent on inductors and capacitors.

Power factor = cos

Complex power (S) = VA = *3 L LV I

The complex value can be expanded to magnitude and angle.

cosP S cosQ S

2 2( )S P Q

Impedance is only a single phase relationship. There is no relationship to line values.

P

P

VZ

I

Neutral current is the sum of the current in the phases.

for wye w/ neutralN A B CI I I I

NI 0 for delta or wye w/o neutral

A key principle of three phase power affirms that regardless of whether the current is Y or ∆, the line

currents lag the line-to-neutral voltages by the phase (impedance) angle.

Z R jX 2 2( )Z R X 1tanX

R

3 P L

P L

V V

I I

3

P L

P L

V V

I I

Van

Vcn Vbn

a

bc

Ia

Ib

Ic

Page 88: Dr. Robert A. Durham, PhD, PE Dr. Marcus O. Durham, PhD, PE … Nutshell Master.pdf · Dr. Robert A. Durham, PhD, PE . Dr. Marcus O. Durham, PhD, PE . A Quick Reference . The book

Page | 3-7

3.3.2 Phase Sequence

Phase sequence is used to determine the direction of rotation of machines and compatibility for

connection to other circuits.

“Look” in the zero axis

Rotate phase in counter-clockwise direction

Write down phasors as they come by

Sequence of voltage phases is every other letter

ab,bc,ca → abc

an,bn,cn → abc

Vab

Van

VcaV

cn

Vbc

Vbn

acb sequence

ab,ca,bc → acb

an,cn,bn → acb

Vab

Van

VbcV

bn

Vca

Vcn

abc sequence

Page 89: Dr. Robert A. Durham, PhD, PE Dr. Marcus O. Durham, PhD, PE … Nutshell Master.pdf · Dr. Robert A. Durham, PhD, PE . Dr. Marcus O. Durham, PhD, PE . A Quick Reference . The book

Page | 3-8

3.4 Power Transfer across a Reactance Power is converted to heat in a resistor; however, power is transferred across a reactance and

causes a phase shift in the voltage.

EA

V2V1

The standard power computation is the starting relationships.

*S VI

P S pf

The real and reactive power transfer contains the phase shift.

1 2 1 21 2

sin( )VVP

X

2

1 2 1 1 2 1 2

1cos( )Q V V V

X

To create a consistent analysis, assume the angle relationships for voltage and apparent power.

1 0V V

S Z

From this, the remaining angles and the current are found.

I S

cospf

(cos sin )I I j

Page 90: Dr. Robert A. Durham, PhD, PE Dr. Marcus O. Durham, PhD, PE … Nutshell Master.pdf · Dr. Robert A. Durham, PhD, PE . Dr. Marcus O. Durham, PhD, PE . A Quick Reference . The book

Page | 3-9

3.5 Power One-Line Most power analysis uses a one-line diagram. The sources or generators (G), loads or motors (M),

and transformers (T) are identified with a note about wye or delta connection. Transmission lines (Z) are

described by impedance. A connection is called a bus (B). Voltage, current, and power transfer is

calculated at each bus.

G1

T1Z1

Z2

B1 M2

M1

The description and specification of each component is displayed on the one-line. If there is

inadequate room, a table may be used. Impedance is often in terms of Ohms per distance.

Location/ condition

kVA Pf or /Z P Q VLL /V IL /I Z distance R X

G1

T1

B1

Z1

M1

Z2

M2

The known values are placed in the table. Other values in the table are calculated as required based

on the known parameters. The voltage, current, and power transfer is simply calculated at each node.

Page 91: Dr. Robert A. Durham, PhD, PE Dr. Marcus O. Durham, PhD, PE … Nutshell Master.pdf · Dr. Robert A. Durham, PhD, PE . Dr. Marcus O. Durham, PhD, PE . A Quick Reference . The book

Page | 3-10

3.6 Power Problem Plan There is a sequence that assists in solving complex electrical problems. Although all these may not

be required, they provide a process to assure all conditions are considered. Simpler problems may

require only one of these steps.

Reduce to one port for simple circuits.

Thevenin or Norton equivalents are a boundary condition.

Alternatively use a two port network.

For the network, get a model: π(∆), T(Y), or transfer function.

It has input and output of voltage or current.

Solve input & output independently.

Use conservation (V, I)

The sum of voltage or currents is 0 (KVL, KCL).

Use Ohm’s law

Ratio V

ZI

Use fields.

This includes, electric, magnetic, and inverse-square.

Use power – thermal – conversion

Product *S VI

Power transfer across reactance gives phase shift.

Make a table of values known & missing at each point of interest in circuit.

Frequency response always depends on reactance X.

Remember the goal!

Page 92: Dr. Robert A. Durham, PhD, PE Dr. Marcus O. Durham, PhD, PE … Nutshell Master.pdf · Dr. Robert A. Durham, PhD, PE . Dr. Marcus O. Durham, PhD, PE . A Quick Reference . The book

Page | 3-11

3.7 Mechanical Power Power is the product of potential across and flow rate (number/time) through the machine

*S P Q T FV vi

S = pressure F = Force T = torque v = volts

Q = vol/time V = distance/time ω = revolutions/time i = amps (coul/sec)

A conversion factor (K) may arise because of different units in the measured values. A performance

factor (PF) may be necessary to scale the machine rating.

*convert

PP

K PF

The conversion factor depends on the units

Pconvert = Hp S=psi Q=gpm k=1714 1 gpm=5.45 m3/day

Pconvert = Hp S=feet Q=bpd k=135663 pu k=56,000

Pconvert = Hp T=ft-lb w=rpm k=5250

Pconvert = Hp v=volt i=amp k=746

Pconvert = watt v=volt i=amp k=1

Efficiency is the ratio of output to input power terms

( )

out out

in out loss

P Peff

P P P

The efficiency (eff) depends on the pump (machine) design

triplex=0.9 duplex=0.85 centrifugal=0.7

rod=0.9 beam system=0.4 motor=0.98-0.94

The brake horsepower (BHP) is the required power into the mechanical machine

*

*

S QBHP

K eff

The speed ratio (SR) determines comparative speeds

motor speed pump sheavediameterSR

pump speed motor sheavediameter

Page 93: Dr. Robert A. Durham, PhD, PE Dr. Marcus O. Durham, PhD, PE … Nutshell Master.pdf · Dr. Robert A. Durham, PhD, PE . Dr. Marcus O. Durham, PhD, PE . A Quick Reference . The book

Page | 3-12

Load Torque (TL) is the shaft effort required to be input to the load by the motor

* *L

BHP PF KT

The pump performance factor (PF) depends on the pump design

Duplex – 1.5 Triplex – 1.31 Quintuplex – 1.27

Starting Torque (TS) is the effort required by the motor during starting

* * *rating

S

HP PF SR KT

The motor performance factor depends on the motor torque design characteristic.

NEMA B–1.5 NEMA C–2.25 NEMA D–2.75 NEMA E–1.5

Equipment is sized by using the following sequence:

Determine the horsepower of the fluid,

Convert to brake horsepower of the pump

Determine the pump torque

Select a motor with horsepower rating greater than BHP and a NEMA torque design rating greater

than the load torque

Size motor and pump sheaves to maintain appropriate pump speed

Page 94: Dr. Robert A. Durham, PhD, PE Dr. Marcus O. Durham, PhD, PE … Nutshell Master.pdf · Dr. Robert A. Durham, PhD, PE . Dr. Marcus O. Durham, PhD, PE . A Quick Reference . The book

Page | 3-13

3.8 Electric Machinery

3.8.1 Basics

Alternating current is created in a coil of wire by a magnet rotating very close to the wire. As the

magnetic pole distance varies, the magnitude of voltage induced on the coil changes.

The chart illustrates the magnet at four positions with the fifth position the same as the starting

point.

V

N

S

V

N S

V

S

N

V

S N

V

N S

+V

-V

Page 95: Dr. Robert A. Durham, PhD, PE Dr. Marcus O. Durham, PhD, PE … Nutshell Master.pdf · Dr. Robert A. Durham, PhD, PE . Dr. Marcus O. Durham, PhD, PE . A Quick Reference . The book

Page | 3-14

3.8.2 Machine Models

Machines can be modeled as a Thevenin equivalent voltage and impedance with a magnetizing

circuit consisting of an inductor with its resistance. The model is a two-port network. Three types of

problems are encountered.

Model parameters and losses require the complete model using circuit theory.

Transients and load flow use the Thevenin equivalent without the magnetizing.

Steady state uses the terminal conditions with complex power.

In determining the model parameters, place the model of machine into a circuit then perform circuit

analysis.

What is the difference between the machines? There are four fundamental classes – DC,

synchronous, induction, and transformer.

The input energy and output energy determine the use. A motor has electrical in and mechanical

out. A generator has mechanical in and electric out. The same machine can be used in either form. It

simply depends on the driver input and the driven output.

Generator: Mechanical In – Electrical Out

Motor: Electrical In – Mechanical Out

Transformer: Electrical In – Electrical Out

Page 96: Dr. Robert A. Durham, PhD, PE Dr. Marcus O. Durham, PhD, PE … Nutshell Master.pdf · Dr. Robert A. Durham, PhD, PE . Dr. Marcus O. Durham, PhD, PE . A Quick Reference . The book

Page | 3-15

Machine Two-Port Models

RfAdj

Rf

-

+A

Ra

Lf

mag flux

Ea

Internal

Generated

+

-

Vt (DC)

DC

Machine

+

-

Vf (DC)

If Ia

Rf

Adj

Rf

-

+A

Ra

Lf

mag flux

Ea

Internal

Generated

+

-

Vp (AC)

Synchronous

Machine

+

-

Vf (DC)

If Ia

jXs

Li R Uses curve of

f AI vs E

1A fE K K i

fK I

A actual actual

A reference reference

E n

E n

( ) DCf f f fadjV I R R

t A A AV E I R

( )p A A A sV E I R jX

Page 97: Dr. Robert A. Durham, PhD, PE Dr. Marcus O. Durham, PhD, PE … Nutshell Master.pdf · Dr. Robert A. Durham, PhD, PE . Dr. Marcus O. Durham, PhD, PE . A Quick Reference . The book

Page | 3-16

R1 R2

R2 (1-s)/s

Mechanical

Rc

Core

jX1 jX2

TωjXm

magnetizing

Air

Gap

Vp

(AC)

+

-

I2I1

E1

Induction

Machine

R1 R2

Rc

Core

jX1 jX2

jXm

magnetizing

Air

Gap

Vp1

(AC)

+

-

I2I1

E1

Vp2

(AC)

+

-

Transformer

Page 98: Dr. Robert A. Durham, PhD, PE Dr. Marcus O. Durham, PhD, PE … Nutshell Master.pdf · Dr. Robert A. Durham, PhD, PE . Dr. Marcus O. Durham, PhD, PE . A Quick Reference . The book

Page | 3-17

Machine Loss Diagrams

I2R

Losses

Core

Loss

Mechanical

Loss

Stray

Loss

in t LP V I

convP

A A ind indE I

out app mP

DC Motor

I2R

Losses

Core

Loss

Mechanical

Loss

Stray

Loss

3 cosin P AP V I

convP

ind ind out load mP

Synchronous Motor

I2R

LossesCore

LossMechanical

LossStray

Loss

in app mP

convP

ind ind

3 cosout p AP V I

Synchronous Generator

I2R

Losses

Stator

CU

Core

Loss

Mechanical

Friction &

Windage

Stray

Loss

3 cosin P PP V I

AGP

ind ind out load mP

Induction Motor convP

I2R

Losses

Rotor

CU

Page 99: Dr. Robert A. Durham, PhD, PE Dr. Marcus O. Durham, PhD, PE … Nutshell Master.pdf · Dr. Robert A. Durham, PhD, PE . Dr. Marcus O. Durham, PhD, PE . A Quick Reference . The book

Page | 3-18

3.8.3 Machine Tests

Tests on machines are conducted with the following instrumentation connections. The source is a

variable voltage such as dc power supply or ac variac that can adjust the voltage into the machine. The

ammeter and the wattmeter may have a shunt that is used to bypass excessive current.

Read power, voltage, and current at each test.

For open circuit tests, connect as shown with no load. Run at rated voltage.

For short circuit tests, short the terminals of the transformer or block the rotor of the machine. Start

at low voltage and increase until near rated current.

PS

Ammeter

Voltmeter

WattMeter

Load

No load / open circuit test

Set rated voltage, frequency

Reduced I

Read I through core

100% Reactiveno loadI

Blocked Rotor / short circuit test

Set rated current, reduced V

Read reduced V

I through rotor

2 2full loadI R X

Page 100: Dr. Robert A. Durham, PhD, PE Dr. Marcus O. Durham, PhD, PE … Nutshell Master.pdf · Dr. Robert A. Durham, PhD, PE . Dr. Marcus O. Durham, PhD, PE . A Quick Reference . The book

Page | 3-19

3.8.4 Rotating Machines

Rotating machines can involve numerous different analyses. These include the torque and voltage

generation, power output, power losses, and performance.

Torque & Voltage Generation

Rotate a coil inside a magnetic field to develop a voltage and torque. The flux = φ and K = a constant

of machine design.

induced K I

inducedarmature

E K

ind

de N

dt

2

ZpK

a (a=turns ratio)

Rather than angular speed, the revolutions are often preferred where n = speed in rpm.

2 n

AE K n

'

60

ZpK

a

AC machines cause flux to change in a sinusoidal fashion.

sine N t

Sinusoids are analyzed using effective or RMS values. Although the machines may be three phase,

the analysis operates on single-phase at a time.

max 2c cE N N f

max

2A

EE

AE E

3y AE E

The number of magnetic poles and the frequency determine the speed.

# poles – p

/ 2p repetitions in one rotation

Frequency, f, is measured in Hertz.

Page 101: Dr. Robert A. Durham, PhD, PE Dr. Marcus O. Durham, PhD, PE … Nutshell Master.pdf · Dr. Robert A. Durham, PhD, PE . Dr. Marcus O. Durham, PhD, PE . A Quick Reference . The book

Page | 3-20

2elec mech

pf f

60

mm

nf

60 2 120

m me

n n ppf

Page 102: Dr. Robert A. Durham, PhD, PE Dr. Marcus O. Durham, PhD, PE … Nutshell Master.pdf · Dr. Robert A. Durham, PhD, PE . Dr. Marcus O. Durham, PhD, PE . A Quick Reference . The book

Page | 3-21

3.8.5 Power Conversion

Machines are about energy conversion from one form to another. The conversion is generally

investigated using power which is energy in some time.

Mechanical Power Converted

conv shaft mP

Electrical Power Converted

conv in stray mechanical coreP P loss loss loss

conv A AP E I

Output Power

2out conv I RP P loss

AC Machine

3 cos 3 cosout p p L LP V I V I

DC Machine

out L LP V I

Page 103: Dr. Robert A. Durham, PhD, PE Dr. Marcus O. Durham, PhD, PE … Nutshell Master.pdf · Dr. Robert A. Durham, PhD, PE . Dr. Marcus O. Durham, PhD, PE . A Quick Reference . The book

Page | 3-22

3.8.6 Losses

Losses are encountered anytime there is a resistance or opposition. These are electric or copper,

magnetic or ferrous core, and mechanical or friction. The inaccuracies of measurement are

compensated as stray losses.

Electric - Copper Losses (I2R)

DC Machine - (armature) (field)

AC Machine - (stator) (field)

Magnetic - Core Losses – Hysteresis & Eddy

m

c

E

R

Mechanical losses – friction & windage

No load rotational = (mechanical losses + core losses)

~Proportional to n3

Stray losses

miscellaneous - ~1% of output power

Brush losses (DC Only)

BD BD AP V I ( 2 )BDV V

2

A A AP I R2

f f fP I R

23s A AP I R2Pf f fI R

Page 104: Dr. Robert A. Durham, PhD, PE Dr. Marcus O. Durham, PhD, PE … Nutshell Master.pdf · Dr. Robert A. Durham, PhD, PE . Dr. Marcus O. Durham, PhD, PE . A Quick Reference . The book

Page | 3-23

3.8.7 Performance

Performance is a comparison of maximums and minimums. They are normalized, so they are

dimensionless.

Efficiency

N = efficiency = 100 100out in loss

in in

P P P

P P

Voltage regulation

1 1

1

100%n f

R

f

V VV

V

Speed regulation

1 1

1

100%n f

R

f

n nS

n

Positive SR means speed drops with load

Page 105: Dr. Robert A. Durham, PhD, PE Dr. Marcus O. Durham, PhD, PE … Nutshell Master.pdf · Dr. Robert A. Durham, PhD, PE . Dr. Marcus O. Durham, PhD, PE . A Quick Reference . The book

Page | 3-24

3.9 Transformer A transformer is a machine that does not rotate. Otherwise, it is

very similar to an induction AC machine. In application, the ideal

transformer is represented simply as two coils.

The model of a transformer fits the Thevenin equivalent output

with a magnetizing circuit all within a two port network

The relationship between input and

output sides are dependent on the turns

ratio, a.

in out

out in

V Ia

V I

The equivalent elements can be

referred to the primary or input.

1 pR R 1 pjX jX

22 sR a R 2

2 sjX ja X

Alternately, the circuit can be referred to the output or secondary.

1 2

pRR

a 1 2

pjXjX

a

2 sR R 2 sjX jX

R1 R2

Rc

Core

jX1 jX2

jXm

magnetizing

Air

Gap

Vp1

(AC)

+

-

I2I1

E1

Vp2

(AC)

+

-

Transformer

I1 I2

V1 V2

Page 106: Dr. Robert A. Durham, PhD, PE Dr. Marcus O. Durham, PhD, PE … Nutshell Master.pdf · Dr. Robert A. Durham, PhD, PE . Dr. Marcus O. Durham, PhD, PE . A Quick Reference . The book

Page | 3-25

3.9.1 Transformer Tests

Transformer tests are conducted according to the standard test procedures. The model is first

determined as a one-port circuit.

Open Circuit test

Conduct with the secondary or output

disconnected. Apply rated full voltage.

Most voltage drop is across the

excitation coil, so the tests yield values of

the core impedance.

Measure ocV , ocI , ocP

The calculations can refer to primary.

1 1

E

E c c

jY

Z R X

ocE

oc

IY

V

The angles are determined from measured magnitudes of power, voltage, and current.

cos oc

oc oc

Ppf

V I

EZ

EY

Short Circuit Test

Conduct with secondary winding shorted. Apply reduced voltage from a variac, and increase the

voltage until rated current is measured.

Most current flowing is low resistance series path. This shows values of copper impedance. Very

little current flows through excitation branch.

Measure scV , scI , SCP

Then calculate the impedance values.

scSE

sc

VZ

I

cos sc

sc sc

Ppf

V I

EZ

Core

Loss

Mag

I loss

R1

n2R2'n2jX2'

jX1'

IL I2

n2Z2V2'Non-ideal

approximation

Page 107: Dr. Robert A. Durham, PhD, PE Dr. Marcus O. Durham, PhD, PE … Nutshell Master.pdf · Dr. Robert A. Durham, PhD, PE . Dr. Marcus O. Durham, PhD, PE . A Quick Reference . The book

Page | 3-26

2 2( ) ( )

SE eq eq

p s p S

Z R jX

R a R j X a X

Because of the connections, we cannot separate

primary & secondary impedance, but usually this is not

necessary.

The impedances cause a shift in the angles

associated with the current and voltage drops.

Page 108: Dr. Robert A. Durham, PhD, PE Dr. Marcus O. Durham, PhD, PE … Nutshell Master.pdf · Dr. Robert A. Durham, PhD, PE . Dr. Marcus O. Durham, PhD, PE . A Quick Reference . The book

Page | 3-27

Transformer Characteristic Example

Ratings Open Circuit Values Short Circuit Values

20 kVA V=8,000V V=489V

8000/240V I=0.214A I=2.5A

60 Hz P=400W P=240W

Tests on primary

Open Circuit

Find values associated with magnetizing current.

400cos 0.234 lagging=76.5

(8000)(0.214)

oc

oc oc

Ppf

V I

0.21476.5 0.0000062 0.0000261

8000

ocE

oc

IY j

V

1 1

159 , 38.4

E

c m

c m

Y jR X

R k X k

Short Circuit

Find values associated with series current.

240cos 0.196 lagging=78.7

(489)(7.5)

sc

sc sc

Ppf

V I

48978.7 38.4 1.92

2.5

scse

sc

VZ j

I

38.4 1.92eq eqR X

These are the values associated with both windings in series– primary and secondary are not

separated

Page 109: Dr. Robert A. Durham, PhD, PE Dr. Marcus O. Durham, PhD, PE … Nutshell Master.pdf · Dr. Robert A. Durham, PhD, PE . Dr. Marcus O. Durham, PhD, PE . A Quick Reference . The book

Page | 3-28

3.9.2 Transformer Turns

Transformers consist of two inductors that are closely coupled. Usually an iron core provides an

improved magnetic path. Laminations are used in the iron to reduce the hysteresis and eddy current

losses. There are no moving parts to a transformer. It simply converts the voltage on one side to a

different voltage dependent on the number of turns on each side. The current is converted inversely to

the turns.

Transformer windings are identified either by location or by terminal markings. Primary windings are

labeled with "H". Secondary windings are identified with "X".

Subscripts identify the separate terminals

The coupling between the turns is determined by the polarity.

Normal polarity is subtractive. The same subscripts are aligned

between the primary and secondary. Additive polarity has the

opposite subscripts aligned.

The voltage (V) ratio between the primary and secondary is equal to the corresponding turns (N)

ratio.

p p

s s

V N

V N

The inverse of the current (I) ratio between the primary and secondary is equal to the turns (N)

ratio.

pS

P s

NI

I N

The same transformer can be used as a step-up or step-down unit. A step-up transformer has a

higher voltage and a lower current on the secondary. Conversely, a step-down transformer has a lower

voltage and higher current on the secondary.

An autotransformer has the secondary and the primary connected together. The voltage is placed

on the primary. One terminal becomes common with the output. The other

primary terminal is connected to one of the secondary terminals. The remaining

secondary terminal becomes the second output terminal. If the secondary is

connected with additive polarity, it is a boost connection. If the secondary is

connected with subtractive polarity, it is a buck connection.

The input is the common coil, cN , while winding 2 becomes the series coil,

SEN , which is added or subtracted from the input.

cL

H c se

NV

V N N

c seL

H c

N NI

I N

IL

I2V2

VHVL V1

I1 I2

V1 V2

Page 110: Dr. Robert A. Durham, PhD, PE Dr. Marcus O. Durham, PhD, PE … Nutshell Master.pdf · Dr. Robert A. Durham, PhD, PE . Dr. Marcus O. Durham, PhD, PE . A Quick Reference . The book

Page | 3-29

The apparent power into and out of the transformer must be equal.

in out ios s s

The apparent power in the windings is the same in the common and the series winding.

w c c se ses V I V I

So the ratio of the apparent power gives a “gain” or apparent power advantage.

io se c

w c

s N N

s N

Page 111: Dr. Robert A. Durham, PhD, PE Dr. Marcus O. Durham, PhD, PE … Nutshell Master.pdf · Dr. Robert A. Durham, PhD, PE . Dr. Marcus O. Durham, PhD, PE . A Quick Reference . The book

Page | 3-30

Example transformer turns:

Given: A transformer has a 120 volt primary and a 12 volt secondary. Primary current is 10

amps.

Find: Turns ratio

Secondary current

VA rating of each winding

120

12

p p p

s s s

V N N

V N N Turns ratio – 10:1

10

10 1

ps s

p s

NI I

I N

10*10

1001

sI

120*10 1200p p s sV I V I VA

Example autotransformer:

Given: Connect the transformer in the above example as a boost autotransformer with 120 volt

primary.

Find: output voltage

output current

output power

120 12 132Vout c sev v v

100out seI I A

132*100 13,200io H Hs V I VA

Page 112: Dr. Robert A. Durham, PhD, PE Dr. Marcus O. Durham, PhD, PE … Nutshell Master.pdf · Dr. Robert A. Durham, PhD, PE . Dr. Marcus O. Durham, PhD, PE . A Quick Reference . The book

Page | 3-31

Vt

IL

IARA Field

weak

3.10 DC Machines DC Machines are the typical example of a Thevenin equivalent for the armature and a magnetizing

inductance for the field. The field is a stationary stator and the armature is a rotating rotor.

The field can be connected separately, parallel, or in series. Other than changing the value of the

voltage, the calculations are very similar. However, the performance is very different.

Generic Relationships

The magnetic flux is determined by the field current.

Li R

mmf F FN IF

FF

F

VI

R

The flux couples to the armature to produce voltage and

torque.

indind A AK I I

K

1A FE K K i

A actual actual

A reference reference

E n

E n

Separately Excited

A separately excited machine has different

dc voltage on the field and the armature.

t A A AV E I R

L AI I

)(

FF

F adj

VI

R R

( )F F F FadjV I R R V

Shunt Connected

A shunt excited machines has the field connected across the terminals.

A F LI I I

F TV V

RfAdj

Rf

-

+A

Ra

Lf

mag fluxEa

Internal

Generated

+

-

Vt (DC)

DC

Machine

+

-

Vf (DC)

If Ia

Page 113: Dr. Robert A. Durham, PhD, PE Dr. Marcus O. Durham, PhD, PE … Nutshell Master.pdf · Dr. Robert A. Durham, PhD, PE . Dr. Marcus O. Durham, PhD, PE . A Quick Reference . The book

Page | 3-32

The terminal voltage controls flux, torque, and speed.

T A A AV E I R

T i F A AV K i I R

indT AV K R

K

2

T Aind

V R

K K

Series Connected

( )t A A A SV E I R R

A S LI I I

-

+A

RA +

-

IA Is&IL

Rs

Vt

Radj

RF

LF

compound

-

+A

RaRf adj

+

-

Vt

IA IL

Lf

Page 114: Dr. Robert A. Durham, PhD, PE Dr. Marcus O. Durham, PhD, PE … Nutshell Master.pdf · Dr. Robert A. Durham, PhD, PE . Dr. Marcus O. Durham, PhD, PE . A Quick Reference . The book

Page | 3-33

3.11 AC Machines - Synchronous AC machines are the typical example of a Thevenin equivalent for the armature and a magnetizing

inductance for the field. The armature is stationary and the field is on the rotor. The armature is AC and

is connected directly to the line. The field may be dc which creates a synchronous machine or ac which

creates an induction machine.

Synchronous machine field is created by a separate DC source on the rotor.

Induction machine field is creates magnetic induction on the stator.

Stator has 3φ or 1φ currents.

120º equal magnetic current. in a 3φ machine

2 pole (1N-1S) is the minimum number and results in a synchronous speed of 3600 rpm. The

synchronous speed (nsync) of the machine can be determined from the number of poles. There is

always an even number of poles.

120

#sync

fn

poles

2 f speed

3.11.1 Synchronous Machine

A synchronous machine rotation locks onto the power line frequency which results in a constant

speed, dependent only on the frequency. The model is conducted per phase. The model is for a motor

or generator.

A p S A A AE V jX I R I

sin

cos AA

S

EI

X

3 sinp A

out

S

V EP

X

The variables have the following meanings.

Xs=Synchronous Reactance

δ = torque angle:

max τ @ 90º

15 20

Power and torque are available on the shaft.

P

Rf

Adj

Rf

-

+A

Ra

Lf

mag fluxEa

Internal

Generated

+

-

Vp (AC)

Synchronous

Machine

+

-

Vf (DC)

If Ia

jXs

Page 115: Dr. Robert A. Durham, PhD, PE Dr. Marcus O. Durham, PhD, PE … Nutshell Master.pdf · Dr. Robert A. Durham, PhD, PE . Dr. Marcus O. Durham, PhD, PE . A Quick Reference . The book

Page | 3-34

3 sinp A

m s

V E

X

fK I

A generator has additional considerations.

1A fE K K i

( )f f f fadjV I R R

( )t A A A sV E I R jX

Li R

Droop

1n sys

droop

f f

f

1n droop sysf f f

Page 116: Dr. Robert A. Durham, PhD, PE Dr. Marcus O. Durham, PhD, PE … Nutshell Master.pdf · Dr. Robert A. Durham, PhD, PE . Dr. Marcus O. Durham, PhD, PE . A Quick Reference . The book

Page | 3-35

3.11.2 Induction Machine

The induction machine follows the Thevenin equivalent for the armature with a magnetizing

inductance for the field all within a two-port network; however, the magnetizing circuit cannot be

separated, but is an integral part of the machine. The field is on the stator and the armature is on the

rotor.

The induction machine is simply a transformer that has the secondary shorted and allowed to spin.

As such it is a purely AC machine with no DC field. The rotor field has shorted bars with voltage induced

from the stator field.

Because of the rotation, additional calculations are required compared to the transformer. The rotor

speed varies with load. The difference in the synchronous speed and the mechanical shaft speed is

called the slip.

Slip

Slip, s, is normalized or per unit.

100sync m

sync

s

100 100slip sync mech

sync synce

n n ns

n n

0 rotor @ sync speeds

1 rotor stationary (locked)s

slip sync mn n n - Slip Speed

(1 )m syncn s n

Example slip:

Speed = 1720, s = 0.04 = 4%

Losses

3 cosin T LP V I

air gap in stator cu coreP P P P

conv air gap rotor cu ind mP P P

R1 R2

R2 (1-s)/s

Mechanical

Rc

Core

jX1 jX2

jXm

magnetizing

Air

Gap

Vp

(AC)

+

-

I2I1

E1

Induction

Machine

Page 117: Dr. Robert A. Durham, PhD, PE Dr. Marcus O. Durham, PhD, PE … Nutshell Master.pdf · Dr. Robert A. Durham, PhD, PE . Dr. Marcus O. Durham, PhD, PE . A Quick Reference . The book

Page | 3-36

&out conv f w stray load mP P P P

Power & Torque

Only real elements consume power and convert it to mechanical energy or heat.

213stator cuP I R

21

23core c

c

EP R

R

air gap in s cu coreP P P P

2 223ag

RP I

s

22 23rotor cu agP I R sP

The developed mechanical power is dependent

on the slip.

conv ag r cuP P P

2 2 222 23 3

RI I R

s

22 2

(1 )3

sI R

s

& ( )out conv f w misc strayP P P P

This allows calculating locked rotor and no load examples of an induction machine.

The results give a combined X for the rotor and stator. These are separated based on the machine

design, whether A, B, C, D.

Core

Loss

f&w

R1 R2'

R’L

jX2'jX1'

2

(1 )L

sR R

s

22 2

(1 )sL

R R sR R R

s s

IL I2

Page 118: Dr. Robert A. Durham, PhD, PE Dr. Marcus O. Durham, PhD, PE … Nutshell Master.pdf · Dr. Robert A. Durham, PhD, PE . Dr. Marcus O. Durham, PhD, PE . A Quick Reference . The book

Page | 3-37

3.12 Transmission Lines Power transmission is investigated by making a two-port network for the line. The components are

distributed and are typically measured in /mile, /foot, or

/km. Work the problem on per- phase.

Reactance of most cables is published by the manufacturer.

The reactance in ohms per 1000 feet of aerial cables with one

foot spacing can be found with the following formula.

1

0.02298 lnLXGMR

GMR is the geometric mean radius in feet. It can be calculated by multiplying the wire O.D. in inches

by .03245.

0.25 0.3894 0.03245GMR R e D d

R = wire radius in feet

D = wire diameter in feet

d = wire diameter in inches

The reactance of aerial cable depends on the spacing between wires. Reactance at spacings other

than one foot can be calculated with the following formula.

ln

11

lnnew old

spacingX X

GMR

Cable operating temperature has an effect on the resistance of a cable. Most cables have a rated

operating temperature of 90 °C. Aerial cable is rated 75 °C. Cables have higher resistances at their rated

operating temperature ratings than at ambient temperature. The resistance at rated operating

temperature can be calculated from the resistance at ambient temperature using the following formula.

C/2 C/2

R L

Page 119: Dr. Robert A. Durham, PhD, PE Dr. Marcus O. Durham, PhD, PE … Nutshell Master.pdf · Dr. Robert A. Durham, PhD, PE . Dr. Marcus O. Durham, PhD, PE . A Quick Reference . The book

Page | 3-38

2 1 2 11R R T T

R2 = resistance at operating temperature

R1 = resistance at ambient temperature

T2 = rated operating temperature

T1 = ambient temperature

α = temperature coefficient of resistivity corresponding to temperature T1 (0.00393 for copper at 20

°C)

The current that causes the cable to reach its highest temperature may not be the maximum

available current. The current depends on the cable’s resistance and the resistance depends on the

current. The maximum available current should be the current that causes maximum temperature.

Page 120: Dr. Robert A. Durham, PhD, PE Dr. Marcus O. Durham, PhD, PE … Nutshell Master.pdf · Dr. Robert A. Durham, PhD, PE . Dr. Marcus O. Durham, PhD, PE . A Quick Reference . The book

Page | 3-39

3.13 Per Unit Notation Per unit notation is used to reduce the complexity when working with circuits that have multiple

voltage levels. Per unit is nothing more than performing calculations based on percentages. Both Ohm’s

law and the power relationship permit a third term to be calculated from only two terms.

Two parameters are selected as the reference or base values. These are generally S and V. A

different base V is used on each side of a transformer. The base current and base impedance can be

determined from these two values

basebase

base

SI

V

2base

base

base

VZ

S

All the circuit equipment voltages and currents are then converted to per unit (percentage) values

before normal circuit calculations are made

*100equip

pu

base

SS

S

*100equip

pu

base

VV

V

*100equip

pu

base

II

I

*100equip

pu

base

ZZ

Z

As an example, transformer impedance is usually rated in per unit values. To find the actual

impedance, combine the above equations

100

pu

equip base

ZZ Z

2

100

pu baseequip

base

Z VZ

S

An example illustrates the relationship between per unit values and short circuit capability.

Transformer, 10bases kVA , 120basev , 2%puZ

22120

1000.0288

10000equipZ

Page 121: Dr. Robert A. Durham, PhD, PE Dr. Marcus O. Durham, PhD, PE … Nutshell Master.pdf · Dr. Robert A. Durham, PhD, PE . Dr. Marcus O. Durham, PhD, PE . A Quick Reference . The book

Page | 3-40

10010000 5000

2

base

pu

SSCC kVA

Z

4167basesc

equip base equip

VV SCCI A

Z V Z (use pre-fault voltage)

Page 122: Dr. Robert A. Durham, PhD, PE Dr. Marcus O. Durham, PhD, PE … Nutshell Master.pdf · Dr. Robert A. Durham, PhD, PE . Dr. Marcus O. Durham, PhD, PE . A Quick Reference . The book

Page | 3-41

3.14 Short Circuit Considerations

3.14.1 Introduction

A short circuit condition differs from normal current operations only by virtue of a sudden decrease

in the circuit impedance. The decrease in impedance is caused by a fault.

The power source is generally rated by a short circuit capacity (SCC) rating in volt-amps. This is the

product of the pre-fault voltage and the post-fault current. Short circuit current is restricted only by the

source impedance, since the load is greatly reduced.

1scc pre scVA V I

3 3scc pre scVA V I

With the short circuit capability and the voltage rating, the source impedance can be determined.

The impedance calculated is for each phase, if the system is three-phase.

2p

source

VZ

SCC

The SCC of a magnetic device, such as a transformer or machine, can be found using the percent

impedance (Zpu) and the device rating

100

pu pu

kVASCC kVA

Z Z

The available fault current is also restricted by the SCC of the transformer

3

3

3SC

line

SCCI

V

1

1SC

line

SCCI

V

The available fault current is restricted by the source fault current and the transformer turns ratio.

sec

primary

ondary

VN

V

secondary primarySC SCI I N

The available fault current is the smaller value that is calculated using the two methods above.

Other impedance in the wiring will further restrict the fault current.

pre

SC

fault

VI

Z

Page 123: Dr. Robert A. Durham, PhD, PE Dr. Marcus O. Durham, PhD, PE … Nutshell Master.pdf · Dr. Robert A. Durham, PhD, PE . Dr. Marcus O. Durham, PhD, PE . A Quick Reference . The book

Page | 3-42

Short circuit contribution from induction machines continues after a fault. Inertia causes the

machine to continue turning with a collapsing magnetic field. This results in approx 25% of the

machine’s capability contributing to the fault current.

Page 124: Dr. Robert A. Durham, PhD, PE Dr. Marcus O. Durham, PhD, PE … Nutshell Master.pdf · Dr. Robert A. Durham, PhD, PE . Dr. Marcus O. Durham, PhD, PE . A Quick Reference . The book

Page | 3-43

3.14.2 Fault Analysis

Fault analysis is commonly called a short-circuit study. Fault current

differs from normal current only by an accidental decrease in circuit Z.

Under fault conditions, the load (ZL) may be 1 or more Ohm, while

the fault is ~0.0001 ohm.

The resulting Z is a parallel combination.

41 1 11 10 1

0.0001 1TZ

The load is negligible compared to the fault.

A Afault

T f

V VI

Z Z

Realistically, the ZT will provide a significant restriction on fault current. The first part of the process

is to select the base values for the one-line diagram.

Need complete one-line diagram

Convert to per unit (percent)

Normally pick bS & bV , and then calculate bI & bZ

bb

b

SI

V

2

2 3

3

bbbb

b b b

VVVZ

I S S

Because to the three-phase characteristics, we can use either per phase values or line values for 3-

phase current and voltage calculations

Do all calculations on single phase basis for Z. Impedance is a phase measurement only.

EA Zf ZL

Page 125: Dr. Robert A. Durham, PhD, PE Dr. Marcus O. Durham, PhD, PE … Nutshell Master.pdf · Dr. Robert A. Durham, PhD, PE . Dr. Marcus O. Durham, PhD, PE . A Quick Reference . The book

Page | 3-44

3.14.3 Symmetrical Components

Faults by definition indicate there is an imbalance in the system. This impacts the impedance and

the resulting current and voltage drops. Because of the asymmetry, the interaction in the calculation

would be very convoluted. A preferred method is to make a transform that converts the system to

balanced, and separates the components by the 120o normally expected in three-phase power systems.

The transform is called symmetrical components. The operator is α

0.5 0.866 1 120j

2 0.5 0.866 1 120j

Sequence Currents

Sequence currents are positive, negative, and zero sequence. These are found from the transform

operating on the phase currents.

213 1( )A A B C pI I I I I I

213 2( )A A B C nI I I I I I

130 ( )A A B C o GI I I I I I

Line Currents

Phase currents can be obtained by taking the inverse transform on the sequence currents.

0A A A AI I I I

0B B B BI I I I

0C C C CI I I I

Symmetrical components are used to take any unbalance combination of V & I and make them

operate as balanced 3φ model.

The only use is to aid the algebra. Symmetrical components are not “real”.

Page 126: Dr. Robert A. Durham, PhD, PE Dr. Marcus O. Durham, PhD, PE … Nutshell Master.pdf · Dr. Robert A. Durham, PhD, PE . Dr. Marcus O. Durham, PhD, PE . A Quick Reference . The book

Page | 3-45

3.14.4 Ratings & Reactances

Under fault conditions, the power system waveform goes through a transient condition until it

stabilizes. Three types of transient are used to describe the changes in state.

1. Momentary is the first cycle. Use all induction motors and subtransient (Xd’’) reactances

2. Interrupting is for contact parting. Neglect branches w/ pure induction motors and use only

transient (Xd’) reactances, except below 600V

3. Asymmetrical – use multiplier from tables (if not known, use 1.6)

Reactance values are dependent of frequency. Therefore, the reactance will change as the slope of

the waveform varies. There are three conditions.

Sub-transient is during the first cycle to 0.1 seconds of the fault.

Transient is during 30 cycles to 2 seconds.

Synchronous is the steady state rating.

Rules of Thumb

(steady state) 1.0dX

(transient) 0.33dX

(subtransient) 0.25 600dX for V

(subtransient) 0.2 600dX for V

Page 127: Dr. Robert A. Durham, PhD, PE Dr. Marcus O. Durham, PhD, PE … Nutshell Master.pdf · Dr. Robert A. Durham, PhD, PE . Dr. Marcus O. Durham, PhD, PE . A Quick Reference . The book

Page | 3-46

3.14.5 Short Circuit Study

A short circuit study is dependent on how the fault is connected within the one-line diagram.

1. Draw single line diagram w/ all sources of fault current, such as generators & motors, and

utility connections.

2. Replace all components, including reactance, with resistors (impedance) symbol, and label

with letters.

3. Show all transformer secondary feeding induction motors, whether motors are indicated or

not.

4. Join all components by “infinite bus” (neutral)

5. The source is not the infinite bus, but is simply another reactance.

6. Rearrange impedances into series & parallel.

7. Reduce to single Z.

8. Convert Δ blocks to star to further reduce (Thevenin Z)

Page 128: Dr. Robert A. Durham, PhD, PE Dr. Marcus O. Durham, PhD, PE … Nutshell Master.pdf · Dr. Robert A. Durham, PhD, PE . Dr. Marcus O. Durham, PhD, PE . A Quick Reference . The book

Page | 3-47

Example

The circuit in Figure 1 has a fault at X.

Calculate the Thevenin Z at the fault.

Use pre-fault V at the fault to find fault current, I.

1)

G

G1

M

M1

Induction

MS

M2

Synchronous

M

M3

Fault

2) Fault

T1

G1

M1 M2

M3

3) Fault

T1

G1M1 M2

M3

4) Fault

T1

M1//G1//M2 M3

5) Fault

M1//G1//M2+T1 M3

6) Fault

Ztotal=(M1//G1//M2+T1)//M3

Page 129: Dr. Robert A. Durham, PhD, PE Dr. Marcus O. Durham, PhD, PE … Nutshell Master.pdf · Dr. Robert A. Durham, PhD, PE . Dr. Marcus O. Durham, PhD, PE . A Quick Reference . The book

Page | 3-48

3.14.6 Unbalanced Faults

The previous fault analysis development was for a 3-phase fault.

0A B CI I I

Unbalanced conditions are redefined in terms of 3 components.

Positive sequence (+, p, 1)

System with sources rotating

“Normal” conditions

Negative sequence (-, n, 2)

The negative sequence is the same circuit as positive, but without sources.

Z may have different value

Zero Sequence (0, Z, 0)

The zero sequence circuit is simply the ground path.

Δ has no ground

has no Ground

has a ground path

Draw three circuits, one for each sequence.

Leave sources in positive

Make negative w/o sources

Zero indicates ground paths

Connect each sequence with fault impedances for each component.

3Zf represents the fault impedance in pos, neg & zero sequences.

Page 130: Dr. Robert A. Durham, PhD, PE Dr. Marcus O. Durham, PhD, PE … Nutshell Master.pdf · Dr. Robert A. Durham, PhD, PE . Dr. Marcus O. Durham, PhD, PE . A Quick Reference . The book

Page | 3-49

3.14.7 Faults with Rotating Machines

EA vA

neutral

AV E Z I

0V Z I

0 0 00V Z I

For 3φ faults, use the positive sequence only.

0A B CI I I

0A A A AI I I I

For one phase, calculate the symmetrical current..

13A AI I , since 0A A AI I I

The current is drawn at the fault.

The zero sequence is the path through ground. It will

change depending on the xformer connection or Y .

Z0 will be different since transformer & machine ground

path may not be connected

Z0 motor or gen = 3 nZ

Z0 = 0 for solidly connected neutrals

In zero sequence, include machine impedance to ground.

3Zn=3 times impedance of any phase

Use in place of source voltage

An assumption can be made for the sub-transient reactance.

120 0d motor dZ Z

EA ZL

Xs+

ZL

Xs-

Xs0

IA+

IA-

IA0

3Zf

Page 131: Dr. Robert A. Durham, PhD, PE Dr. Marcus O. Durham, PhD, PE … Nutshell Master.pdf · Dr. Robert A. Durham, PhD, PE . Dr. Marcus O. Durham, PhD, PE . A Quick Reference . The book

Page | 3-50

3.14.8 Fault Illustrations

A one-line diagram shows a generator, transformers, transmission line, and motor load. Convert the

one-line into positive, negative, and zero sequences.

G M1 2

Xng

Xg XL

XTu XTd

Xm

Up dn

Xnm

EgEm

1 2

Xg1 XL1 Xm1Xu1 Xd1

Positive

Sequence

1 2

Xg2 XL2 Xm2Xu2 Xd2

Negative

Sequence

Xg0 XL0 Xm2Xu0 Xd0

Zero Sequence3Xng0 3Xnm0

If the connection on a transformer to ground is , then insert the connection and

impedance. Else, leave the ground connection open.

Illustrations are developed for four types of

faults.

Three phase fault at 2, use positive sequence

only.

0 2 0V V

1

f

eqt

VI

Z

0 2 0I I

Single phase line to ground at 2, use positive, negative, and zero sequence in series.

0 1 2 13 fV V V Z I

1

1 2 0 3

fault

f

VI

Z Z Z Z

1 2 oI I I

Line to line, use positive in series with negative

and connect with Zf.

Eg

Zg1 ZL1

3Zf

Zu1 Zd1

short

Eg

Zg1 ZL1

3Zf

Zu1 Zd1

short

Vpu

Z1 Z2Zf

+

-V1

+

-

V2

Page 132: Dr. Robert A. Durham, PhD, PE Dr. Marcus O. Durham, PhD, PE … Nutshell Master.pdf · Dr. Robert A. Durham, PhD, PE . Dr. Marcus O. Durham, PhD, PE . A Quick Reference . The book

Page | 3-51

1

1 2

f

f

VI

Z Z Z

1 2I I

Double line to ground, use positive with negative in parallel

with zero.

1

1 2 0( || 3 )f

VfI

Z Z Z Z

1 2 0I I I

Vpu

Z1 3Zf

Z2

+

-

V0

+

-

V2 Z0

Page 133: Dr. Robert A. Durham, PhD, PE Dr. Marcus O. Durham, PhD, PE … Nutshell Master.pdf · Dr. Robert A. Durham, PhD, PE . Dr. Marcus O. Durham, PhD, PE . A Quick Reference . The book

Page | 3-52

Symmetrical RMS Fault Currents and Voltages in Terms of Sequence Impedances

Three-phase fault through three-phase fault impedance Zf

Line-to-Line fault. Phase b and c shorted through fault impedance Zf

Line-to-line fault. Phase a grounded through fault impedance Zf

Double line-to-ground fault. Phases b and c shorted, then grounded through fault impedance Zf

aI 1

f

f

V

Z Z 0

0 1 2

3

3

f

f

V

Z Z Z Z 0

bI 2

1

f

f

a V

Z Z

1 2

3f

f

Vj

Z Z Z

0

0 2

1 2 1 2 0

33

3

f

f

f

Z Z aZj V

Z Z Z Z Z Z

cI 1

f

f

aV

Z Z

1 2

3f

f

Vj

Z Z Z 0

20 2

1 2 1 2 0

33

3

f

f

f

Z Z a Zj V

Z Z Z Z Z Z

aV 1

f

f

f

ZV

Z Z

2

1 2

2 f

f

f

Z ZV

Z Z Z

0 1 2

3

3

f

f

f

ZV

Z Z Z Z

2 0

1 2 1 2 0

3 2

3

f

f

f

Z Z ZV

Z Z Z Z Z Z

bV 2

1

f

f

f

a ZV

Z Z

22

1 2

f

f

f

a Z ZV

Z Z Z

22 0

0 1 2

3 3

3

f

f

f

a Z j Z aZV

Z Z Z Z

2

1 2 1 2 0

3

3

f

f

f

Z ZV

Z Z Z Z Z Z

cV 1

f

f

f

aZV

Z Z

2

1 2

f

f

f

aZ ZV

Z Z Z

22 0

0 1 2

3 3

3

f

f

f

aZ j Z a ZV

Z Z Z Z

2

1 2 1 2 0

3

3

f

f

f

Z ZV

Z Z Z Z Z Z

bcV

1

3f

f

f

Zj V

Z Z

1 2

3f

f

f

Zj V

Z Z Z

0 2

0 1 2

3 23

3

f

f

f

Z Z Zj V

Z Z Z Z

0

caV

2

1

3f

f

f

a Zj V

Z Z

22

1 2

33

f

f

f

a Z j Zj V

Z Z Z

20 2

0 1 2

33

3

f

f

f

a Z Z Zj V

Z Z Z Z

2 0

1 2 1 2 0

3 33

3

f

f

f

Z Z ZV

Z Z Z Z Z Z

abV

1

3f

f

f

aZj V

Z Z

2

1 2

33

f

f

f

aZ j Zj V

Z Z Z

0 2

0 1 2

33

3

f

f

f

a Z Z Zj V

Z Z Z Z

2 0

1 2 1 2 0

3 33

3

f

f

f

Z Z ZV

Z Z Z Z Z Z

Page 134: Dr. Robert A. Durham, PhD, PE Dr. Marcus O. Durham, PhD, PE … Nutshell Master.pdf · Dr. Robert A. Durham, PhD, PE . Dr. Marcus O. Durham, PhD, PE . A Quick Reference . The book

Page | 3-53

Page 135: Dr. Robert A. Durham, PhD, PE Dr. Marcus O. Durham, PhD, PE … Nutshell Master.pdf · Dr. Robert A. Durham, PhD, PE . Dr. Marcus O. Durham, PhD, PE . A Quick Reference . The book

Page | 3-54

Page 136: Dr. Robert A. Durham, PhD, PE Dr. Marcus O. Durham, PhD, PE … Nutshell Master.pdf · Dr. Robert A. Durham, PhD, PE . Dr. Marcus O. Durham, PhD, PE . A Quick Reference . The book

Page | 3-55

Chapter 3 Problems Practice Problem 3-1(Old Style)

SITUATION:

A paper mill is supplied by a 13.8kV, 3-phase, 60 Hz system with appropriate transformers.

The total load is as follows:

Induction Motors 600Hp Efficiency – 85% power factor – 0.8 lagging

Heating and Lighting 100kW unity power factor

Synchronous Motors 200Hp efficiency – 90% leading power factor

The synchronous motors are being operated at rated kVA and are over excited to correct the plant

power factor to 0.95 lagging.

It is desired to increase the mill output by 20%

A plant survey indicates that the installed induction motor capacity is adequate for this increase, but

the synchronous machines are at rated kVA.

It is suggested that it might be possible to increase the output of the synchronous motors by a

sufficient amount by reducing the excitation to unity power factor and providing power factor

correction with a static bank of capacitors.

REQUIREMENTS:

Determine if it is possible to increase the output of the synchronous motors by 20% by reducing

excitation without exceeding kVA ratings. Explain your answer

Determine the capacitance per phase, delta connected, to correct the power factor to 0.95 lagging if

the capacitors are connected across the 13.8kV line.

Page 137: Dr. Robert A. Durham, PhD, PE Dr. Marcus O. Durham, PhD, PE … Nutshell Master.pdf · Dr. Robert A. Durham, PhD, PE . Dr. Marcus O. Durham, PhD, PE . A Quick Reference . The book

Page | 3-56

SOLUTION:

Given Existing System:

Induction motors:

600 0.746 1* * 526.6

1 85%( )

Hp kWP kW

Hp eff

526.6

658.20.8( )

kWS kVA

pf

2 2658.2 526.6 394.9Q kVAR

Heat & Light:

100P kW

100

1001.0( )

kWS kVA

pf

2 2100 100 0Q kVAR

Synchronous Motors:

200 0.746 1

* * 165.81 90%( )

Hp kWP kW

Hp eff

a) Plant:

792.4P kW

792.4

834.10.95( )

kWS kVA

pf

2 2834.1 792.4 260.5Q kVAR

Synchronous Motor:

260.5 394.9 134.4

plant indQ Q Q

kVAR

165.8 134.4 213.4 39.09S j

cos( 39.09) 0.776pf

By reducing exciting of synchronous motors to give unity power factor, P of synchronous motors

increases to 213.4kW.

S P Q Pf

Induction Motors 658.2 526.6 394.9 0.8 Heat & Light 100 100 0 1.0 Synchronous Motors 165.8

Plant 792.4

S P Q Pf

Induction Motors 658.2 526.6 394.9 0.8 Heat & Light 100 100 0 1.0 Synchronous Motors 165.8

Plant 834.1 792.4 260.5 0.95

S P Q Pf

Induction Motors 658.2 526.6 394.9 0.8 Heat & Light 100 100 0 1.0 Synchronous Motors 213.4 165.8 -134.4 -0.78

Plant 834.1 792.4 260.5 0.95

Page 138: Dr. Robert A. Durham, PhD, PE Dr. Marcus O. Durham, PhD, PE … Nutshell Master.pdf · Dr. Robert A. Durham, PhD, PE . Dr. Marcus O. Durham, PhD, PE . A Quick Reference . The book

Page | 3-57

% increase = (213.4 165.8)

28.7%165.8

So, synchronous load increased by more than 20%, planned upgrades are possible.

b) With 20% plant increase (20% increase in

induction motor load and 20% increase in

synchronous motor load)

Q needed for 95% pf = 1930.9tan(cos 0.95) 930.9tan(18.2 ) 306.1kVAR

95% 396.9 306.1 90.8cap total pfQ Q Q kVAR

/

90.830.3

3 3

cap

cap ph

Q kVARQ kVAR

2V

SZ

2

c

VQ

X

/

2 2

30,3000.422 /

13,800 *(2 60)

cap ph

ph

QC F ph

V

S P Q Pf

Induction Motors 789.9 631.9 473.9 0.8 Heat & Light 100 100 0 1.0 Synchronous Motors 213.4 199 -77 0.93

Plant 1012.0 930.9 396.9 0.92

Page 139: Dr. Robert A. Durham, PhD, PE Dr. Marcus O. Durham, PhD, PE … Nutshell Master.pdf · Dr. Robert A. Durham, PhD, PE . Dr. Marcus O. Durham, PhD, PE . A Quick Reference . The book

Page | 3-58

Practice Problem 3-2 (old style) 136

A generator is connected through a step-up transformer to a very large system (infinite bus).

The generator is rated: The transformer is rated:

840 MVA

90% power factor

24kV

Xd = 160%

Xd' = 25%

Xd'' = 18%

The transformer is delivering 600MW at 85% power factor to the system at 500kV when the

operator increases the output to 750MW without changing the setting of the generator voltage

regulator so that the generator terminal voltage is unchanged.

The operator then adjusts the voltage regulator so that the output of the transformer is 800VA.

REQUIREMENTS:

Determine the generator terminal voltage and the new transformer power factor and MVA output

after the power output is increased, but before the voltage regulator is adjusted.

Determine the generator terminal voltage and the transformer output power factor after the

voltage regulator is adjusted

Note: Answer to at least four (4) significant figures.

SOLUTION:

Generator: 840MVA, 90% pf, 24kV

Transformer: 800MVA, 22kV pri/500kV sec, 14%tX

Secondary connected to infinite bus – V fixed, f fixed

Select Per Unit Base – use transformer

800baseS MVA

2 500baseV kV

1 22baseV kV

2

8001.6

500baseI kA

S VI (cos sin )I I j *P S pf

1 2 1 21 2

sin( )VVP

X

21 1 2 1 2

1 2

cos( )V V VQ

X

800 MVA

22 – 500 kV

X = 14%

G

V1 V2

jXs jXT

Page 140: Dr. Robert A. Durham, PhD, PE Dr. Marcus O. Durham, PhD, PE … Nutshell Master.pdf · Dr. Robert A. Durham, PhD, PE . Dr. Marcus O. Durham, PhD, PE . A Quick Reference . The book

Page | 3-59

1 1 2 2;V V

For infinite bus, fixed V&f , vary P&Q

For fixed load, fixed P&Q, vary V&f

Start Solution

Initial Conditions:

600

705.880.85

MWS MVA

pf

1705.880.8824 (cos 0.85) 0.8824 31.79

800pu

MVAS

MVA

2 1.0 0puV (infinite bus)

* 0.8824 31.790.8824 31.79

1.0 0

pu

pu

pu

SI

V

0.8824 31.79 (0.75 4649)puI j

21 (0.75 0.4649)( 0.14) (0.6508 0.105)tV IjX j j

1 2 21 (1 0) (0.06508 0.105)

1.0651 0.105

1.0703 5.6303

V V V j j

j

Check using power transfer equation

1 2 1 21 2

sin( )

1.0702*1*sin(5.6303 0)0.75 600

0.14

V VP

X

MW

Page 141: Dr. Robert A. Durham, PhD, PE Dr. Marcus O. Durham, PhD, PE … Nutshell Master.pdf · Dr. Robert A. Durham, PhD, PE . Dr. Marcus O. Durham, PhD, PE . A Quick Reference . The book

Page | 3-60

Increase power to 750MW – no change to voltage

750

0.9375800

puP

2 1 0V (infinite bus)

1 1.0702V

The angle will change since P increases, but voltage magnitude has not changed

1 2 1 2 11 2 1 2

1 2

sin( )sin( )

VV P XP

X V V

1 1

0.9375*0.14sin( 0) 7.0445

1.0702*1

1 1.0702 7.0445 1.0621 0.1313V j

1 2( ) (1.0621 0.1313) (1 0)

0.14

0.9375 0.4436 1.0375 25.31

t

V V j jI

jX j

j

25.31S Z I

1cos ( 25.31) 0.904pf

* 1 0*1.0375 25.31 1.0375 25.31outS VI

* 800*1.0375 830rate base puS S S MVA

1 1 1* 22 *1.0702 23.54rate base puV V V kV kV

Page 142: Dr. Robert A. Durham, PhD, PE Dr. Marcus O. Durham, PhD, PE … Nutshell Master.pdf · Dr. Robert A. Durham, PhD, PE . Dr. Marcus O. Durham, PhD, PE . A Quick Reference . The book

Page | 3-61

Change voltage so transformer is at rated MVA (800MVA)

800 1.0puS MVA S

1

11

pu

pu

pu

SI

V

2 2

750750 0.9375

800puP MW P

2 2 2

2

2 2

cos

0.9375cos 0.9375

1*1

20.364

P V I

P

V I

(cos sin )

1(0.9375 0.3480) 1 20.364

I I j

j

1 2

1 (0.9375 0.3480)( 0.14)

1 0.0487 0.13125 1.0487 0.13125

1.05689 7.13363

tV V IjX

j j

j j

Check using power transfer equation

1 2 1 21 2

sin( )

1.05689*1*sin(7.133 0)0.9375 750

0.14

V VP

X

MW

1 22 *1.05689 23.252rateV kV kV

Page 143: Dr. Robert A. Durham, PhD, PE Dr. Marcus O. Durham, PhD, PE … Nutshell Master.pdf · Dr. Robert A. Durham, PhD, PE . Dr. Marcus O. Durham, PhD, PE . A Quick Reference . The book

Page | 3-62

Practice Problem 3-3 (old style)

SITUATION:

A refinery is planning to install a compressor-expander connected to an induction machine.

Under certain conditions the induction machine, taking 8 MVA at 85% power factor, will act as a

motor to drive the compressor-expander to compress a certain gas.

At other times, the compressor-expander will act as a turbine to drive the electric machine as a

generator supplying 6 MW at 82% power factor.

Also connected to the plant bus:

Induction motors taking 4 MVA at 86% pf

Incandescent lighting taking 2MW

Synchronous motor taking 2 MW at 90% pf

The plant will receive the necessary power from the utility at 13.8kV

REQUIREMENTS:

In order to formulate the conceptual design of the new substation, determine the plant load in MW,

amperes and power factor that the utility must supply under the following conditions:

When the induction machine is driving the compressor-expander.

When the compressor-expander is driving the induction machine.

SOLUTION:

Keep track of P’s and Q’s – Solve for S, I and pf.

System – Induction motor driving compressor expander

Induction motors:

4S MVA

0.86pf

4 *0.86 3.44P MVA MW

2 24 3.44 2.041Q MVAR

Lighting

2P MW

1.0pf

2

2.01.0

MWS MVA

S P Q Pf

Induction Motors 4 0.86 Lighting 2 1.0 Synchronous Motors 2 0.9 leading Compressor-Expander 8 0.85

Plant

Page 144: Dr. Robert A. Durham, PhD, PE Dr. Marcus O. Durham, PhD, PE … Nutshell Master.pdf · Dr. Robert A. Durham, PhD, PE . Dr. Marcus O. Durham, PhD, PE . A Quick Reference . The book

Page | 3-63

2 21 1 0Q MVAR

Synchronous Motors

2P MW

0.9( )pf leading

2

2.2220.9

MWS MVA

2 22.222 2 0.9686Q MVAR

Compressor – Expander

0.85pf

8S MVA

8

6.80.85

MVAP MW

2 28 6.8 4.215Q MVAR

a) 14.24plantP MW

14.24 5.286 15.189 20.37plantS j

cos20.37 0.9375pf

*

*

3

3

15.189635.5

3 *13.8

S VI

SI

V

MVAAmp

kV

b) Same as above, but compressor-expander driving induction generator

Compressor – Expander

S P Q Pf

Induction Motors 4 3.44 2.041 0.86 Lighting 2 2 0 1.0 Synchronous Motors 2.222 2 -0.9686 0.9 leading Compressor-Expander 8 6.8 4.214 0.85

Plant 14.24 5.286

Page 145: Dr. Robert A. Durham, PhD, PE Dr. Marcus O. Durham, PhD, PE … Nutshell Master.pdf · Dr. Robert A. Durham, PhD, PE . Dr. Marcus O. Durham, PhD, PE . A Quick Reference . The book

Page | 3-64

6.0P MW

0.82pf

6.0

7.3170.82

S MVA

2 2( 7.317) ( 6.0) 4.188Q MVAR

1.44plantP MW

1.44 5.260 5.454 74.69plantS j

cos74.69 0.264pf

*

3

5.454228.2

3 *13.8

SI

V

MVAAmp

kV

S P Q Pf

Induction Motors 4 3.44 2.041 0.86 Lighting 2 2 0 1.0 Synchronous Motors 2.222 2 -0.9686 0.9 leading Compressor-Expander -7.317 -6.0 4.188 0.85

Plant 1.44 5.26

Page 146: Dr. Robert A. Durham, PhD, PE Dr. Marcus O. Durham, PhD, PE … Nutshell Master.pdf · Dr. Robert A. Durham, PhD, PE . Dr. Marcus O. Durham, PhD, PE . A Quick Reference . The book

Page | 3-65

Practice Problem 3-4 (old style – similar to new style)

SITUATION:

In order to accommodate the needs of larger plant power systems, a switchgear manufacturer is

offering a new line of vacuum circuit breakers rated in accordance with ANSI C 37.04 “Rating Structure

for A-C High Voltage Circuit Breakers.

The following data apply:

Nominal voltage: 7.2kV

Nominal 3-phase MVA class: 700 MVA

Rated maximum voltage: 8.25kV

Rated voltage range factor (k): 1.3

Rated insulation level – low frequency: 36kV

Rated continuous current: 3000 A

Rated short circuit current: 46kA

Rated interrupting time: 5 cycles

REQUIREMENTS:

Determine the following related required capabilities (to three significant figures)

The symmetrical interrupting capability with the prefault voltage @ 7.2kV

The symmetrical interrupting capability with the prefault voltage @ 6.1kV

The symmetrical interrupting capability with the prefault voltage @ 8.5kV

Closing and latching capability operating voltage at 7.2kV

Closing and latching capability operating voltage at 6.1kV

Closing and latching capability operating voltage at 8.5kV

Three-second short time current carrying capability operating @ 7.2kV

If the circuit breaker is applied at a point where the system impedance is 0.08Ω/phase, what is the

change in margin of symmetrical interrupting capability over short circuit current available when the

operating voltage is changed from 7.2kV to 7.4kV?

Page 147: Dr. Robert A. Durham, PhD, PE Dr. Marcus O. Durham, PhD, PE … Nutshell Master.pdf · Dr. Robert A. Durham, PhD, PE . Dr. Marcus O. Durham, PhD, PE . A Quick Reference . The book

Page | 3-66

SOLUTION:

Symmetrical interrupting capability at 7.2kV

Max kVA remains the same, so, as voltage goes down, current goes up, as long as voltage is within

voltage range (1.3)

8.25

1.157.2

, so keep constant kVA

3 3max 8.2546 10 52.71 10

7.2SC rated

nom

VI I Amp

V

Symmetrical interrupting capability at 6.1kV

Max kVA remains the same, so, as voltage goes down, current goes up, as long as voltage is within

voltage range (1.3)

8.25

1.356.1

, so limit voltage ratio to 1.3

3 3*1.3 46 10 *1.3 59.8 10SC ratedI I Amp

Equipment is rated at 8.25kV, do not use at 8.5kV

Max interrupting current: 3 31.3*46 10 59.8 10mI Amp

Refer to ANSY C 37.04 standards

Closing & latching is 1.6 times the max current (at any voltage below rated)

3& *1.6 95.68 10c l mI I Amp

Closing & latching is 1.6 times the max current (at any voltage below rated)

3& *1.6 95.68 10c l mI I Amp

Equipment is rated at 8.25kV, do not use at 8.5k

Three second current capability = max interrupting current (3s < 5s rating)

3 31.3*46 10 59.8 10mI Amp

7.2kV is L-L voltage, need L-N (phase) voltage

Page 148: Dr. Robert A. Durham, PhD, PE Dr. Marcus O. Durham, PhD, PE … Nutshell Master.pdf · Dr. Robert A. Durham, PhD, PE . Dr. Marcus O. Durham, PhD, PE . A Quick Reference . The book

Page | 3-67

7200

4156.93 3

LLLN

VV V

System Z is 0.08,

3720051.96 10

3 *0.08scI Amp

From a) above, Max interrupting current at 7.2kV is 52.71x103 Amp, so margin is 748 Amp

at 7.4kV, short circuit current is

3740053.406 10

3 *0.08scI Amp

Max short circuit current is

3 3maxmax

8.2546 10 51.283 10

7.4SC rated

nom

VI I Amp

V

So, equipment is overrated by 2,122 Amps. Change in margin is 2,870 Amps.

Page 149: Dr. Robert A. Durham, PhD, PE Dr. Marcus O. Durham, PhD, PE … Nutshell Master.pdf · Dr. Robert A. Durham, PhD, PE . Dr. Marcus O. Durham, PhD, PE . A Quick Reference . The book

Page | 3-68

Practice Problem 3-5(old style)

SITUATION:

An industrial plant has four generators connected to a 13.8kV bus as shown in Figure 433 below. The

neutral of each generator is connected through a neutral circuit breaker to a common neutral bus. This

bus is connected through a neutral resistor to ground.

G

G1

G

G2

G

G3

G

G3

Individual generator reactances in pu:

2

0

0.16

0.16

0.08

dX j

X j

X j

REQUIREMENTS:

a) Determine the value of the neutral resistor required to limit a line-to-ground fault to that of

a three-phase fault when only one of the units is operating.

b) If a 0.1 pu neutral resistor is used, determine the ground fault current in pu when all units

are operating, but with only one neutral circuit breaker closed.

c) Same as b., except all neutral circuit breakers closed.

SOLUTION:

Estimate positive, negative and zero sequence impedances from data given.

1

2 2

0 0

0.16

0.16

0.08

dZ X j

Z X j

Z X j

a) f resistorZ Z

Page 150: Dr. Robert A. Durham, PhD, PE Dr. Marcus O. Durham, PhD, PE … Nutshell Master.pdf · Dr. Robert A. Durham, PhD, PE . Dr. Marcus O. Durham, PhD, PE . A Quick Reference . The book

Page | 3-69

3

1

f

ph

f

vI

z z

1

0 1 2

3

3

f

ph

f

vI

Z Z Z Z

Operating at 100% voltage (1.0 pu)

3

1.06.25

0.16 0phI j

j

1

3*1.0 3

0.16 0.08 0.16 3 0.4 3ph

resistor resistor

Ij j j Z j Z

Set 1phI = 3 phI and solve for resistorZ

36.25

0.4 3

30.4 3

6.25

3 0.48 0.4

0.026667

resistor

r

R

R

jj Z

j Zj

Z j j

Z j

b)

1

0 1 2

12 2 2 2

3 3*1

3 0.08 0.04 0.04 3* 0.1

3 3

( 0.16) (0.3)

8.8235

f

ph

f

ph

vI

Z Z Z Z j j j j

Ij X R

pu

c) With all breakers closed, Zo=0.02j

1

0 1 2

12 2 2 2

3 3*1

3 0.08 0.04 0.02 3* 0.1

3 3

( 0.1) (0.3)

9.487

f

ph

f

ph

vI

Z Z Z Z j j j j

Ij X R

pu

3 x 0.1

j0.16

j0.16

j0.08

j0.16

j0.16

j0.08

j0.16

j0.16

j0.08

j0.16

j0.16

j0.08

Positive sequence

impedance

Negative sequence

impedance

Zero sequence

impedance

One breaker

closed

Page 151: Dr. Robert A. Durham, PhD, PE Dr. Marcus O. Durham, PhD, PE … Nutshell Master.pdf · Dr. Robert A. Durham, PhD, PE . Dr. Marcus O. Durham, PhD, PE . A Quick Reference . The book

Page | 3-70

Problem 3-6 (old)

SITUATION:

A 1000/1250 kVA, OA/FA, 13.2kV:4160V single phase transformer is part of a 3000/3750 kVA Y-Δ

bank.

Factory tests are made on this transformer at 25°C and the following data recorded.

DC Resistance: r1 = 0.40 Ω r2 = 0.035 Ω

With secondary open and 13.2kV applied to the primary: I1 = 10A, Pin = 5500W

With secondary shorted and 800V applied to the primary: I1 = 75.76A , Pin = 5800W

Assume the three single phase transformers are equal.

REQUIREMENTS:

For the operating temperature of 75°C, determine:

The percent effective resistance on the self-cooled rating base

The percent reactance on the self-cooled rating base.

The percent impedance on the self-cooled rating base

The no-load loss of the three-phase bank (kW)

The total loss of the three-phase bank (kW) with the transformer operating at its force cooled rating.

The efficiency of the bank carrying 3750 kVA at 85% pf

Background

1000/1250 kVA OA/FA

13.2kV/4.16 kV

DC Resistance: r1=0.40Ω r2=0.035Ω

Open Circuit Test: V1=13.2kV I1=10A Pin=5500W

Short Circuit Test: I1=75.76A Pin=5800W

Fan Load = 750W

Sbase=1,000 kVA Vbase=13.2kV Zbase=2 2(13,200)

174.241,0000,000

base

base

V

S

Turns Ratio: 13.2

3.1734.16

p

s

Va

V

Page 152: Dr. Robert A. Durham, PhD, PE Dr. Marcus O. Durham, PhD, PE … Nutshell Master.pdf · Dr. Robert A. Durham, PhD, PE . Dr. Marcus O. Durham, PhD, PE . A Quick Reference . The book

Page | 3-71

Solution:

(a)Percent effective (ac) resistance on the self-cooled rating base

ac dc core mechr =r +r + r (rmech is 0 for transformer)

Equivalent dc resistance referred to primary:

2 21 2(25 ) 1.4 3.173 *0.035 0.7524dcr C r a r

Effective resistance from short circuit test

2 21

5800(25 ) 1.0105

(75.76)

inac e

Pr r C

I

The components of the ac resistance at test temperature

ac dc core

core

core

r (25 C)=r (25 C)+r (25 C) 1.0105

1.0105 0.7524 r (25 C)

r (25 C) 1.0105 0.7524

0.2581

Resistance changes with temperature.

rdc increases with temp (positive temp coeff)

rcore resistance decreases with temp (negative temp coeff)

∆R = α T0

∆T

or

R = R0 [ 1 – α (T – T0)]

For copper, the inferred absolute zero coefficient is -234.4.

So the equation reverts to

R = 234.4 + T

R0 234.4 + T0

Apply to both the copper and the core resistance.

ac

234.5 75 234.5 25r (75 C)=0.7524 0.2581

234.5 25 234.5 75

1.1138

Convert to per unit.

1.1138

( 75 ) 0.006392 0.6392%174.24

acr pu C

Page 153: Dr. Robert A. Durham, PhD, PE Dr. Marcus O. Durham, PhD, PE … Nutshell Master.pdf · Dr. Robert A. Durham, PhD, PE . Dr. Marcus O. Durham, PhD, PE . A Quick Reference . The book

Page | 3-72

(b) Percent reactance on the self-cooled rating base

Impedance

Z = V = √R2 + X2

2 2

2 2

80010.56

75.76

10.56 1.1138

10.501

10.501( ) 0.060267 6.0267%

174.24

scac

sc

ac ac ac

ac

ac

VZ

I

X Z r

X

X pu

(c) Percent impedance on the self-cooled rating base

10.56( ) 0.0606 6.06%

174.24acZ pu

(d) No-load loss of 3 phase bank (from open circuit test)

Pno-load=3*Pin = 3*5500 = 16.5kW

(e) Total loss of 3 phase bank operating at FA rating

S = VI* → I = S / V

2

2

125094.697

13.2

3*( )

3*(94.697 *1.1138 5500)

46.46

FA

lossFA FA ac no load

kVAI A

kV

P I r P

kW

(f) Efficiency

3750 *0.85

3,187.5

3,187.598.54%

(3,187.5 46.46 0.75 )

out

out

in

P kVA pf

kW

P kWeff

P kW kW kW

Page 154: Dr. Robert A. Durham, PhD, PE Dr. Marcus O. Durham, PhD, PE … Nutshell Master.pdf · Dr. Robert A. Durham, PhD, PE . Dr. Marcus O. Durham, PhD, PE . A Quick Reference . The book

Page | 3-73

Problem 3-7 (old) 436

SITUATION:

A generating station is connected as shown in Figure Problem 2-7 below. Transformer T2 was destroyed

and must be replaced; however, no records exist of the nameplate, and the proper phase relations must

be determined so that a new transformer can be specified.

REQUIREMENTS:

Neatly sketch and label phasors A'B'C', and state sequence A'B'C' or C'B'A'.

Neatly sketch and label phasors A''B''C'' and state sequence A''B''C'' or C''B''A''

Complete the nameplate for T2 – ratings not required.

H0 H1 H2 H3

X1 X2 X3

GENT2

H1 H2 H3

X0 X1 X2 X3

T1

A’B’

C’

ABC

N

B’’C’’N

A’’

T3 H0 H1 H2 H3

X0 X1 X2 X3

A

C

B

N

H3

H1H2

H0

X3

X1

X2

H3

H1

H2

H3

H1H2

H0

X3

X1X2

X0

T1

T2

T3

Page 155: Dr. Robert A. Durham, PhD, PE Dr. Marcus O. Durham, PhD, PE … Nutshell Master.pdf · Dr. Robert A. Durham, PhD, PE . Dr. Marcus O. Durham, PhD, PE . A Quick Reference . The book

Page | 3-74

SOLUTION:

This is a problem about phase sequences. It illustrates the phase shifting between (1) wye and delta,

(2) between line-line and line-ground, and (3) between line and phase. Although these are obviously

related, the actual connections can be quite different.

Delta Delta Wye Wye

Phase Line Phase Line

L-L L-L L-N L-L

Phase sequence is drawn from the perspective of looking down the x-axis to the left. The phasors

rotate CCW. Record the phase sequence AN, BN, CN or CN, BN, AN or record the line sequence AB, BC,

CA or CA, BC, AB. Select every other letter. The sequence is positive ABC or negative CBA.

For a transformer the terminals are labeled on the primary and secondary.

Neutral

Primary H1 H2 H3 H0

Secondary Additive X1 X2 X3 X0

Transformers in a wye-delta configuration are shown. Note the

corresponding orientation that does not result in a phase shift. AN-

XY, BN-YZ, CN-ZX

Steps for determining transformer connection. Make a table of

the line connections and the transformer connections. Fill in the

rows of unknowns. Note the order that data is filled.

Order Action Options

1 Reference phase AN BN CN or AB BC CA

AN BN CN

2 Transformer primary connection H1 H2 H3 H0

3 Primary actual phase/line connection AN BN CN or AB BC CA

4 Transformer secondary connection X1 X2 X3 X0

5 Secondary actual phase/line connection AN BN CN or AB BC CA

6 Orientation of primary & secondary draw sketch

0o 120o 240o or 90o 210o 330o

7 Sequence ABC or CBA

A B

N

C

X

Y

Z

Page 156: Dr. Robert A. Durham, PhD, PE Dr. Marcus O. Durham, PhD, PE … Nutshell Master.pdf · Dr. Robert A. Durham, PhD, PE . Dr. Marcus O. Durham, PhD, PE . A Quick Reference . The book

Page | 3-75

Transformer T3 is a wye-wye. The primary and secondary are aligned in phase.

Order Action Connection

1 Reference phase AN BN CN or AB BC CA

AN BN CN

2 Transformer primary connection H1 H2 H3 H0

H1H0 H2H0 H3H0

3 Primary actual phase/line connection AN BN CN or AB BC CA

AN BN CN

4 Transformer secondary connection X1 X2 X3 X0

X1X0 X2X0 X3X0

5 Secondary actual phase/line connection AN BN CN or AB BC CA

C”N B”N A”N

6 Orientation of primary & secondary draw sketch

0o 120o 240o or 90o 210o 330o

0 o 120 o 240o

7 Sequence ABC or CBA CBA

Transformer T1 is a wye-delta. The primary and secondary are shifted in phase.

Order Action Connection

1 Reference phase AN BN CN or AB BC CA

AN BN CN

2 Transformer primary connection H1 H2 H3 H0

H3H0 H2H0 H1H0

3 Primary actual phase/line connection AN BN CN or AB BC CA

AN BN CN

4 Transformer secondary connection X1 X2 X3 X0

X3X1 X2X3 X1X2

5 Secondary actual phase/line connection AN BN CN or AB BC CA

A’C’ B’A’ C’B’

6 Orientation of primary & secondary draw sketch

0o 120o 240o or 90o 210o 330o

0 o 120 o 240o

7 Sequence ABC or CBA ABC

Page 157: Dr. Robert A. Durham, PhD, PE Dr. Marcus O. Durham, PhD, PE … Nutshell Master.pdf · Dr. Robert A. Durham, PhD, PE . Dr. Marcus O. Durham, PhD, PE . A Quick Reference . The book

Page | 3-76

Transformer T2 is a delta-wye. The primary & secondary are shifted in phase. The secondary

orientation is unknown

Order Action Options

1 Reference phase AN BN CN or AB BC CA

AN BN CN

4 Transformer primary connection H1 H2 H3 H0

H1H3 H2H1 H3H2

2 Primary actual phase/line connection AN BN CN or AB BC CA

A’C’ B’A’ C’B’

5 Transformer secondary connection X1 X2 X3 X0

X1X0 X2X0 X3X0

3 Secondary actual phase/line connection AN BN CN or AB BC CA

C”N B”N A”N

6 Orientation of primary & secondary draw sketch

0o 120o 240o or 90o 210o 330o

90o 210o 330o

7 Sequence ABC or CBA CBA

Page 158: Dr. Robert A. Durham, PhD, PE Dr. Marcus O. Durham, PhD, PE … Nutshell Master.pdf · Dr. Robert A. Durham, PhD, PE . Dr. Marcus O. Durham, PhD, PE . A Quick Reference . The book

Page | 4-1

Chapter 4 - Electronics

4.1 Introduction Electronics involves a small signal operating on active devices to produce a different signal. Active

devices are non-linear energy amplifiers. Passive devices are simply modeled as RLC components.

Electronics is predominantly small signal variations that exist around a larger operating point. This fits

the standard response or solution, without the exponent, or essentially the cos( )t portion.

( ) cost

y t F I F e t

The constant part of the signal is the power supply, called the bias. The dc voltage and current

define an operating point , quiescent point (q-point), or bias. The bias dictates what portion of the curve

the device is operating on.

The small signal is the AC component. Generally AC analysis involves investigating a transfer

function operating on the input signal. The function is a two port model of the circuit.

The signal voltage and current cause slight variations around the operating point defined on the dc

bias line.

0 1 cosi I I t

ln1

1ln

new old

spacingX X

GMR

oI and oV are the DC operating point. The cos ωt term is the signal varying about the operating

point.

Electronic functions are by definition non-linear systems. In order to simplify the solution, it is

necessary to use assumptions.

Linearize with time varying about operating point is appropriate if

1 0cos( )I t I

For AC small signal, the DC supply voltage appears as a short

Page 159: Dr. Robert A. Durham, PhD, PE Dr. Marcus O. Durham, PhD, PE … Nutshell Master.pdf · Dr. Robert A. Durham, PhD, PE . Dr. Marcus O. Durham, PhD, PE . A Quick Reference . The book

Page | 4-2

4.1.1 Solid State Device Characteristics

Electronics often use semiconductors. The conductivity of a semiconductor material is a physics

property.

n pq n p

electron mobilityn

hole mobilityp

electron concentrationn

hole concentrationp

-19charge on electron (1.6 10 )q C

The silicon material is doped to change the number of electrons (negative) or holes (positive).

p-type material; p ap N

n-type material; n dn N

Carrier concentrations at equilibrium are the product of the p and n concentration.

2( )( ) ip n n

in = intrinsic concentration

Built in potential or contact potential of a p-n junction depends on Boltzmann’s constant,

temperature and the electrical charge.

2

ln a do

i

N NkTV

q n

Thermal voltage is the constant, temperature, and charge components.

0.026T

kTV V

q at 300°K

Na = acceptor concentration

Nd = donor concentration

T = temperature (K)

k = Boltzmann’s Constant = 1.38x10-23 J/K

Page 160: Dr. Robert A. Durham, PhD, PE Dr. Marcus O. Durham, PhD, PE … Nutshell Master.pdf · Dr. Robert A. Durham, PhD, PE . Dr. Marcus O. Durham, PhD, PE . A Quick Reference . The book

Page | 4-3

Capacitance of abrupt p-n junction diode is a function of the voltage.

( )1

o

bi

CC V

VV

Co = junction capacitance at V=0

V = potential of anode with respect to cathode

Vbi = junction contact potential

Resistance of a diffused layer is based on a sheet of material.

s

LR R

W

Rs = sheet resistance = d

in ohms per square

= resistivity

d = thickness

L = length of diffusion

W = width of diffusion

Page 161: Dr. Robert A. Durham, PhD, PE Dr. Marcus O. Durham, PhD, PE … Nutshell Master.pdf · Dr. Robert A. Durham, PhD, PE . Dr. Marcus O. Durham, PhD, PE . A Quick Reference . The book

Page | 4-4

4.2 Boundary Conditions The response to a signal is investigated at three different conditions: the midpoint, the upper limit

or boundary, and the lower limit or boundary. Frequency changes the impedance and the time constant.

As a result, the voltage and gain of the circuit is changed.

The upper and lower boundary analysis determines the frequency of the transition.

High Frequency Cutoff – Shunt Capacitance dominates

f

Series capacitor is conducting 1

02

se

se

XfC

Frequency is determined by shunt capacitance& parallel resistors.

1 2

1 1 1 1c shC C

RC R R R

Low Frequency Cutoff – Series resistance and capacitance dominates

0f

Shunt capacitor is open 1

2sh

sh

XfC

Frequency is determined by series (coupling) capacitor and resistance in series.

1 2

1c serR R R C C

RC

3 dB Frequency

The response is the voltage that changes with frequency. It has an exponential rise to the low cutoff,

transitions to a bandwidth that is relatively flat, then transitions to an exponential decay past the upper

cutoff. The cut-off frequencies are called 3dB because the voltage has decayed to that value.

Vmax

.707 Vmax

ω ωoω3db

Page 162: Dr. Robert A. Durham, PhD, PE Dr. Marcus O. Durham, PhD, PE … Nutshell Master.pdf · Dr. Robert A. Durham, PhD, PE . Dr. Marcus O. Durham, PhD, PE . A Quick Reference . The book

Page | 4-5

3 2 between +3db and -3dbndb

20log o

in

Vdb

V

30.15 log

20

o

in

Vdb

V

0.1510 2 o

in

V

V

3db point @ 2o inV V

Attenuation

Attenuation is the reduction in signal. It is influence by the cut-off of each stage in an amplifier.

ATF (Attenuation Factor) = 1

2 1n

n= # of stages

*high onef f ATF

fone =one stage frequency

fhigh = all stage high frequency

flow = all stage low frequency

onelow

ff

ATF

Page 163: Dr. Robert A. Durham, PhD, PE Dr. Marcus O. Durham, PhD, PE … Nutshell Master.pdf · Dr. Robert A. Durham, PhD, PE . Dr. Marcus O. Durham, PhD, PE . A Quick Reference . The book

Page | 4-6

4.3 Diodes & Rectifiers Diodes are the simplest semi-conductor device that consists of a single p-n junction. Functionally,

these are rectifiers.

4.3.1 Diodes

Diodes are non-linear as noted in the waveforms. They are used for power supplies, wave shaping,

and logic. For most circuit analysis, the ideal model of the diode can be used. The assumptions for an

ideal diode are zero current in reverse direction and zero voltage drop in forward direction. Real diodes

are somewhat less idyllic.

Ideal Diode

I=0+ -

V=0- +

Real Diode

(Piecewise Linear)

+ - - +

The Shockley equation defines the V/I relationship in a real diode.

1D

T

vV

D si I e

sI = saturation current

η = emission coefficient, typically 1 for Si

TV = thermal voltage = kT

q

Page 164: Dr. Robert A. Durham, PhD, PE Dr. Marcus O. Durham, PhD, PE … Nutshell Master.pdf · Dr. Robert A. Durham, PhD, PE . Dr. Marcus O. Durham, PhD, PE . A Quick Reference . The book

Page | 4-7

4.3.2 Rectifiers & Clippers

The application or performance of diodes is a rectifier. Representative circuits and their output are

shown. The input voltage Vin is a sinusoid.

Input Waveform

Half Wave Rectifier

RL VoAC

Full Wave Rectifier

RLVin

Vo

Voltage Clipper

V1

RLVo

AC

Page 165: Dr. Robert A. Durham, PhD, PE Dr. Marcus O. Durham, PhD, PE … Nutshell Master.pdf · Dr. Robert A. Durham, PhD, PE . Dr. Marcus O. Durham, PhD, PE . A Quick Reference . The book

Page | 4-8

4.4 Operational Amplifiers

Ideal

An ideal op-am has infinite input impedance, zero output impedance, and almost infinite gain. The

gain, A is described by the relationship.

1 2( )ov A v v

A is large (>104) and v1 and v2 are small enough so the amplifier is not

saturated. For the ideal operational amplifier, assume that the input

currents are zero and that the gain A is infinite. Then, when operating

linearly, the voltage difference is 2 1 0v v .

Differential Amplifier

Two ungrounded input terminals create a

differential amplifier. The device amplifies the

voltage difference between the inputs.

The general form is shown. The significant

relationships are calculated.

out

in

VA

V

The op amp has a high Z input. That implies the

voltage between the terminals is essentially 0. So,

there is no current flow.

V- = V+

Compare the current on the negative and positive input branches. Use those functions to calculate

the output voltage.

01

1 2

V VV VI

Z Z

2

3 4

0V V VI

Z Z

The circuit is a voltage divider between the feedback side and balancing side.

2 4 1 20 1 2

1 1 3 4

Z Z Z ZV V V

Z Z Z Z

(Voltage divider)

-

+

v2

v1

vo

V1

Z1

V2

Z3

Z2

Z4

-

+

Vo

Page 166: Dr. Robert A. Durham, PhD, PE Dr. Marcus O. Durham, PhD, PE … Nutshell Master.pdf · Dr. Robert A. Durham, PhD, PE . Dr. Marcus O. Durham, PhD, PE . A Quick Reference . The book

Page | 4-9

The connection of the op-amp changes the gain. Complex reactance creates mathematics functions

of integrator and differentiator.

Inverting Amplifier

Voltage on – terminal

2 2

1 1

Z RA

Z R

-

+

R1 R2

ViA

Non-Inverting Amplifier

Voltage on + terminal

1 2

1

R RA

R

-

+

R1 R2A

Vi

Integrator

Capacitor in feedback

2

1 1 1

1 1 1ZA

Z R sC sCR

-

+

R1 AVi

C

Differentiator

Capacitor on input

22 2

1

( )( )Z

A R sC sCRZ

-

+

AVi

R2

C

Page 167: Dr. Robert A. Durham, PhD, PE Dr. Marcus O. Durham, PhD, PE … Nutshell Master.pdf · Dr. Robert A. Durham, PhD, PE . Dr. Marcus O. Durham, PhD, PE . A Quick Reference . The book

Page | 4-10

4.5 Transistors Transistors are the next level of complexity above diodes for semi-conductor devices. A transistor

consists of two junctions. Functionally, transistors are amplifiers. When operating at a boundary

condition, such as in digital circuits, a transistor is a switch.

I

vbe

Q-point

time

time

Vbe

cI

Input Signal

Output Signal

4.5.1 Bias vs. Small Signal

Bias is the DC voltage that determines the operating point. Small signal is the AC message that is

superpositioned on the DC.

Transistor Bias – External

Transistor biasing is a process of external connections similar to those in the figure. Because of the

connection, we know certain items.

Know:

Voltage @ ground = 0

Voltage @ Rc = Vcc

Want to find:

Voltage @ base. It is a divider of Vcc

The Thevenin equivalent input is a voltage divider of the power supply. If Rb

>>10R2, then the base current can be ignored and VBG can be easily determined.

1

1 2

CCBG Thevenin

V RV V

R R

R1

R2

Rc

RE

VL

VccB

C

E

Page 168: Dr. Robert A. Durham, PhD, PE Dr. Marcus O. Durham, PhD, PE … Nutshell Master.pdf · Dr. Robert A. Durham, PhD, PE . Dr. Marcus O. Durham, PhD, PE . A Quick Reference . The book

Page | 4-11

The Thevenin resistance is obtained by shorting the power supply (VCC). The equivalent

circuit is shown in the figure.

where

Next, find the current.

Current @ base (input)

Current @ emitter (output)

The analysis must include loop elements. In essence do per unit conversion. The current through the

transistor has gain. IB is the base current. If Rbase (=hfeRE) is not >> than 210* R , then VBG must be

determined from this circuit.

The operating point of the transistor is referred to as the Quiescent point (Q point). This is the point

that the circuit will operate at when only the DC signal is in place. The Q-point is defined by the Ic

current at the Q-point (ICQ) and VCE at the Q-point (VCEQ). For an existing circuit, the Q-point can be

determined by using the Thevenin equivalents.

2

1 2

1 2

( )

TH CC

TH

TH BECQ

THE

FE

CEQ CC CQ C E

RV V

R R

R R R

V VI

RR

h

V V I R R

Often when designing an amplifier

circuit, a graphical approach is easier. The

Q-point is the intersection of the DC load

line and the AC load line. The DC load line is

the line drawn between the

1 2/ /THR R R

1CC

THCC TH

VR R

V V

2CC

THTH

VR R

V

( 1)

( 1)

E B C B

TH BE TH B E E

TH BEB

TH E

I I I I

V V R I R I

V VI

R R

R1

R2

VBGVcc

RTH

RE

IE

VTH

VBE

IB

DC

CCV

R

CQI

CEQVCCV

Page 169: Dr. Robert A. Durham, PhD, PE Dr. Marcus O. Durham, PhD, PE … Nutshell Master.pdf · Dr. Robert A. Durham, PhD, PE . Dr. Marcus O. Durham, PhD, PE . A Quick Reference . The book

Page | 4-12

Collector/Emitter current when the transistor is conducting (shorted) and the Collector voltage (VCC).

Collector/Emitter current is VCC divided by RDC where

The load line is shown superimposed on the transistor characteristic curves in the figure to the right.

The next step is to select a Q-Point. The goal for most linear (Class A) amplifiers is to select a Q-point

near the middle of the load line. This allows for an equal amount of swing above and below the Q-point.

Once the Q-point is selected, VCEQ and ICQ can be determined.

The AC load line is the line that represents the small signal that is superimposed on the DC bias

circuit. AC signals bypass (short) VCC; it also includes the load impedance (RL). The endpoints of the ac

load line are found using the following

formulae

( )

( )

( )

( )

CEQ CE off

c sat CQ

C ac

CE off CEQ CQ C

ac C L E

V vi I

R R

v V I R

R R R R

The ideal situation for a Class A

amplifier is for the AC load line is for the Q-

point to be in the middle of the AC load

line. This can be adjusted by modifying RC.

(no AC coupling)dc C ER R R

DC

CCV

R

V

CQI

CEQV

( )CE off

ac

v

R

( )CE offvCCV

Page 170: Dr. Robert A. Durham, PhD, PE Dr. Marcus O. Durham, PhD, PE … Nutshell Master.pdf · Dr. Robert A. Durham, PhD, PE . Dr. Marcus O. Durham, PhD, PE . A Quick Reference . The book

Page | 4-13

4.5.2 Transistor Mathematical Relationships

Name and Schematic Symbol Mathematical Relationships

NPN Bipolar Junction Transistor (BJT)

Relationships valid in active mode of operation.

PNP Bipolar Junction Transistor (BJT)

Relationships valid in active mode of operation.

N-Channel Junction Field Effect Transistor (JFET)

P-Channel Junction Field Effect Transistor (JFET)

Cutoff Region:

Triode Region:

Saturation Region:

C

E

B

iC

iE

iB

E B Ci i i

C Bi i

C Ei i

1

BE

T

VV

C si I e

emitter saturation currentSI

thermal voltageTV

C

E

B

iC

iE

iB

E B Ci i i

C Bi i

C Ei i

1

BE

T

VV

C si I e

emitter saturation currentSI

thermal voltageTV

D

S

G

iD

iS

D

S

G

iD

iS

GS pv V

0Di

GS Pv V

GD Pv V

2

22DSS

D DS GS P DS

p

Ii v v V v

V

GS Pv V

GD Pv V2

1 GSD DSS

P

vi I

V

Page 171: Dr. Robert A. Durham, PhD, PE Dr. Marcus O. Durham, PhD, PE … Nutshell Master.pdf · Dr. Robert A. Durham, PhD, PE . Dr. Marcus O. Durham, PhD, PE . A Quick Reference . The book

Page | 4-14

N-Channel Depletion MosFET (NMOS)

IDSS = drain current with (in the saturation region

K = conductivity factor VP = pinch off voltage

P-Channel Depletion MosFET(NMOS)

Same as N-Channel, with current directions and voltage polarities reversed

D

S

G

iD

iS

B

N-Channel Enhancement MosFET (NMOS)

Cutoff Region:

GSv Vt

0Di

Triode Region:

GSv Vt

GD tv V

22D DS GS t DSi K v v V v

Saturation Region:

GS tv V

GD tv V

2

D GS ti K v V

IDSS = drain current with 0GSv (in the

saturation region K = conductivity factor Vt = pinch off voltage

D

S

G

iD

iS

B

P-Channel Enhancement MosFET(NMOS)

Same as N-Channel, with current directions and voltage polarities reversed

D

S

G

iD

iS

B

0GSv

2

DSS PI KV

D

S

G

iD

iS

B

Page 172: Dr. Robert A. Durham, PhD, PE Dr. Marcus O. Durham, PhD, PE … Nutshell Master.pdf · Dr. Robert A. Durham, PhD, PE . Dr. Marcus O. Durham, PhD, PE . A Quick Reference . The book

Page | 4-15

4.5.3 General Two-port Models

Two-port models have external measurements which take three forms; impedance, admittance, or

hybrid. Impedance models have voltage in terms of current. Admittance models have current in terms of

voltage. Hybrid models have both voltage and current as outputs. The internal connections have three

arrangements: T or wye, Π or delta, and L.

4.5.4 BJT Transistor Models

Bipolar Junction Transistor – Use Common emitter as general example

The complete model is shown on the left, while the simplified version is on the right.

The hybrid two-port network is the generic form for the BJT transfer function and includes the

current, voltage, and hybrid parameters, h.

1 1 2

2 1 2

i r

f o

v h i h v

i h i h v

The definition of the hybrid parameters are in terms in input (1) , and output (2).

General Form Specific to Common Emitter

2

1

1 0

i in

v

vh R

i

(1 )ie ib b ich h i h

2

2

1 0

f

v

ih

i

1

cfe

b

ih

i

1

1

2 0

r

i

vh

v

= gain of circuit

1

2

2 0

1o

i

ih

v out

= gain of transistor

hi

hrV2 hfi11/ho

i1 i2B

E

C

V1 V2

hfe

ib ib

hfeib

B

E

C

Page 173: Dr. Robert A. Durham, PhD, PE Dr. Marcus O. Durham, PhD, PE … Nutshell Master.pdf · Dr. Robert A. Durham, PhD, PE . Dr. Marcus O. Durham, PhD, PE . A Quick Reference . The book

Page | 4-16

The second subscript is determined by which terminal connects to the common lead for the two-

port model.

For common base – b input – e, output – c

For common emitter – e input – b, output – c

For common collector – c input – b, output – e

Common collector is also called an emitter follower.

Common collector circuits permit assumptions which greatly simplify the circuit.

2 0reh v

1

0 ignoreoe

o

hh

H- parameters are real at low frequencies and complex a high frequencies. At high frequencies, the

capacitance has more effect.

Since this is a small signal investigation on internal parameters, the DC supply appears as a short.

Page 174: Dr. Robert A. Durham, PhD, PE Dr. Marcus O. Durham, PhD, PE … Nutshell Master.pdf · Dr. Robert A. Durham, PhD, PE . Dr. Marcus O. Durham, PhD, PE . A Quick Reference . The book

Page | 4-17

Common Emitter – Hybrid Parameters

The common emitter is the most common configuration for

transistor amplifiers.

The h-parameters for a common emitter model are listed.

4

Q point

10coe

ce

ih

v

Q point

cfe

b

ih

i

3

Q point

(25 10 ) febe Tie fe

b EQ EQ

hv Vh h

i I I

i feA h

The small signal circuit removes the DC components.

The equivalent circuit input and output impedance are found.

b iei ie

b ie

R hZ h

R h

If the base bias resistor Rb>>hie, then the output is greatly

simplified.

1

o

oe

Zh

Conditions Small Signal Equivalent Model

Complete

hi

hrV2 hfi11/ho

i1 i2B

E

C

V1 V2

As hre0

hie hfeib1/hoe

ib icB

E

C

vbe vce

iB

ic

+

V

be

-

+

Vce

-

iL

Rb

VBB Re

RL

Ce

VCC

i1

RbRL

ib

ic

ie

+

Vbe-

+

Vce-

iL

Page 175: Dr. Robert A. Durham, PhD, PE Dr. Marcus O. Durham, PhD, PE … Nutshell Master.pdf · Dr. Robert A. Durham, PhD, PE . Dr. Marcus O. Durham, PhD, PE . A Quick Reference . The book

Page | 4-18

As hre0 and hoe e0

hie hfeib

ib icB

E

C

vbe vce

Complete common-

emitter amplifier

hie hfeib

ib icB

E

C

vbe vceRbRLii iL

ZiZo

Page 176: Dr. Robert A. Durham, PhD, PE Dr. Marcus O. Durham, PhD, PE … Nutshell Master.pdf · Dr. Robert A. Durham, PhD, PE . Dr. Marcus O. Durham, PhD, PE . A Quick Reference . The book

Page | 4-19

Hybrid Model – Common Emitter Model at High Frequency

The hybrid is a two port network defined in terms of both h-parameters and circuit parameters.

As a result, it incorporates both the non-linear transistor and a two-port model as shown below.

B

C

E

B C

E

rbb’rb’e Cb’e

rb’crce

Cb’c

Eb’

e

Ebe Ece

b’

Because of the capacitance, the frequency has a major impact on impedance. The Miller effect

resolves the situation where the voltages at both ends of a capacitor change at the same time. Rather

than consider the actual effect, the model uses a large capacitor, CMiller . The Miller replaces Cb’c. At low

freq assume Cm is included in Cb’c. The circuit assumptions are noted.

Cmiller '(1 )Lb c

d

RC

r

' ' '

0.025@ 300

fe

ie bb b e bb

EQ

hh r r r T K

I

β = common emitter low frequency current gain = 1

feh

Ebe = diode drop forward biased b-e voltage 0.6V

The equivalent circuit has the input side coupled to the output by the input voltage operating on the

transconductance, gm. By using superposition, the input is calculated first. Then the results are used on

the output side.

Page 177: Dr. Robert A. Durham, PhD, PE Dr. Marcus O. Durham, PhD, PE … Nutshell Master.pdf · Dr. Robert A. Durham, PhD, PE . Dr. Marcus O. Durham, PhD, PE . A Quick Reference . The book

Page | 4-20

Rbb’

ReVbe Zc

gmVbe

B

E

C

Input Side

Calculated 1st

Output Side

Use input to

calculate

gmVbe

B

E

C

Rce

The frequency effects are calculated. At the cutoff frequency, the gain is down 3dB.

Cutoff frequency = ' ' ' ' '

1 1

2 ( ) 2b e b e b c b e b e

fr C C r C

3 db bandwidth = fef h f 1

fb

i

ve

hA

jh

The internal circuit parameters have the following relationships.

'

40 300fe

m EQ

b e

hg I T K

r

' 10 50

2 5 (hi freq caps)

bbr

'

'

fe mb e

T b e T

h gC

r

'

0.26

40

fe

b e

EQ E

hr

I I

1 1' 2 3

5 pb c ob cbC C v pf p

Page 178: Dr. Robert A. Durham, PhD, PE Dr. Marcus O. Durham, PhD, PE … Nutshell Master.pdf · Dr. Robert A. Durham, PhD, PE . Dr. Marcus O. Durham, PhD, PE . A Quick Reference . The book

Page | 4-21

Common Base – Hybrid Parameters

A common base is used for high frequency applications because the base separates the input and

output, minimizing oscillations at high frequency. It has a high voltage gain, relatively low input

impedance and high output impedance compared to the common collector.

1 0

11

e

cob

o cb i i

ih

Z v

0

1

cb

cfb

e v

ih

i

0

1cb

eb ieTib i

e EQ fev

v hVh Z

i I h

i fbA h

A simplified circuit model is developed.

Complete Circuit

ri

vi

VCC

iE iLRL

R2R1Cb

iC

Small Signal Circuit

ri

vi

iE

iL

RLiC

Page 179: Dr. Robert A. Durham, PhD, PE Dr. Marcus O. Durham, PhD, PE … Nutshell Master.pdf · Dr. Robert A. Durham, PhD, PE . Dr. Marcus O. Durham, PhD, PE . A Quick Reference . The book

Page | 4-22

Small Signal Equivalent Model

ri

E

vi hrbvcb

hib

hebi1 1/hob

RL

B

C

iE

iL

iC

i1

Simplified Small Signal Equivalent

Model

0rbh

1fbh

0obh

ri

E

vi

hib

i1 RL

B

C

iE

iL

iC

i1

Page 180: Dr. Robert A. Durham, PhD, PE Dr. Marcus O. Durham, PhD, PE … Nutshell Master.pdf · Dr. Robert A. Durham, PhD, PE . Dr. Marcus O. Durham, PhD, PE . A Quick Reference . The book

Page | 4-23

T Model – Common Base Model at High Frequency

The T-model is simply a configuration that uses conventional circuit analysis without special

parameters.

re

Ce

rb

Zc

IcIee

b

c

At low frequency, capacitors are high impedance and appear as open circuits.

e eZ R

'' is close to unity(1)b e

e m b e

e

VI i g V

r

c cZ R

At high frequency, capacitors are low impedance and appear as a short circuit which passes signals.

1 1

2e e e

e t e

Z sC CR f r

1 tc b

c

fZ I I

sC f

c E cBOI I I

The variables have the following meanings.

α = fraction of emitter current collected

Icbo = reverse current characteristics of collector base junction

IE = dc value of emitter current

ft = high frequency performance (where hfe decreases to unity)

The emitter resistance is the base to emitter diode resistance. It is also called rb’e & rd.

26 T

E

E e

Vr

I r at room temperature

The typical ranges of values are noted.

Rb range 10’s to 100’s ohms – base spreading resistor

α range 0.9 to 0.999

rc range hundreds megohms

Vb’e 0.6v (0.5 – 0.7v) which is diode drop forward bias voltage.

Page 181: Dr. Robert A. Durham, PhD, PE Dr. Marcus O. Durham, PhD, PE … Nutshell Master.pdf · Dr. Robert A. Durham, PhD, PE . Dr. Marcus O. Durham, PhD, PE . A Quick Reference . The book

Page | 4-24

Page 182: Dr. Robert A. Durham, PhD, PE Dr. Marcus O. Durham, PhD, PE … Nutshell Master.pdf · Dr. Robert A. Durham, PhD, PE . Dr. Marcus O. Durham, PhD, PE . A Quick Reference . The book

Page | 4-25

Common Collector (Emitter Follower)– Hybrid Parameters

The common collector amplifier is also called an emitter follower because the output is taken from

the emitter resistor. The connection has a gain of near unit. It can be an impedance matching device

since its input impedance is much higher than its output impedance. It is used as a buffer in digital

circuits.

'

1

io ib

fe

rZ h

h

( 1)i ie fe eZ h h R

1vA

Complete Circuit

ri

vi

iE

ib

+

vE

-

BC

E

RE

RB

VCC

Small Signal Circuit ri

viiE

Rb

ib

+

vE

-

BC

E

RE

+

vb

-

Small Signal Equivalent Model

Looking into base

ri

viiE

hib

Rb +

vE

-

B

C

E

RE

+ vbe -

Page 183: Dr. Robert A. Durham, PhD, PE Dr. Marcus O. Durham, PhD, PE … Nutshell Master.pdf · Dr. Robert A. Durham, PhD, PE . Dr. Marcus O. Durham, PhD, PE . A Quick Reference . The book

Page | 4-26

Small Signal Equivalent Model

Looking into emitter

v’i

iE

hib

+

vE

-

B

C

E

RE

+ vbe -

'

1

i

fe

r

h

Page 184: Dr. Robert A. Durham, PhD, PE Dr. Marcus O. Durham, PhD, PE … Nutshell Master.pdf · Dr. Robert A. Durham, PhD, PE . Dr. Marcus O. Durham, PhD, PE . A Quick Reference . The book

Page | 4-27

Emitter Follower (common collector) Model at High Frequency

High frequency performance is very different because of the capacitors created in the device and in

the circuit. Therefore, alternative analysis assumptions are needed.

Circuit

Ri

vi Cb’c

Cb’e

Re

i1 ib

ie

vb

iCb’e

iRe

Equivalent Circuit

Ri

vi Cb’c

Cb’e

Ze

i1

vb

rb’e

ve

The frequency impact on the impedance is readily observable.

1

( 1)1

Te fe

j

Z hj

Assumptions reduce the complexity of the problem.

Cb’c is relatively small, and '1fe e b e ih R r R

e

1' R

1( )

1 1

T

i

T

j

ev R

i

vA

v j

Upper cutoff frequency 11 i

e

Th R

R

ff

If Cb’c is large enough for its impedance to be equal to Ri at fh2<<fh1, then the gain is in terms of h-

parameters.

Page 185: Dr. Robert A. Durham, PhD, PE Dr. Marcus O. Durham, PhD, PE … Nutshell Master.pdf · Dr. Robert A. Durham, PhD, PE . Dr. Marcus O. Durham, PhD, PE . A Quick Reference . The book

Page | 4-28

''

1 1( )

11

fe e

v

i b cb e fe e

h RA

j R Cr h R

Upper cutoff frequency 2

'

1

2h

i b c

f fR C

If 1 2h hf f the gain changes.

2 2

1

111

T

i

e

j

v RR

h T T h

Aj j

j

Page 186: Dr. Robert A. Durham, PhD, PE Dr. Marcus O. Durham, PhD, PE … Nutshell Master.pdf · Dr. Robert A. Durham, PhD, PE . Dr. Marcus O. Durham, PhD, PE . A Quick Reference . The book

Page | 4-29

4.5.5 FET Transistor Models

Field Effect Transistors (FET) are devices that are less affected by frequency. They can also handle

more power, in the range of amps. They are also more expensive.

Circuit

Vdd

RdId

Drain

SourceGate

RsRs CsVin

Equivalent circuit

Rg

Rd

rd

gmVqVqin

CsRs

The following constraints apply.

There may be capacitor coupling which includes Cqd Cqs & Cds in the small signal.

Vp is the pinch off voltage, which is negative. Pinch-off is the condition that stops conducting ID but

still has leakage current IDSS.

VDS is the breakdown voltage where the transistor fails.

Keep the supply voltage, VDD, below the breakdown voltage, VDS.

Igs is the gate leakage current in nanoamperes. DI is the drain current.

For a JFET, the drain current is dependent on the leakage current and drain voltage.

2

D gs

D DSS

D

V VI I

V

Page 187: Dr. Robert A. Durham, PhD, PE Dr. Marcus O. Durham, PhD, PE … Nutshell Master.pdf · Dr. Robert A. Durham, PhD, PE . Dr. Marcus O. Durham, PhD, PE . A Quick Reference . The book

Page | 4-30

Example

A FET circuit with self bias is desired. The FET limits

and the supply consumption are provided.

Vp = -3 volts with current of 6ma

VBD = 30 volts

Required: Determine the bias for 10V Drain to

Source and channel current of 4ma.

Given:

ID = 4 mA

IDSS = 6 mA

|VD| = 3V

Solution:

2 2

34 6

3

0.55

D gs gs

D DSS

D

gs

V V VI I

V

V V

Assumptions:

Select Rg = 1MΩ

This makes voltage drop across Rg0 (Input resistance >10X source)

Calculations:

Write Kirchhoff current law @ FET

0 0D g S g

D S

I I I I

I I

For 30BDV , use 30DDV = 24 volts (80%)

Write KVL around loop for Bias

0D SDD R DS RV V V V

Substitute using Ohm’s law & values

24 10 0D D D SI R I R

Vgs is determined by ID & RS

RdId

Drain

SourceGate

RgRsVin

VDD

Page 188: Dr. Robert A. Durham, PhD, PE Dr. Marcus O. Durham, PhD, PE … Nutshell Master.pdf · Dr. Robert A. Durham, PhD, PE . Dr. Marcus O. Durham, PhD, PE . A Quick Reference . The book

Page | 4-31

Vgs = IDRS 3

0.55140

4 10

gs

S

D

VR

I

Substitute in KVL

3

14 0

14 4 10 ( ) 0

3500

D D D S

D S

D S

I R I R

R R

R R

Use RS = 140

3500 3.3kD SR R

Page 189: Dr. Robert A. Durham, PhD, PE Dr. Marcus O. Durham, PhD, PE … Nutshell Master.pdf · Dr. Robert A. Durham, PhD, PE . Dr. Marcus O. Durham, PhD, PE . A Quick Reference . The book

Page | 4-32

Basic FET Models

The FET has several different models that are used for design purposes.

Current Source Model

rds

G

S

D

gmvgs

Voltage Source Model

rds

G

S

D

μvgs

Complete Circuit

Common source voltage amplifier

ri

vi

cc1

R1

R2

R3

Rd

Rs Cs

Cc2

RL

DC

Small Signal Equivalent ri

vi R3+(R1||R2) Rdrds RL+

vL

-

gmvgs

Page 190: Dr. Robert A. Durham, PhD, PE Dr. Marcus O. Durham, PhD, PE … Nutshell Master.pdf · Dr. Robert A. Durham, PhD, PE . Dr. Marcus O. Durham, PhD, PE . A Quick Reference . The book

Page | 4-33

The equations for the FET are the normal big three: input impedance, output impedance, and gain.

3 1 2( || )iZ R R R

||o d dsZ R r

1 3 1 2

1( || )

1 / ( || )v m L oA g R Z

r R R R

Voltage Gain

usually 1 3 1 2( || )r R R R

If RL<<Zo then

v m LA g R

Page 191: Dr. Robert A. Durham, PhD, PE Dr. Marcus O. Durham, PhD, PE … Nutshell Master.pdf · Dr. Robert A. Durham, PhD, PE . Dr. Marcus O. Durham, PhD, PE . A Quick Reference . The book

Page | 4-34

Source Follower (Common Drain) Amplifier

The FET source follower is similar to the BJT emitter follower configuration.

Complete Circuit

vi

cc R1

Rs1

Rs2

VDD

+

vs

-

ii

Small Signal analysis

vi

Rs=Rs1+Rs2

VDD

+

vs

-

ii

Small Signal Equivalent

rds

vi

μvgs

+

vs

- Rs=Rs1+Rs2

ii

G

S

D

Thevenin Equivalent Circuit

Zo

vi

Zi μvgs+

vs

-

Rsii

GS

D

Assumptions assist in solving for the coupling voltage.

1R

s gs o ds sv v i r v

The big three are then calculated.

1

1

dso

m

rZ

g

2

1 2

11

1 ( )

( 1)1 s

s s

g

i Ri R R

v RZ R

i

'1

vA

Page 192: Dr. Robert A. Durham, PhD, PE Dr. Marcus O. Durham, PhD, PE … Nutshell Master.pdf · Dr. Robert A. Durham, PhD, PE . Dr. Marcus O. Durham, PhD, PE . A Quick Reference . The book

Page | 4-35

4.6 State Space State space models can be used for a wide range of problems from controls to communications and

computer modeling. State space is a technique to represent differential equations when applied to the

LaPlace s-domain.

4.6.1 The 6-Minute Approach

Definitions

The state equation has the form ( )

( ) ( )dx t

Ax t Bu tdt

The output equation has the form ( ) ( )y t Cx t

The state variables can be defined as 1 2( ) ( ), ( ) / ( )x t x t dx t dt x t

A, B, and C are matrices while u is input, y is output, and x is the transfer variable.

The transfer function, Y(s)/U(s) can be broken into parts by ( ) ( )

( ) ( )

X s Y s

U s X s. Then Y(s)/X(s) is the

numerator of the transfer function and X(s)/U(s) is 1 over the denominator.

The s-operator is defined as the first derivative. ' /s x dx dt

S-domain to Differential

Convert the numerator Y(s)/X(s) to ( ) ( )* ( )Y s numerator X s .

Substitute to obtain y(t)= f(x(t)), using #6 definition.

( ) / ( ) 4

( ) ' 4

Y s X s s

y t x x

Develop an equation for the highest derivative of x to be the same power as the highest power of s

in the denominator.

2

1

3 6s s

''x

Replace the highest power with the input.

'' ...x u

Replace the remaining s-values using #6 definitions.

'' 3 ' 6x u x x

The result is two differential equations.

Page 193: Dr. Robert A. Durham, PhD, PE Dr. Marcus O. Durham, PhD, PE … Nutshell Master.pdf · Dr. Robert A. Durham, PhD, PE . Dr. Marcus O. Durham, PhD, PE . A Quick Reference . The book

Page | 4-36

( ) ' 4y t x x

'' 3 ' 6x u x x

Substitute 1 2( ) ( ) and ( ) / ( ) ( )x t x t d x d t x t into the matrix form.

'1 1

'22

1

2

0 1 0

6 3 1

4 1

x xu

xx

xy

x

Page 194: Dr. Robert A. Durham, PhD, PE Dr. Marcus O. Durham, PhD, PE … Nutshell Master.pdf · Dr. Robert A. Durham, PhD, PE . Dr. Marcus O. Durham, PhD, PE . A Quick Reference . The book

Page | 4-37

4.6.2 Description

State space is used to solve ordinary differential equations. The process uses op-amps to create

summers and integrators. Note, differentiation produces noise. The output is limited to maximum and

input is limited to minimum because of noise. The values are selected so the constant coefficient has a

range of 0.1 to 10.0. Because of the op-amp circuit, each operation is always negative or inverting

Multiplier Summer Integrator

KVi Vo

V1

Vn

Vo

V1V2

V3

Vn

vo

o iV KV and 0 1K 1 1( )o n nV a V a V 1 1

0( )

t

o n nV a v a v

Third order differential equation

2 1 0 ( ) 0x a x a x a x f t

1 11x

x x x

a2

a1

a0

1

f(t)

Page 195: Dr. Robert A. Durham, PhD, PE Dr. Marcus O. Durham, PhD, PE … Nutshell Master.pdf · Dr. Robert A. Durham, PhD, PE . Dr. Marcus O. Durham, PhD, PE . A Quick Reference . The book

Page | 4-38

State Space method

òS S ò òS

a2

a1

a0

- - -

f(t)x

Scaling and Sampling Time

Time – T=ht

h = scaling factor

T = simulation time

t = time between samples

3 2 2 1 0 ( ) 0Th x h a x ha x a x fh

Simply replace 3 by x h x etc.

Time scaling factor of 1

hreplaces 1 on the input of each integrator.

( ) ( )

( ) ( )( )

t kT

y t Fx t

y kT Fx kt

k = which sample

T = interval between samples

Magnitude scaling

Used to get full linear range from an op-amp.

X=kx

X = simulation variable

x = dependent variable

k = scaling factor

max

max

Xk

x

k is chosen and incorporated in each term so that it does not change the equation.

Page 196: Dr. Robert A. Durham, PhD, PE Dr. Marcus O. Durham, PhD, PE … Nutshell Master.pdf · Dr. Robert A. Durham, PhD, PE . Dr. Marcus O. Durham, PhD, PE . A Quick Reference . The book

Page | 4-39

3 0 02 2 1 1

3 2 1 0

( )0

m

m

k x a k x f f ta k x a k x

k x k k k f

Simplify equations but keep terms in parentheses together. This is scaled variable X.

Example

0.82 18 250 400 0tx x x e

Initial conditions (0) 35 (0) 10x x

Maximum values max max max35 250 2500x x x

Maximum simulation V = 10 volts

Solution:

Find scaling factors

max2

max

100.004

2500

vk

x

max

max1

max

100.04

250

vk

x

max0

max

100.2

35

vk

x

maxmax

100.025

400

vf

f

Make differential w/ unity coefficient on USB

Divide by coefficient (example = 2)

0.89 125 200 0tx x x e

Substitute scaling factors

0.8200 0.0250.004 9 0.04 125 0.2

00.004 0.04 0.2 0.025

tex x x

Make differential w/ unity coefficient on USB

0.80.9 0.04 2.5 0.2 0.8 0tx x x e

Multiply scaling factor time initial value to obtain scaled initial values

1

0

(0) (0.04)(10) 0.4

(0) (0.2)(35) 7

k x

k x

Page 197: Dr. Robert A. Durham, PhD, PE Dr. Marcus O. Durham, PhD, PE … Nutshell Master.pdf · Dr. Robert A. Durham, PhD, PE . Dr. Marcus O. Durham, PhD, PE . A Quick Reference . The book

Page | 4-40

Adders (summers) use round values (0.1, 1, 10)

coefficients are obtained by using multiplier k and

summer value

Example: 2.5(0.2x)

Adjust scale coefficients by integrator coefficient

Example:

Draw differential equation (from above)

100.004x 0.04x

5

0.9

0.25

0.8e-0.8t = force

0.2x

-0.4 7

0.25 10(0.2x)

summer

100.004x 0.04x

Page 198: Dr. Robert A. Durham, PhD, PE Dr. Marcus O. Durham, PhD, PE … Nutshell Master.pdf · Dr. Robert A. Durham, PhD, PE . Dr. Marcus O. Durham, PhD, PE . A Quick Reference . The book

Page | 4-41

Integrator Realization – Draw it Out

Locate input x(t) and output y(t) along a line

Place as many integrators as order (n) of system

Place a summer before the output and one before each integrator. Total number of summers = n+1,

or the maximum number of y terms

Use summer to bring signals together. Use connectors for departing signals

Draw forward, x-input signals. Use scalars for gain on each signal. Terminate the signal after as many

integrators as the order. Feedback will prove the differentiation.

Draw feedback, y-output signals. Use scalars for gain on the signal. Terminate the signal after as

many of the integrators as the order. This represents the differentiation.

Draw the main line. Remove all operators that are null. Use as many feed-forward and feedback taps

as necessary to complete the system. If the previous two steps are consistent, this will be one line.

Page 199: Dr. Robert A. Durham, PhD, PE Dr. Marcus O. Durham, PhD, PE … Nutshell Master.pdf · Dr. Robert A. Durham, PhD, PE . Dr. Marcus O. Durham, PhD, PE . A Quick Reference . The book

Page | 4-42

Chapter 4 Problems

Problem 4-1 (Old Style)

SITUATION:

The operational amplifier circuit shown below is connected to provide a variable output resistance

at terminals a-b.

-

+

VinRo

Ra

LOAD

a

b

REQUIREMENTS:

Find a Thevenin equivalent circuit as a function of the potentiometer setting α at terminals a-b.

Assume the op-amp to be ideal and that R is much larger than RA.

SOLUTION:

Circuit is a non-inverting amplifier (voltage on + terminal).

Redraw into standard form

Find Thevenin Equivalent voltage (VTH).

Open circuit terminals, leave sources active, calculate voltage across open terminals

( (1 ) )

TH ab

ab A

A

V v

v I R R

I R

abA

vI

R

-

+(1-a)R

RA Vin

aR

b Vo a

Page 200: Dr. Robert A. Durham, PhD, PE Dr. Marcus O. Durham, PhD, PE … Nutshell Master.pdf · Dr. Robert A. Durham, PhD, PE . Dr. Marcus O. Durham, PhD, PE . A Quick Reference . The book

Page | 4-43

( (1 ) )

( (1 ) )

( (1 ) )

in A A

abA

inab A

V I R R

vR R

R

Vv R R

R

Find Thevenin Equivalent Impedance (ZTH)

Short voltage sources, open current sources, then calculate series/parallel resistances.

Alternately, short circuit the terminals, leave sources active, then find ISC, TH

TH

SC

VZ

I

inSC

A

VI

R

( (1 ) )

( (1 ) )

in

ATHTH

SC in

A

A

A

RVR RV

RI V

R

RR

R R

Page 201: Dr. Robert A. Durham, PhD, PE Dr. Marcus O. Durham, PhD, PE … Nutshell Master.pdf · Dr. Robert A. Durham, PhD, PE . Dr. Marcus O. Durham, PhD, PE . A Quick Reference . The book

Page | 4-44

Problem 4-2 (Old Style)

A FET amplifier composed of three identical stages is shown in the Figure below. It may be assumed

that Cs adequately bypasses Rs. It may also be assumed that there is 40 pF of stray shunt capacitance per

stage. Determine a value of RD such that the mid-frequency amplification is 60dB if

32.5 10mg 8dr k 100gR k

Cb

Rg

Rd

CsRs

Cb

Rg

Rd

CsRs

Cb

Rg

Rd

CsRs

Cb

Rg

REQUIREMENTS:

Determine a value of RD such that the mid-frequency amplification is 60dB.

SOLUTION:

FET Model

ReId

Drain

SourceGate

RgRg CSVin

Cb

Rgout Cstray/stage

Page 202: Dr. Robert A. Durham, PhD, PE Dr. Marcus O. Durham, PhD, PE … Nutshell Master.pdf · Dr. Robert A. Durham, PhD, PE . Dr. Marcus O. Durham, PhD, PE . A Quick Reference . The book

Page | 4-45

Rg

VDD

Rd

rd

gmVgsVgin

CsRs

Rgout

Assumptions:

Gain of FET is across rd. If CS shunts RS, then we can simply connect rd to ground.

At the signal frequency, VDD is a short.

Once input and output stages are isolated using model, each can be solved individually.

Rg

Rd

rd

gmVq

Vqin

Cb

Rgout

a

Given:

32.5 10mg 8dr k 100gR k 40strayC pf shunt

Gain relates the output and input voltage.

20logo o

in in

V VGain A db

V V

For gain of 60db overall:

3

60 20log

60log 3

20

10

o

in

o

in

o

in

V

V

V

V

V

V

For 3 stage amplifier, gain for each stage is 10 (10*10*10=103)

Page 203: Dr. Robert A. Durham, PhD, PE Dr. Marcus O. Durham, PhD, PE … Nutshell Master.pdf · Dr. Robert A. Durham, PhD, PE . Dr. Marcus O. Durham, PhD, PE . A Quick Reference . The book

Page | 4-46

Circuit analysis:

For an AC signal, the Capacitor bC is a short, which means it conducts.

Calculate the nodal equation at “a”.

31 1 1( 0)( ) 2.5 10N out in

d D g

I V Vr R R

Use the gain relationship to find the resistance.

3

3

1 1 1

1 1 110 ( 2.5 10 )

8 100

18.7

1.15 10

outm

in d D g

D

D

vA g

v r R R

k R k

R k

Page 204: Dr. Robert A. Durham, PhD, PE Dr. Marcus O. Durham, PhD, PE … Nutshell Master.pdf · Dr. Robert A. Durham, PhD, PE . Dr. Marcus O. Durham, PhD, PE . A Quick Reference . The book

Page | 4-47

Problem 4-3

In the figure of Problem 4-2 above determine the high frequency cutoff for the entire amplifier if

32.5 10mg 8dr k 100gR k 8.7DR k

SOLUTION

Cutoff for each stage is reciprocal of the time constant. The capacitance is given as the stray:

1

2hi cutoff

sh sh

fC R

40shC stray pf

The resistors are in parallel.

1 1 1 1

4

SH d D g

SH

R r R R

R k

Calculate the frequency.

12 3

10.995

2 (40 10 )(4 10 )hi cutofff MHz

Cutoff for entire amplifier is a root containing the number of stages.

1

1

36 6

2 1

0.995 10 2 1 0.995 10 0.51 0.501

nhi all hi one one

hi all

f f ATF f

f MHz

Page 205: Dr. Robert A. Durham, PhD, PE Dr. Marcus O. Durham, PhD, PE … Nutshell Master.pdf · Dr. Robert A. Durham, PhD, PE . Dr. Marcus O. Durham, PhD, PE . A Quick Reference . The book

Page | 4-48

Problem 4-4

In the figure of Problem 4-2 above determine a value of Cb such that the low frequency cutoff for

the entire amplifier is not greater than 200Hz (indicate any further assumptions made) if

32.5 10mg 8dr k 100gR k 8.7DR k

SOLUTION:

Low frequency cutoff for each stage:

1

lo cutoff

series seriesC R ?seriesC

Calculate the equivalent resistance.

511.042 10

1 1series g

d D

R R

r R

The frequency range is reduced by multiple stages.

1

1

13

2 1

2 1

200 2 1 102

n

n

lo onelo all lo one lo all

ff f f

Hz Hz

Cut-off frequency is time constant dependent.

5

1 10.015

2 (102)(1.042 10 )series

lo series

C fR

Page 206: Dr. Robert A. Durham, PhD, PE Dr. Marcus O. Durham, PhD, PE … Nutshell Master.pdf · Dr. Robert A. Durham, PhD, PE . Dr. Marcus O. Durham, PhD, PE . A Quick Reference . The book

Page | 4-49

Problem 4-5

A voltage amplifier is shown in the figure below. Determine the high-frequency cutoff. Use

appropriate approximations.

' 20b br ' 500b er ' 0.05b ec equiv F 0.004mg

100kΩ

50kΩ

5kΩ

5kΩ Xc=0

Vo

a a’

SOLUTION:

The circuit is a common emitter BJT amp, draw equivalent hybrid small signal model

Rbb’

Recbe

gmVbe

B

E

C

Input Side

Calculated 1st

Output Side

Use input to

calculate

+

Vbe

-

Ignore R’s which are used for Bias.

Large C will pass anything.

Resistors rb’e & rb’b are in parallel on input.

20*500

2020 500

inr

The input circuit to the transistor is shown at the right.

20

cbe

B +

Vbe

-

Page 207: Dr. Robert A. Durham, PhD, PE Dr. Marcus O. Durham, PhD, PE … Nutshell Master.pdf · Dr. Robert A. Durham, PhD, PE . Dr. Marcus O. Durham, PhD, PE . A Quick Reference . The book

Page | 4-50

'

11

1 1b e in

sCV VsCRR

sC

The cutoff freq is inversely related to the time constant.

8

8

1 110 / sec

20 5 10c rad

RC

810

1592 2

f kHz

Page 208: Dr. Robert A. Durham, PhD, PE Dr. Marcus O. Durham, PhD, PE … Nutshell Master.pdf · Dr. Robert A. Durham, PhD, PE . Dr. Marcus O. Durham, PhD, PE . A Quick Reference . The book

Page | 4-51

Problem 4-6

In the voltage amplifier shown in Problem 4-6, a 10μH coil is put in place between the a and a'.

Determine the upper frequency cutoff with this coil in place.

' 20b br ' 500b er ' 0.05b ec equiv F 0.004mg

SOLUTION:

The circuit is a common emitter BJT amp, draw equivalent hybrid small signal model.

Rbb’

Recbe

gmVbe

B

E

C

Input Side

Calculated 1st

Output Side

Use input to

calculate

+

Vbe

-

Ignore R’s, they are used for Bias.

Large C – pass anything

Resistors rb’e & rb’b are in parallel on input.

20*50020

20 500inr

The input circuit of the transistor is shown at right.

Use the standard bandpass or resonance equation for a second order

system. The equation can be stated in terms of circuit elements or in terms of

frequency as noted in chapter 2.

2

/( )

1

s LY s

Rs s

L LC

2 2

0

/( )

s LY s

s s

From the admittance, calculate the voltage drop across the capacitor.

12

' 2 6 122

1 2 10

1 2 10 2 10

in in

b e

V VLCV

R s ss sL LC

20

cbe

B

E

+

Vbe

-

10-5H

Page 209: Dr. Robert A. Durham, PhD, PE Dr. Marcus O. Durham, PhD, PE … Nutshell Master.pdf · Dr. Robert A. Durham, PhD, PE . Dr. Marcus O. Durham, PhD, PE . A Quick Reference . The book

Page | 4-52

The denominator is the characteristic equation when the input is set to zero. Equate the frequency

form and the circuit element forms to find the frequency.

2 22 n ns s

2 12 61 2 10 1.414 10n nLC

6

0.7072 10

o nR QL Q

1

2n

R

Q L

The 3db cutoff frequency is 225.2n kHz

Page 210: Dr. Robert A. Durham, PhD, PE Dr. Marcus O. Durham, PhD, PE … Nutshell Master.pdf · Dr. Robert A. Durham, PhD, PE . Dr. Marcus O. Durham, PhD, PE . A Quick Reference . The book

Page | 4-53

Problem 4-7

The circuit shown in the figure below is a cascade connection of a FET and a junction transistor.

100ieh 10DR k 10dr k 100feh 5s cR R k 310 (FET)mg

You can neglect hoe, hre and all capacitance.

Determine the overall small signal amplification out

in

VV

for this cascode connection. A cascode is a

high frequency connection with an input as a common source ant the output is a common gate.

+

Vin

-

RD

RS

Rc

+

Vout

-

SOLUTION

Draw the small signal models and solve.

Rg

Rd

rd

gmVqVqin

Rs

hie

Rc

hfeib

Page 211: Dr. Robert A. Durham, PhD, PE Dr. Marcus O. Durham, PhD, PE … Nutshell Master.pdf · Dr. Robert A. Durham, PhD, PE . Dr. Marcus O. Durham, PhD, PE . A Quick Reference . The book

Page | 4-54

Simplify:

Since d ier h and D ieR h , in a parallel arrangement, you can drop both and leave only hie.

Determine the output voltage.

( )

(1 ( ))

o s s C C

m g fe m g s fe m g c

m g fe s c

v i R i R

g V h g V R h g V R

g V h R R

Assume the forward parameter is not a contribution.

For hfe>>1

( ( ))o m g fe s cv g v h R R

Next determine the input voltage so the gain relationship can be determined.

(1 )

(1 )

1

( )

1

g i m g fe s

in fe m g s

in fe m s g

ing

m fe s

fe m c sout

in m fe s

v v g V h R

V h g V R

V h g R V

VV

g h R

h g R RV

V g h R

Substitute values to obtain the gain.

3

3

( )

1

100 10 (5 5 )

1 10 100 5

2

fe m c soutv

in m fe s

h g R RVA

V g h R

k k

k

RSgmVg

hie

hfeib=-hfegmVg

vo

Rc

Page 212: Dr. Robert A. Durham, PhD, PE Dr. Marcus O. Durham, PhD, PE … Nutshell Master.pdf · Dr. Robert A. Durham, PhD, PE . Dr. Marcus O. Durham, PhD, PE . A Quick Reference . The book

Page | 5-1

Chapter 5 - Controls

5.1 Introduction

Controls are predominantly about using transfer functions to describe the response of a system to signal

inputs.

The transfer function is usually a two-port network. The most complex circuit is a second order, which

has been discussed in everything from circuits and power to electronics. By having multiple transfer

functions which result in multiplication, the system transfer function can become very long. Controls

analysis is the process of deciphering these complex functions into a physically realizable performance in

terms of real and angular components.

The standard waveforms for signals are the big three: dc or step, exponential, and sinusoidal.

Special cases of the exponential are also considered. These are impulse and ramp.

Page 213: Dr. Robert A. Durham, PhD, PE Dr. Marcus O. Durham, PhD, PE … Nutshell Master.pdf · Dr. Robert A. Durham, PhD, PE . Dr. Marcus O. Durham, PhD, PE . A Quick Reference . The book

Page | 5-2

5.2 Controls Basics

5.2.1 Introduction

Control basics set the profile. The profile includes manipulation of the one-line or block diagram, the

errors, and the overshoot. A characteristic equation is used for stability analysis. The roots of the

numerator are called zeroes while the roots of the denominator are called poles. The gain of the

characteristic equation can be manipulated to improve stability.

5.2.2 Block Diagrams

A block diagram is used as a one-line representation of the process. It consists of three components:

input, R; output, Y; and the system transfer function. For analog systems, the diagram is in the LaPlace

transformed or s-domain. Obviously, this can be transformed directly into the frequency domain.

s j

Transfer Function – Output / Input = F=Y/R

Forward Gain – 1 2G G G

Feedback Gain – H

Unity Feedback – H = 1

Error signal – E R HY

Output – Y GE

Open Loop Transfer – ( )

( )

K s zGH

s p

Closed Loop Transfer – 1

Y GF

R GH

Characteristic Equation – 1 0GH ; 1GH

Denominator – 0

Open Loop 1 1 180

Roots of Characteristic Equation are the eigenvalues

eigenvalues – poles of YR

- closed loop poles

If a system is stable, all roots are in the Left Half Plane

The system is marginally stable if 1 root at origin, or 1 complex conjugate on jω axis

For frequency questions - s j

S G2Gain

G1Plant

HFeedbac

k

EError

RInput

YOutput

+

-

Page 214: Dr. Robert A. Durham, PhD, PE Dr. Marcus O. Durham, PhD, PE … Nutshell Master.pdf · Dr. Robert A. Durham, PhD, PE . Dr. Marcus O. Durham, PhD, PE . A Quick Reference . The book

Page | 5-3

Low frequency gain - K (DC, ω=0)

00

( )

( )

Y K j zF

R j p

Bandwidth ( BW ) is at amplitude equal to 12

* low frequency gain

0

1

2BW

BW

Y YF

R R

- Solve for BW

Page 215: Dr. Robert A. Durham, PhD, PE Dr. Marcus O. Durham, PhD, PE … Nutshell Master.pdf · Dr. Robert A. Durham, PhD, PE . Dr. Marcus O. Durham, PhD, PE . A Quick Reference . The book

Page | 5-4

5.2.3 Steady State Errors

Steady state errors are the response of the system to inputs. A second order system will have three

error responses; position (t=0), velocity (t), and acceleration (t2). The inputs that cause these responses

are step (DC), ramp (constant slope), and parabolic (exponential).

Use unity feedback & transfer function

Use final value theorem

0 0

( )lim ( ) lim

1 ( )s s

sR sess sE s

G s

The signal must be stable – not oscillating.

Error constants are the results of stability.

Position Velocity Acceleration

constant 0

lim ( )ps

k G s

0

lim ( )vs

k S s

2

0lim ( )s

ka S s

Laplace ds

dt

22

2

ds

dt

Steady state calculation depends on the order of the input.

step ramp parabolic

Time ( ) 1r t t 212

t

s 1( )R ss

21

s 3

1s

ess 0

1

lim1 ( )s

ss

G s

2

0

1

lim1 ( )s

ss

G s

3

0

1

lim1 ( )s

ss

G s

0

1

1 lim ( )s

G s

0

1lim

( )s sG s

20

1lim

( )s s G s

1

1 pk

1

vk

1

ak

0 no erroress

no control, explodesess

0ess as add pure integrators (increase system type)

System type = power of s in denominator

Page 216: Dr. Robert A. Durham, PhD, PE Dr. Marcus O. Durham, PhD, PE … Nutshell Master.pdf · Dr. Robert A. Durham, PhD, PE . Dr. Marcus O. Durham, PhD, PE . A Quick Reference . The book

Page | 5-5

5.2.4 Time Response

The time response of a system is the familiar three component DC, exponential, sinusoid.

From this, the time domain positions can be calculated. The waveform chapter in the section RLC

System Response has additional relationships.

Percent overshoot exists in under-damped systems only.

For second order system

211 ( )

% 100( )

e yos

y

( ) 1 for normalized systemy

21(max) 1 for normalized systemy e

Rise time is the amount of time to rise from 10% to 90% of final value

Settling time is the amount of time necessary to reach and stay within a band around the final value

The damping frequency is a shift from the natural or resonant where the wave is dropping off.

2 2d n

( ) ( )% 100 max

( )

y t yos

y

max ( )y t

p

d

nt

( )y

Page 217: Dr. Robert A. Durham, PhD, PE Dr. Marcus O. Durham, PhD, PE … Nutshell Master.pdf · Dr. Robert A. Durham, PhD, PE . Dr. Marcus O. Durham, PhD, PE . A Quick Reference . The book

Page | 5-6

5.3 Routh-Hurwitz Criteria

5.3.1 Introduction

Controls problems are the result of multiplications which often yield functions that are higher than

second order. Consequently, traditional solutions become unworkable and alternative approaches are

necessary. Factoring is a possibility, but can be very tedious. The Routh-Hurwitz criteria were developed

to eliminate factoring. The coefficients of a function are used to find stability.

5.3.2 Rules

Determine the function to be analyzed for stability.

Put coefficients of the function in 1st two rows of array – as even/odd powers

Next row – use determinant of 1st column & column to right of location being evaluated

Complete the row (use -1 on determinants)

If have a zero as last element in row, simply move last element of previous row to present row

Interpretation

# of roots in right half plane = # sign changes in first column

Example

4 3 2( ) 6 13 12 4f s s s s s

Create Routh table.

4

3

22,1 2,2

11,1

00,1

1 13 4

6 12 0

11 4

9.8 0

4

s

s

s P P

s P

s P

2,1

1 13

6 121 11

6P 2,2

1 4

6 01 4

6P

6 12

11 41,1 1 9.8

11P

2,1

(1*12 6*13)1 11

6P

2,2

(1*0 6*4)1 4

6P

1,1

(6*4 11*12)1 9.81

11P

5.3.3 Routh-Hurwitz – Special Cases

Page 218: Dr. Robert A. Durham, PhD, PE Dr. Marcus O. Durham, PhD, PE … Nutshell Master.pdf · Dr. Robert A. Durham, PhD, PE . Dr. Marcus O. Durham, PhD, PE . A Quick Reference . The book

Page | 5-7

A zero in the first column of the array implies division by zero – not kosher.

Two cases

First Case – zero in first column of row & some non-zero elements in the row.

Replace the zero by a small number (ε) and proceed.

Second Case – all elements in row are zero – This occurs if roots are on jω axis or roots are symmetrical

on axis about origin – ( p root ).

Form auxiliary polynomial fa using coefficients of row before zero row.

Take derivative of auxiliary polynomial and use as substitute for zero row.

Derivative gives max/min when zero.

Example 1

Determine the stability of the function.

3 2( ) 2 2f s s s s

Develop the Routh array from the coefficients.

3

2

11,1

0

1 1

2 2

4 0

2 0

s

s

s p

s

1,1

1 1

2 21 0

2p - would give zero row for s1

Instead, make fa using row above

2

1,1

2 2

4

4

a

a

f s

dfs

ds

P

Interpretation – first column positive – no sign change

no poles in RH plane, - stable

Example 2

Page 219: Dr. Robert A. Durham, PhD, PE Dr. Marcus O. Durham, PhD, PE … Nutshell Master.pdf · Dr. Robert A. Durham, PhD, PE . Dr. Marcus O. Durham, PhD, PE . A Quick Reference . The book

Page | 5-8

Determine the value of k for stability.

ΣRInput

Y(s)Output

+

-

2

2

1

( 1)( 2 2)

k s

s s s

Closed Loop Characteristic Equation - 1 0GH

In this problem, H=1.

2

2

( 1)1 1 [1] 0

( 1)( 2 2)

k sGH

s s s

Expand to obtain the stability function.

RInput

Y(s)Output

1

G

GH

2 2

3 2

( 1)( 2 2) ( 1)

( 3) (4 2 ) ( 2)

f s s s s k s

s k s k s k

Create Routh table.

3

2

11,1

00,1

1 4 2

3 2

s k

s k k

s P

s P

To be stable, the first column must have no sign changes. In this case the signs remain positive.

2,1 0 3 0P k 3k

0,1 0 2 0P k 2k

2

1,1

2 3 100 0

3

k kP

k

Denominator will be positive if ( 3k ). For the coefficient to be positive, the numerator must be >0

22 3 10 0k k

Solve using quadratic equation.

0,1 2P k

2

1,1

1 4 2

3 2 2 3 10( 1)

3 3

k

k K k kP

k k

Page 220: Dr. Robert A. Durham, PhD, PE Dr. Marcus O. Durham, PhD, PE … Nutshell Master.pdf · Dr. Robert A. Durham, PhD, PE . Dr. Marcus O. Durham, PhD, PE . A Quick Reference . The book

Page | 5-9

( 3) 9 (4)( 2)(10)

2( 2)

3 89

4

3.11&1.61

3.11 1.61

k

k

Combine all the constraints.

2 1.61k

If k = limits, have poles on jω @ the limits

If k outside limits, have instability

Page 221: Dr. Robert A. Durham, PhD, PE Dr. Marcus O. Durham, PhD, PE … Nutshell Master.pdf · Dr. Robert A. Durham, PhD, PE . Dr. Marcus O. Durham, PhD, PE . A Quick Reference . The book

Page | 5-10

Example 3

For the problem above, what are the poles on the jω axis?

ΣRInput

Y(s)Output

+

-

2

2

1

( 1)( 2 2)

k s

s s s

Use Routh table with the k used to calculate the limits.

When k=-2 when k=1.61

3

2

1

0

1 8

1 0

8

0

s

s

s

s

3

2

1

0

1 1.68

4.61 3.61

0

3.61

s

s

s

s

If any rows are zero, develop an alternative function.

s0 row = zero row s1 row – zero row

8

@ 0

8( ) 0

0

a

a

f s

s j f

j

2

*

* 2

4.61 3.61

@ 0

4.61( ) 3.61 0

a

a

f s

s j f

j

The frequencies of the roots are available.

@ origin, 0s j 2 3.61

4.61j

3.61

4.61

0.7831

0.885s j j

Page 222: Dr. Robert A. Durham, PhD, PE Dr. Marcus O. Durham, PhD, PE … Nutshell Master.pdf · Dr. Robert A. Durham, PhD, PE . Dr. Marcus O. Durham, PhD, PE . A Quick Reference . The book

Page | 5-11

5.4 Root Locus

5.4.1 Introduction

Root Locus is the process of determining the location of all the roots to a feedback function. The

location of the roots will change as the value of the gain, k changes. By adjusting k, the stability can be

improved or degraded. The function that is investigated is the characteristic equation, which is the

denominator of the transfer function.

Closed Loop Transfer Function 1

Y G

R GH

Open Loop Transfer Function – GH

When H=1. the open loop is the plant, G

Characteristic Equation

1 0GH

Solving characteristic equation implies 1GH

1GH

,( 1,3,5,7)

(2 1),( 0, 1, 2)

GH m m

or

GH k k

Adjust the gain, k, so the open loop transfer is one. Substitute the actual function values for GH and set

the equation to one.

1 2

1 2

( )( )1

( )( )

k s z s zGH

s p s p

Calculate the system gain, K.

product of vector length of poles

product of vector length of zeroesK

2( 1)GH s zi s pi k

Page 223: Dr. Robert A. Durham, PhD, PE Dr. Marcus O. Durham, PhD, PE … Nutshell Master.pdf · Dr. Robert A. Durham, PhD, PE . Dr. Marcus O. Durham, PhD, PE . A Quick Reference . The book

Page | 5-12

5.4.2 Object of Root Locus

The object of the root locus process is to determine the movement of the roots as k changes. The

analysis goes through this development.

Make plot of open loop function for 0k to k .

See where the function is stable & unstable. Avoid or fix unstable points.

Make sketch of significant points to draw.

Use the six rules to aid in the construction.

Page 224: Dr. Robert A. Durham, PhD, PE Dr. Marcus O. Durham, PhD, PE … Nutshell Master.pdf · Dr. Robert A. Durham, PhD, PE . Dr. Marcus O. Durham, PhD, PE . A Quick Reference . The book

Page | 5-13

5.4.3 Rules for Root Locus Construction

Each rule provides a significant factor to assist in the sketch of the roots or eigenvalues. The sketch then

shows the stability as the roots change due to the gain, k being adjusted.

1) Starting and Ending Points Root Locus plots start ( 0k ) on the open-loop poles and

end ( k or k ) on the open-loop zeros

2) Root Locus Segments on the Real Axis Root loci occur on a particular segment of the real axis if and only if there are an odd number of total poles and zeros of the open-loop transfer function lying to right of that segment.

3) Imaginary Axis Intersections Use the Routh-Hurwitz criterion to determine j axis crossings of the root locus plots. Both the

gain k and the value of ω* may be found from the Routh table.

4) Asymptotes (for p z )

Root locus plots are asymptotic to straight lines with angles given by

(2 1)

( )A

k

p z

as s approaches infinity. These straight lines intersect at a point 1 on the real axis specified by:

1

poles of ( ) zeros of ( )GH s GH s

p z

Where p is the number of finite poles of GH(s) and z is the number of finites zeros of GH(s).

5) Angles of Departure and Arrival Assume a point s arbitrarily near the pole (for departure) or the zero (for arrival) and then apply

the fundamental angel relationship

angles of zeros of ( ) angles of poles of ( ) (2 1)GH s GH s k

6) Breakaway Points Breakaway points may be determined by expressing the characteristic equation for the gain k as a function of s and then solving for the breakaway points sB from

( )

0

Bs s

dk s

ds

Page 225: Dr. Robert A. Durham, PhD, PE Dr. Marcus O. Durham, PhD, PE … Nutshell Master.pdf · Dr. Robert A. Durham, PhD, PE . Dr. Marcus O. Durham, PhD, PE . A Quick Reference . The book

Page | 5-14

Example 1

Sketch the root locus plot for the system in the diagram.

ΣRInput

Y(s)Output

+

-

( 1)( 2)

( 4)

k s s

s s

Set up the open loop function.

( 1)( 2)

( 4)

k s sGH G

s s

Plot the locus from poles to zeroes.

X – poles – denominator

O – zeros – numerator

Use rule 1, 2

Stay on axis only when have odd # of p&z to right

Example 2

Plot the root locus from the plant and feedback functions.

( 2) ( 3)

( 10) ( 1)

k s sG H

s s

The location of the poles and zeroes are obvious. However, the path is somewhat more tedious.

Obviously the path needs to cross from the right to

the left plane. Use Routh with the characteristic

equation which contains the open loop transfer

function.

1 0GH

Substitute the values.

Set up the Routh table.

2 2

2

( 1)( 10) ( 2)( 3) 0

9 10 5 6 0

( 1) (5 9) (6 10) 0

s s k s s

s s ks ks k

k s k s k

k=0k=k=k=0

-1-2-4

k=0

-1-2-3 10

2

1

0

1 6 10

5 9

6 10

s k k

s k

s k

Page 226: Dr. Robert A. Durham, PhD, PE Dr. Marcus O. Durham, PhD, PE … Nutshell Master.pdf · Dr. Robert A. Durham, PhD, PE . Dr. Marcus O. Durham, PhD, PE . A Quick Reference . The book

Page | 5-15

Make s1=0 with 95

k .

Develop the auxiliary function fa from the row s2

2( 1) (6 10)af k s k

Determine the frequency for the crossing the axis, jω*

*( 1)( ) (6 10) 0k j k

A complex conjugate results in complementary frequencies. When 95

k , the root locus crosses the

vertical axis.

2

95

6 10

1 k

k

k

0.54

Make s0 =0 for 106

k

When 106

k , the root locus crosses jω @ 0 on its path to the right half plane, RHP.

The root locus moves from one pole in the LHP, crosses into the RHP at 0 . It collides with a pole in

the RHP. The path then comes back @ 95

k in complex conjugate pairs. It stays close to the origin.

The track should be circle, since make polynomial is s2. If the roots are s3 or higher, the path becomes an

ellipse.

k=0

-1-2-3

Example 3

Develop the root locus for an open loop transfer function.

Page 227: Dr. Robert A. Durham, PhD, PE Dr. Marcus O. Durham, PhD, PE … Nutshell Master.pdf · Dr. Robert A. Durham, PhD, PE . Dr. Marcus O. Durham, PhD, PE . A Quick Reference . The book

Page | 5-16

2( 2)( 2 2)

kGH

s s s

The second order poles can be factored.

( 2)( 1 )( 1 )

kGH

s s j s j

Observations can be made about the expected results.

Not all poles are real. jω

p>z asymptotes

complex angle of departure

Follow the rules to develop all the plotting parameters.

Rule 2 – on real axis

Rule 3 – not all real, use Routh to find k & jω

21 ( 2)( 2 2) 0GH s s s k

3 21 4 6 (4 ) 0GH s s s k

Make the Routh table.

3

2

1

0

1 6

4 4

20

4

4

s

s k

ks

s k

1,0

1 6

4 4 20

4 4

k kP

Determine the frequency to make a row zero.

1 0 @ 20s k

24 (4 )af s k

* 24( ) 24 0j

6 @ 20k

Rule 4 - p z 3 0 3 3zeros @ p z

Page 228: Dr. Robert A. Durham, PhD, PE Dr. Marcus O. Durham, PhD, PE … Nutshell Master.pdf · Dr. Robert A. Durham, PhD, PE . Dr. Marcus O. Durham, PhD, PE . A Quick Reference . The book

Page | 5-17

, ,3 33 0

A

Intersection

( 2) ( 1 ) ( 1 ) 0 4

3 0 3i

poles zeros j j

p z

Rule 5 - The complex conjugates need to 180 to all other poles & zeros

Pole @ 1 902

s j

zeros of GH - poles of GH=(2 )k

from -1-j = 902

from -2= 454

All these are s from poles, from local pole = x

(0) ( ) (2 )2 4x k

4x

, the θ for conjugate is complex

Plot the calculated points and draw the root locus from poles to zeroes.

-1- j

-4/3-2

-1+ j

Page 229: Dr. Robert A. Durham, PhD, PE Dr. Marcus O. Durham, PhD, PE … Nutshell Master.pdf · Dr. Robert A. Durham, PhD, PE . Dr. Marcus O. Durham, PhD, PE . A Quick Reference . The book

Page | 5-18

5.5 Frequency Response Plots

5.5.1 Introduction

Frequency plots indicate the performance of a system over a range of frequencies. The plots can be

linear, semi-log (Bode), log-log, or polar plots (Nyquist). All these tools have the same information, and

are used to determine relative stability.

In contrast, the transition from the RHP using Routh- Hurwitz gives absolute stability in a limited system

without time shift.

A comparison of the costs and benefits of each response plot assists in selecting the one most

appropriate for a particular problem.

Linear – direct, but cumbersome math

Bode – easiest. Use log10 of open loop frequency response

Magnitude values in decibels (db) are plotted on the vertical

1020log ( )G j

Perform for each pole & zero @ K, then add the sketches.

Phase angle is plotted under the magnitude plot

Start w/ ( )G j in standard form

2

2

1( ) ( )

( ) (1 2 ( )

n

n n

j

ms j j

kG j G s

j

Plot the asymptotes.

These differ from actual values by 3db @ limits (ωn is corner frequency)

Slope of asymptotes = 20*m on the db/decade

Slope is negative for poles and positive for zeros.

m is the exponent power of s associated with the root.

Determine the phase angle @ the values.

1tann

90 *m for all ω

At the root, cross the 45o angle on the way from 0o two decades before the root in route to 90o two

decades after the root.

Page 230: Dr. Robert A. Durham, PhD, PE Dr. Marcus O. Durham, PhD, PE … Nutshell Master.pdf · Dr. Robert A. Durham, PhD, PE . Dr. Marcus O. Durham, PhD, PE . A Quick Reference . The book

Page | 5-19

5.5.2 Bode Plots – Basic rules

The basic rules for Bode are quite short. The examples are better illustrations than a wordy explanation.

1) Magnitude – make break @ ωn, plot asymptote 20*m db/decade, where m = power of s.

2) Phase angle – make 45 break @ ωn plot from 2 decades before to 2 after the root.

3) Break down for poles and up for zeros

4) Plot 2 decades before to two decades after

5) Σ plots to get effective

Example 1

Make a frequency plot of a plant function.

1000( 1)

( )( 2)( 10)( 20)

s j

sG s

s s s

Convert the form to a magnitude and phase angle.

2

2 2 2 2 2 2

1000 1( )

2 10 20G j

1 1 1 1( ) tan tan tan tan1 2 10 20

G j

The above equations are used for the linear plots.

Example 2

Make a frequency plot of a plant function above.

Convert the function to standard form for Bode & log-log.

1000(1)1

(2)(10)(20) 1( )

1 1 12 10 20

j

G jj j j

1000(1)

2.5(2)(10)(20)

K

Page 231: Dr. Robert A. Durham, PhD, PE Dr. Marcus O. Durham, PhD, PE … Nutshell Master.pdf · Dr. Robert A. Durham, PhD, PE . Dr. Marcus O. Durham, PhD, PE . A Quick Reference . The book

Page | 5-20

20 log(2.5) = 8, DC portion – normalized

Magnitude is plotted for each root.

Break up + 20db for zero @ 1

Break down -20db for poles @ 2, 10, 20

@ DC, response is8 0 , start phase diagram at 0°

Phase angle is plotted for each root.

+45˚ break points for zero @ ω=1

-45˚ break points for poles @ ω = 2, 10, 20

Combine the magnitude with 3 db smoothing @ corners (1,2,10,20)

1

rad/sec0 dB

-40 dB

20 dB

rad/sec

0 dB

-180°

1 100 10k

+45°

10 10000.1

10 1000

1000100

Combine the angles with a 5.7o smoothing at each break.

Page 232: Dr. Robert A. Durham, PhD, PE Dr. Marcus O. Durham, PhD, PE … Nutshell Master.pdf · Dr. Robert A. Durham, PhD, PE . Dr. Marcus O. Durham, PhD, PE . A Quick Reference . The book

Page | 5-21

Page 233: Dr. Robert A. Durham, PhD, PE Dr. Marcus O. Durham, PhD, PE … Nutshell Master.pdf · Dr. Robert A. Durham, PhD, PE . Dr. Marcus O. Durham, PhD, PE . A Quick Reference . The book

Page | 5-22

5.5.3 – Phase Margin and Gain Margin

The phase margin and gain margin of a system are indicators of the relative stability of the system. The

phase margin basis of the system is the absolute value of the difference between the actual phase angle

and -180° when the gain is 0dB.

phasemargin ( 180 ) @0dB

The gain margin basis of the system is the absolute value of the difference between the actual gain and

0dB when the phase angle is -180°.

gain margin ( ) 0G j dB @-180°

Example:

For the system response shown below, determine the phase margin and the gain margin.

0.01

rad/sec0 dB

-80 dB

40 dB

0.1 100.001 1001

-160 dB

0.01

rad/sec0 dB

-90°

0.1 100.001 1001

-180°

At approx 0.01 rad/sec, the gain crosses 0dB. At that frequency, the phase angle is -90°

0.01

phasemargin 90 ( 180 ) 90

At approx 10 rad/sec. the phase angle crosses -180°. At that frequency, the gain is approx -70dB.

10

gain margin 70 (0 ) 70dB dB dB

Page 234: Dr. Robert A. Durham, PhD, PE Dr. Marcus O. Durham, PhD, PE … Nutshell Master.pdf · Dr. Robert A. Durham, PhD, PE . Dr. Marcus O. Durham, PhD, PE . A Quick Reference . The book

Page | 5-23

5.5.3 Polar - Nyquist Plot.

Re ( )G j vs Im ( )G j

Use the Bode Plot from the previous example to determine points on the Plot. Only a couple of points

are needed to determine the overall curve

The following steps outline the process.

@ ω=0

(8/20)10 2.5

0

G

G

Plot the point (2.5,0)

At the point that the phase = 0 real axis, called ωpc (phase crossing)

@= 0

(12/20)10 3.98

0

G

G

Plot the point (3.98,0)

Page 235: Dr. Robert A. Durham, PhD, PE Dr. Marcus O. Durham, PhD, PE … Nutshell Master.pdf · Dr. Robert A. Durham, PhD, PE . Dr. Marcus O. Durham, PhD, PE . A Quick Reference . The book

Page | 5-24

At the point when 1G , this is called ωgc , arc from origin

@ 1G

01 10

125

G

Plot the point (-0.574,-0.819)

If zeros of |G| in left HP, called minimum phase

Plot the mirror image for complex conjugate pairs

Page 236: Dr. Robert A. Durham, PhD, PE Dr. Marcus O. Durham, PhD, PE … Nutshell Master.pdf · Dr. Robert A. Durham, PhD, PE . Dr. Marcus O. Durham, PhD, PE . A Quick Reference . The book

Page | 5-25

5.6 Analog Filters

Filters are classified according to the order of the transfer function, and according to function. The most

common functions are low-pass, high-pass, band-pass and band-reject filters. Other types are phase-

lead and phase-lag. A chart of filter information, including transfer functions, system response and

circuits are shown below for some of the most common type filters.

First Order Low-Pass Filters

Frequency Response

ωc0 ω

H

1( ) (0)

2cH j H

RC Voltage Source (Thevenin)

v1

R1

C R2

+

v2

-

2

1 1

1( )

1

eq

eq

RvH s

v R sR C

1 2

1 2

eq

R RR

R R

1c

eqR C

RL

v1

R1

R2

+

v2

-

L

2 2

1

1( )

1eqeq

v RH s

Lv R sR

1 2eqR R R eq

c

R

L

RC Current Source (Norton)

R1 R2C

i2

i1

2

1 1

1( )

1

eq

eq

RiH s

i R sR C

1 2

1 2

eq

R RR

R R

1c

eqR C

Page 237: Dr. Robert A. Durham, PhD, PE Dr. Marcus O. Durham, PhD, PE … Nutshell Master.pdf · Dr. Robert A. Durham, PhD, PE . Dr. Marcus O. Durham, PhD, PE . A Quick Reference . The book

Page | 5-26

First Order High-Pass Filters

Frequency Response

1( ) ( )

2cH j H j

ωc0 ω

H

RC

v1

R1

R2

+

v2

-

C

2 2

1

( )1

eq

eq eq

sR Cv RH s

v R sR C

1 2eqR R R 1

c

eqR C

RL Voltage Source (Thevenin)

v1

R1

R2

+

v2

-L

2

1 1

( )1

eq eq

eq

LsR Rv

H sLv R s

R

1 2

1 2

eq

R RR

R R

eq

c

R

L

RL Current Source (Thevenin)

R1 R2L

i2

i1

2

1 1

1( )

1

eq

p

RvH s

v R sR C

1 2

1 2

eq

R RR

R R

1c

pR C

Page 238: Dr. Robert A. Durham, PhD, PE Dr. Marcus O. Durham, PhD, PE … Nutshell Master.pdf · Dr. Robert A. Durham, PhD, PE . Dr. Marcus O. Durham, PhD, PE . A Quick Reference . The book

Page | 5-27

Band-Pass Filters

Frequency Response

ωo0 ω

H

ωL ωU

0

1( ) ( ) ( )

2L UH j H j H j

3-dB Bandwidth = u LBW

Series

v1

R1

R2

+

v2

-

CL

2

21 1

1( )

1eq

v sH s

Rv R Cs s

L LC

1 2eqR R R 1

0LC

2 2

1 2

( )o

eq

R RH j

R R R

eqRBW

L

Parallel

v1

R1

R2

+

v2

-

L C

2

21 1

1( )

1eq

v sH s

v R C s sR CLC

1 2

1 2

eq

R RR

R R

10

LC

2

1 2 1

( )eq

o

RRH j

R R R

1

eq

BWR C

Page 239: Dr. Robert A. Durham, PhD, PE Dr. Marcus O. Durham, PhD, PE … Nutshell Master.pdf · Dr. Robert A. Durham, PhD, PE . Dr. Marcus O. Durham, PhD, PE . A Quick Reference . The book

Page | 5-28

Band-Reject Filters

Frequency Response

ωo0 ω

H

ωL ωU

1( ) ( ) 1 (0)

2L UH j H j H

3-dB Bandwidth = u LBW

Series

v1

R1

R2

+

v2

-

C

L

2

2 2

21

1

( )1

eqeq

sv R LCH ssv R s

R C LC

1 2eqR R R 1

0LC

2 2

1 2

(0)eq

R RH

R R R

eqRBW

L

Parallel

v1

R1

R2

+

v2

-

L

C

2

2

21 1

1

( )1

eq

eq

sRv LCH sRv R

s sL LC

1 2

1 2

eq

R RR

R R

10

LC

2

1 2 1

(0)eqRR

HR R R

1

eq

BWR C

Page 240: Dr. Robert A. Durham, PhD, PE Dr. Marcus O. Durham, PhD, PE … Nutshell Master.pdf · Dr. Robert A. Durham, PhD, PE . Dr. Marcus O. Durham, PhD, PE . A Quick Reference . The book

Page | 5-29

Phase-Lead Filters Phase-Lag Filters

ω1 ωm ω2

ω

log|H(jω)|

ωm0 ω

φm

ω1 ω2

ω1 ωm ω2

ω

log|H(jω

)|

ωm0 ω

φm

ω1 ω2

v1

R2

+

v2

-C

R1

2 1 1 1

1 1 22

11

( )1 1

eq

eq

sRv sR C

H ssv R sR C

1 2

1 2

eq

R RR

R R

1

1

1

R C 2

1

eqR C

1 2m max ( )m mH j

2 1 2 1

1 2

arctan arctan arctan2

m

m

1

1 2

(0)eqR

HR

1

2

mH j

( ) 1H j

v1

R1

R2

+

v2

-

C

2 2 2

1 1

1 1( )

1 1

eq

eq eq

Rv sR C sR CH s

v R sR C sR C

1 2eqR R R 1

1

eqR C 2

2

1

R C

1 2m min ( )m mH j

1 2 1 2

2 1

arctan arctan arctan2

m

m

(0) 1H 1

2

mH j

2 1

2

( )eq

RH j

R

Page 241: Dr. Robert A. Durham, PhD, PE Dr. Marcus O. Durham, PhD, PE … Nutshell Master.pdf · Dr. Robert A. Durham, PhD, PE . Dr. Marcus O. Durham, PhD, PE . A Quick Reference . The book

Page | 5-30

Chapter 5 Problems

Problem 5-1

The circuit shown below is constructed

from an ideal op-amp (infinite gain

bandwidth product)

RC1 -

+

R

C2

Vout

Vin

The phase shift between input and output signals, as used

below, does not include the inversion (180°) from the

operational amplifier circuit.

20R k

1 100C pF

2 0.01C F

5.1.1 Which of the following is most

correct?

A) This is not a high-pass active filter.

B) The input circuit time constant is 2 μS.

C) The feedback circuit time constant is

0.2mS.

D) The phase of the output lags the phase

of

the input signal

5.1.2 The poles and the zeros of the transfer function

are approximately

A) One pole at 5 kHz and one zero at 500 kHz

B) Two zeros, one at 800 Hz and one at 80 kHz

C) Two poles, one at 5 krad/sec and one at

500krad/sec

D) A paired pole and zero at 55 krad/sec

E) One pole at 800 Hz and one zero at

500krad/sec

5.1.3 The dc response (dB) is

A) 20

B) 0

C) -20

D) -30

E) -40

5.1.4 The response (dB) at 10 MHz is most nearly

A) 10

B) 0

C) -10

D) -20

E) -40

Page 242: Dr. Robert A. Durham, PhD, PE Dr. Marcus O. Durham, PhD, PE … Nutshell Master.pdf · Dr. Robert A. Durham, PhD, PE . Dr. Marcus O. Durham, PhD, PE . A Quick Reference . The book

Page | 5-31

5.1.5 The frequency at which the response

is -20dB is most nearly:

A) 0 Hz

B) 800 HZ

C) 500 kHz

D) 50 krad/sec

E) 80 krad/sec

5.1.6 The maximum phase shift between input and

output signals occurs under approximately which of the

following conditions?

A) When the response is minimum

B) When the response is equal to the square root of the

ratio of the capacitances

C) When the response is equal to the ratio of the time

constants

D) When the response is maximum

E) At a frequency that is twice that of a pole

5.1.7 The phase shift is approximately half

of its maximum value under which of the

following conditions?

A) When the response is -20 dB

B) When the response is equal to the

square root of the ratio of the capacitances

C) When the response is equal to the ratio

of the time constants

D) At the frequencies of a zero and a pole

E) At a frequency that is half that of a zero

5.1.8 The phase shift is 90° under which of the following

conditions?

A) When the response is -10dB

B) When the response is equal to the square root of the

ratio of the capacitances

C) When the response is equal to the ratio of the time

constants

D) At the frequencies of 800Hz and 80kHz

E) Never

5.1.9 The phase shift is approximately 5%

of its maximum value under which of the

following conditions?

A) When the response is -10dB

B) When the response is equal to the ratio

of the capacitances

C) When the response is a maximum

D) At the frequencies of 500 rad/sec and 5

Mrad/sec

E) At a frequency that is half that of a zero

5.1.10 Which of the following is true of this circuit?

A) Its sinusoidal response is maximum at the pole and

minimum at its zero

B) Its response is maximum when it is equal to the ratio

of the square of the capacitances

C) Its response is maximum at dc and minimum at very

high frequency

D) Its response increases with increasing frequency from

its value at dc

E) Its output phase leads the input signal phase

Page 243: Dr. Robert A. Durham, PhD, PE Dr. Marcus O. Durham, PhD, PE … Nutshell Master.pdf · Dr. Robert A. Durham, PhD, PE . Dr. Marcus O. Durham, PhD, PE . A Quick Reference . The book

Page | 5-32

SOLUTION:

Calculate the gain function.

2 1

2

1

11

2

2

5

3

11

11

1

1

5 100.01

5 10

out

in

RRsCV RsC

V RsCRRsC

SRCC

CS

RC

s

s

Convert to standard form in terms of frequency.

5) 5 5

3

3 3

1 10.01(5 10 5 10 5 10

15 10

1 15 10 5 10

out

in

j j

V

j jV

Observe poles and zeros.

Pole at 5x103 rad/sec = 796Hz – Break Down

Zero at 5x105 rad/sec = 7.96kHz – Break Up

Determine the DC valued from K.

1; 20log 0K K , DC response is 0dB

@ DC, the gain is 1 1 180 . The starting point for the Phase diagram is 180°.

Plot gain and phase angle asymptotes.

Page 244: Dr. Robert A. Durham, PhD, PE Dr. Marcus O. Durham, PhD, PE … Nutshell Master.pdf · Dr. Robert A. Durham, PhD, PE . Dr. Marcus O. Durham, PhD, PE . A Quick Reference . The book

Page | 5-33

500 5x103 5x104 5x105 5x106

500 5x103 5x104 5x105 5x106

79.6 796 7.96k 79.6k 796k

rad/sec

Hz

0 dB

-40 dB

180°

90˚

-

3dB

-37dB

-5.7°

101.4°

5.1.1

This is not a high-pass active filter. The correct answer is A)

5.1.2

Zero at 5x105 rad/sec (79.6kHz)

Pole at 5x103 rad/sec (796 Hz)

The correct answer is E)

5.1.3

By inspection @ DC, the gain is 0 dB. The correct answer is B)

5.1.4

At 10 MHz, by inspection, the gain is -40dB. The correct answer is E)

5.1.5

By inspection, -20dB occurs at 50krad/sec (7.96kHz). The correct answer is D)

5.1.6

By inspection, max phase shift occurs at -20dB (0.1), which is 1 2C C . The correct answer is B)

Page 245: Dr. Robert A. Durham, PhD, PE Dr. Marcus O. Durham, PhD, PE … Nutshell Master.pdf · Dr. Robert A. Durham, PhD, PE . Dr. Marcus O. Durham, PhD, PE . A Quick Reference . The book

Page | 5-34

5.1.7

By inspection, the phase shift is approx. half its max value at the zero and the pole. The correct answer is

D).

5.1.8

The phase shift never reaches 90°. The correct answer is E)

5.1.9

The phase shift is approx 5% of its maximum value at one decade above the zero frequency (5x106

krad/sec) and one decade below the pole frequency (500 rad/sec). The correct answer is D)

5.1.10

By inspection, the response is maximum at DC and minimum at very high frequencies. The correct

answer is C).

Page 246: Dr. Robert A. Durham, PhD, PE Dr. Marcus O. Durham, PhD, PE … Nutshell Master.pdf · Dr. Robert A. Durham, PhD, PE . Dr. Marcus O. Durham, PhD, PE . A Quick Reference . The book

Page | 5-35

Problem 5-2

The response to a unit step test is shown in the

figure below.

0.58152

0.5

0.9069

Data List

Peak value of unit step response – 0.58152

Steady-state value of unit step response – 0.5

Time to peak value of unit step response – 0.9069

sec

5.2.1 Data from the data list that are necessary

and sufficient to determine the percent

overshoot are:

A) 1 only

B) 2 only

C) 1 and 2 only

D) 1 and 3 only

E) 2 and 3 only

5.2.3 The expression for the time to peak in terms

of damping ratio and natural frequency of

oscillation is

A) 21

p

n

t

B) 1

p

n

t

C) 1

p

n

t

D) 2

1

1p

n

t

E) p

n

t

5.2.2 The percent overshoot is most nearly

A) 0%

B) 0.163%

C) 16.3%

D) 58.2%

5.2.4 The dc gain is most nearly

A) 0

B) 0.5

C) 1

D) 2

Page 247: Dr. Robert A. Durham, PhD, PE Dr. Marcus O. Durham, PhD, PE … Nutshell Master.pdf · Dr. Robert A. Durham, PhD, PE . Dr. Marcus O. Durham, PhD, PE . A Quick Reference . The book

Page | 5-36

E) 100% E) 8

5.2.5 Data from the data list that are necessary

and sufficient to determine the damping ratio are

A) 1 only

B) 2 only

C) 1 and 2 only

D) 1 and 3 only

E) 2 and 3 only

5.2.7 The natural frequency of oscillation n is

most nearly

A) 1

B) 1.414

C) 2

D) 4

E) 16

5.2.6 The damping ratio is most nearly

A) 0

B) 0.5

C) 1

D) 2

E) 8

5.2.8 The transfer function of the open loop system

is

A) 2

8

8 16s s

B) 2

0.5

8 16s s

C) 2

1

4 16s s

D) 2

8

4 16s s

E) 2

0.5

4 16s s

Page 248: Dr. Robert A. Durham, PhD, PE Dr. Marcus O. Durham, PhD, PE … Nutshell Master.pdf · Dr. Robert A. Durham, PhD, PE . Dr. Marcus O. Durham, PhD, PE . A Quick Reference . The book

Page | 5-37

SOLUTION

5.2.1 max ( ) ( )

% overshoot = 100( )

y t y

y

Need 1 and 2 only. The correct answer is C)

5.2.2 0.58152 0.5

% overshoot = 100 16.304%0.5

. The correct answer is C)

5.2.3 21

p

n

t

. The correct answer is A)

5.2.4 The dc gain is the steady state value divided by the input (unit step), or 0.5. The correct answer is

B

5.2.5 The damping ratio may be found from the equation 21

% overshoot = 100e

. The data needed

are 1 and 2 only. The correct answer is C)

5.2.6 210.58152 0.5

% overshoot = 100 16.304% 1000.5

e

.

21

2

2 2 2

2 2

2

1006.135

16.304

ln 6.135 1.8141

3.29(1 )

( 3.29) 3.29

0.25; 0.5

e

The correct answer is B)

5.2.7

2

2

1

1

4.0 / sec0.9069 0.75

p

n

n

p

n

t

t

rad

The correct answer is D)

5.2.8 Form of the equation is 2

2 22

n

n n

Ks

2

160.5

4 16s s

The correct answer is D)

Page 249: Dr. Robert A. Durham, PhD, PE Dr. Marcus O. Durham, PhD, PE … Nutshell Master.pdf · Dr. Robert A. Durham, PhD, PE . Dr. Marcus O. Durham, PhD, PE . A Quick Reference . The book

Page | 5-38

Problem 5-3

A control system has an open loop response of 2

8

4 16s s . A tachometer feedback system is installed

in the control system in order to make the system critically damped.

5.3.1 The rate gain needed for the tachometer

feedback to achieve a critically damped system is

most nearly

A) 0.125

B) 0.5

C) 1

D) 2

E) 4

5.3.2 The location of the critically damped poles is

most nearly

A) -16

B) -4

C) 0

D) 4

E) 16

SOLUTION

The block diagram, with the tachometer (rate) feedback is shown

5.3.1

The transfer function is developed in the circuit

element form.

.

The frequency form of the second order bandpass or resonance equation is 2

2 22

n

n n

Ks

.

Set the denominator characteristic equation in circuit form to the equivalent frequency form. Then

calculate the frequency variables.

.

For critical damping, ζ=1. Therefore, the gain b can be calculated.

The correct answer is B)

2 2

( ) 8 8

( ) 1 4 16 8 (4 8 ) 16T T

Y s G

X s GH s s K s s K s

2 16 4n n

4 8 2T nK

4 8 2 2(1)(4) 0.5T n TK K

X(s)Input

Y(s)Output

+

-2

8

4 16s s

TK s

S

Page 250: Dr. Robert A. Durham, PhD, PE Dr. Marcus O. Durham, PhD, PE … Nutshell Master.pdf · Dr. Robert A. Durham, PhD, PE . Dr. Marcus O. Durham, PhD, PE . A Quick Reference . The book

Page | 5-39

5.3.2

The critical damped transfer function is

2 2

8 8

8 16 4s s s

. Therefore the poles are -4, -4.

The correct answer is B)

Page 251: Dr. Robert A. Durham, PhD, PE Dr. Marcus O. Durham, PhD, PE … Nutshell Master.pdf · Dr. Robert A. Durham, PhD, PE . Dr. Marcus O. Durham, PhD, PE . A Quick Reference . The book

Page | 5-40

Problem 5.4

A control system is described by the block diagram

X(s)Input

Y(s)Output

+

-2

4

6 13

s

s s

Σ ( )cG s

controller

Data List:

Gs(s)=2

4

6 13

s

s s

Gc(s)=( )

( 1)

K s z

s s

Possible values of z = -3 or -5

5.4.1 The open loop transfer function is

A) K

B) ( )

( 1)

K s z

s s

C) 2

( )( 4)

( 1)( 6 13)

K s z s

s s s s

D) 2( 1)( 6 13)s s s s

E) 2

4

6 13

s

s s

5.4.4 The angles the asymptotes of the root loci

makes with the real axis of the s plane are

A) ±0°

B) ±90°

C) ±180°

D) ±270°

E) ±360°

5.4.2 The poles of the open loop transfer function

are:

A) z, -4 only

B) 0, -1 only

C) -3±j2 only

D) 0,-1, -3±j2 only

E) z, 0, -1, -3±j2, -4

5.4.5 Data from the data list that are necessary

and sufficient to determine the intersection of

the root loci with the real axis of the s plane are

A)1 only

B) 1 and 2 only

C) 1 and 3 only

D) 2 and 3 only

E) 1, 2, and 3

Page 252: Dr. Robert A. Durham, PhD, PE Dr. Marcus O. Durham, PhD, PE … Nutshell Master.pdf · Dr. Robert A. Durham, PhD, PE . Dr. Marcus O. Durham, PhD, PE . A Quick Reference . The book

Page | 5-41

5.4.3 The zeros of the open loop transfer function

are:

A) z, -4 only

B) 0, -1 only

C) -3±j2 only

D) 0,-1, -3±j2 only

E) z, 0, -1, -3±j2, -4

5.4.6 The intersections of the root loci with the

real axis of the s plane, if 3 5z and are

A) 0 and 0

B) 0 and 1

C) 0 and +1 and -1

D) 1 and 1

E) -3 and -5

5.4.7 The characteristic function if z = -3 is

A) 4 3 27 19 13s s s s

B) 4 3 27 20 17s s s s

C) 4 3 27 20 20 12s s s s

D) 4 3 27 34 118 180s s s s

E) 4 3 27 (19 ) (13 7 ) 12s s K s K s K

5.4.9 What is the first term in the third row

Routhian array if z=-3?

A) (156+12K2)

7

B) 120

7

C) 19 K

D) 13 K

E) 12K

5.4.8 Which of the following values of z provide for

a stable system for any value of K?

A) -3 only

B) any z>-5

C) -3 and -5 only

D) any z>-3

E) Neither -3 nor -5

5.4.10 For with values of K is the system stable is

z = -3

A) All K>0

B) All K<0

C) All large K

D) All small K

E) All K<15

Page 253: Dr. Robert A. Durham, PhD, PE Dr. Marcus O. Durham, PhD, PE … Nutshell Master.pdf · Dr. Robert A. Durham, PhD, PE . Dr. Marcus O. Durham, PhD, PE . A Quick Reference . The book

Page | 5-42

SOLUTION:

5.4.1

The open-loop transfer function ( )

( ) ( )( )

c s

Y sG s G s

X s

2

( )( 4)

( 1)( 6 13)

K s z s

s s s s

.

The correct answer is C)

5.4.2

The poles of the open lop transfer function are the roots of 2 6 13 1s s s s

Roots are 0, -1, 3 9 13 =0, -1, 3 2j

The correct answer is D)

5.4.3

The zeros of the open loop transfer function are the roots of ( )( 4)K s z s . Roots are z= -4

The correct answer is A)

5.4.4

Asymptotes – angle = (180 360 )

p z

k

n n

0, , p zk n n ; 4; 2p zn n . Therefore the angle = 90

The correct answer is B)

5.45

- c

p z

poles zeros

n n

H(s), Gc(s) and the value of z are needed.

The correct answer is E)

5.4.6

Page 254: Dr. Robert A. Durham, PhD, PE Dr. Marcus O. Durham, PhD, PE … Nutshell Master.pdf · Dr. Robert A. Durham, PhD, PE . Dr. Marcus O. Durham, PhD, PE . A Quick Reference . The book

Page | 5-43

0 ( 1) ( 3 2) ( 4) - 1.5 0.5

4 2c

p z

j zpoles zerosz

n n

For 3, 0cz . For 5, 1cz

The correct answer is B)

5.4.7

2

2

( 1)( 6 13) ( )( 4)1 ( )

( 1)( 6 13)c

s s s s K s z sG s H

s s s s

The Characteristic equation = 4 3 27 (19 ) 13 (4 ) 4s s K s K z s Kz

For 4 3 23, 7 19 13 (4 ) 12z s s K s K z s K

The correct answer is E)

5.4.8

0 ( 1) ( 3 2) ( 4) - 1.5 0.5

4 2c

p z

j zpoles zerosz

n n

For 0c , there are poles in the right half of the s-plane – unstable.

The correct answer is D)

5.4.9

For z=-3, the Routh array is

2,1

19 (13 7 ) 120

7 7

K KP

2,2 12P K

2,2

1,1

2,1

13 7 713 7 4.9 13 2.1

K PP K K K

P

0,1 2,2 12P P K

The correct answer is B)

5.4.10

4

3

22,1 2,2

11,1

00,1

1 19 12

7 13 7 0

s K K

s K

s P P

s P

s P

Page 255: Dr. Robert A. Durham, PhD, PE Dr. Marcus O. Durham, PhD, PE … Nutshell Master.pdf · Dr. Robert A. Durham, PhD, PE . Dr. Marcus O. Durham, PhD, PE . A Quick Reference . The book

Page | 5-44

From the Routh array above, 2,1 0P , 1,1 0 6.19P for K , 0,1 0 0P for K .

Therefore, the function is stable for all 0K .

The correct answer is A)

Page 256: Dr. Robert A. Durham, PhD, PE Dr. Marcus O. Durham, PhD, PE … Nutshell Master.pdf · Dr. Robert A. Durham, PhD, PE . Dr. Marcus O. Durham, PhD, PE . A Quick Reference . The book

Page | 6-1

Chapter 6 Digital

6.1 Introduction Digital systems are the language of computers and microprocessor hardware. A basic understanding of

the principals associated with digital systems are presented.

6.2 Binary (Digital) Systems

6.2.1 Number Systems

Decimal numbers

When using numbers, we usually write only the coefficient and let the position indication the power of

10. The coefficient range = 0-9 for base 10

Binary numbers

The coefficient range = 0-1 for base 2

3 2 1 02 101010 1 2 0 2 1 2 0 2 10

Octal numbers

Octals are simply binary numbers combined into groups of 3 ( base 8=23)

388 2 1| 001 12

The coefficient range for octal numbers is 0-7

3 2 1 0

107392 7 10 3 10 9 10 2 10

Page 257: Dr. Robert A. Durham, PhD, PE Dr. Marcus O. Durham, PhD, PE … Nutshell Master.pdf · Dr. Robert A. Durham, PhD, PE . Dr. Marcus O. Durham, PhD, PE . A Quick Reference . The book

Page | 6-2

Hexadecimal numbers

Hexadecimal numbers are binary numbers that are combined into groups of 4 (base 16 = 24)

41616 2 0 |1010 0A

The range for hexadecimal numbers is 0-F.

Hex # Decimal Equiv Hex # Decimal Equiv

0 1 8 9

1 2 9 10

2 3 A 11

3 4 B 12

4 5 C 13

5 6 D 14

6 7 E 15

7 8 F 16

Page 258: Dr. Robert A. Durham, PhD, PE Dr. Marcus O. Durham, PhD, PE … Nutshell Master.pdf · Dr. Robert A. Durham, PhD, PE . Dr. Marcus O. Durham, PhD, PE . A Quick Reference . The book

Page | 6-3

Arithmetic

Arithmetic rules for all bases have the same basic set of rules.

Addition

111

101

1100

Subtraction

111

101

010

Multiplication

111

101

111

000

111

100011

Decimal to Binary Equivalence

0 0000 5 0101

1 0001 6 01101

2 0010 7 0111

3 0011 8 1000

4 0100 9 1001

Page 259: Dr. Robert A. Durham, PhD, PE Dr. Marcus O. Durham, PhD, PE … Nutshell Master.pdf · Dr. Robert A. Durham, PhD, PE . Dr. Marcus O. Durham, PhD, PE . A Quick Reference . The book

Page | 6-4

6.2.2 Binary System Wiring

Equivalent Names

5V = 1 = high = True = on = closed

0V = 0 = low = False = off = open

Resistors

Pull-up – from power source –

2.2KΩ

Current limiting – in series with load

330Ω

Switch

When switch = open, output = 1 (True)

When switch = closed, output = 0 (False)

Called an inverting switch

2.2kΩ

output

5 Volts

switch

Transistor

Simply a digital switch with an electrical

input (inverter)

When base switch = off, output = 1

(True)

When base switch = on, output = 0

(False)

2.2kΩ

output

330Ω

5V

Page 260: Dr. Robert A. Durham, PhD, PE Dr. Marcus O. Durham, PhD, PE … Nutshell Master.pdf · Dr. Robert A. Durham, PhD, PE . Dr. Marcus O. Durham, PhD, PE . A Quick Reference . The book

Page | 6-5

Output – LED

Generally connected to ground

Use current limiting resistor

LED Long leg is the ground

This is a common cathode arrangement.

A common anode would have inverted

diodes

330Ω

5 Volts

330Ω

TTL

Standard Arrangement

5V Power, VCC, on upper right pin

Ground on lower left pin

Notch at top

a11

a22

3a3

4a4

b1

b2

b3

b4

14

13

12

11

a15

a26

7a3

b1

b2

b3

10

9

8

5 volts

Problem:

Wire a switch circuit with an LED connected to the output pin.

Page 261: Dr. Robert A. Durham, PhD, PE Dr. Marcus O. Durham, PhD, PE … Nutshell Master.pdf · Dr. Robert A. Durham, PhD, PE . Dr. Marcus O. Durham, PhD, PE . A Quick Reference . The book

Page | 6-6

6.2.3 The Huntington Postulates

Required entities to define an algebra: A defined group of coefficients (R, N, Q, Z [0,1]), Real Natural, Complex Binary

A defined group of operators and a table which defines how each operator works.

A defined group of axioms or postulates (unproven theorems) from which new theorems,

lemmas, corollaries, and propositions may be constructed.

[0,1] = B (coefficients)

OR AND NOT

For all x.y elements of group B: for each operator

P1 (a) x y B (b) *x y B (closure) only 1 or 0

P2 (a) 0x x (add Identity, I=0) (b) *1x x (multiply Identity, I=1) P3 (a) x y y x (b) * *x y y x (commutative)

P4 (a) *( ) ( * ) ( * )x y z x y x z (b) ( * ) ( )*( )x y z x y x z

(distributive)

P5 (a) 1x x (b) * 0x x (complement, unique) P6 (a) B contains at least 2 distinct elements

Useful theorems: listed in pairs that have correspondence – duality – every expression is valid if operator and identity elements are changed. To find the dual, exchange + to * and 1 to 0.

T1 (a) x x x (b) *x x x T2 (a) 1 1x (b) *0 0x

T3 (a) ( )x x (involution)

T4 (a) ( ) ( )x y z x y z (b) ( ) ( )x yz xy z (associative)

DeMorgan’s Theorem – complement function by interchanging AND & OR operators and complementing each literal

T5 (a) ( )x y x y (b) ( * )x y x y

T6 (a) x xy x (b) ( )x x y x (absorption)

T7 (a) x x y x y (b) ( )x x y xy

Operator Precedence

Parentheses () Not AND OR

Complement – take dual and complement each literal

+ 0 1

0 0 1 1 1 1

* 0 1

0 0 0 1 0 1

~

0 1 1 0

Page 262: Dr. Robert A. Durham, PhD, PE Dr. Marcus O. Durham, PhD, PE … Nutshell Master.pdf · Dr. Robert A. Durham, PhD, PE . Dr. Marcus O. Durham, PhD, PE . A Quick Reference . The book

Page | 6-7

6.2.4 Basic Digital Gates

NAND

X

Y

Z

F=(xyz)’

And Invert

XOR

F=x’+y’+z’ = (xyz)’xyz

Invert Or

NOR

X

Y

Z

F=(x+y+z)’

Or Invert

XAND

F=x’y’z’=(x+y+z)’x

yz

Invert And

A one input gate acts like an inverter

To use NAND requires Sum of Products (SOP) form

To use NOR requires Product of Sums (POS) form

Page 263: Dr. Robert A. Durham, PhD, PE Dr. Marcus O. Durham, PhD, PE … Nutshell Master.pdf · Dr. Robert A. Durham, PhD, PE . Dr. Marcus O. Durham, PhD, PE . A Quick Reference . The book

Page | 6-8

6.3 Karnaugh Maps

Definitions: Literal a variable of the problem Don’t Care Output values for which no value is necessary (situations that never occur, or if

they do, can be ignored SOP Abbreviation for Sum of Product POS Abbreviation for Product of Sum Reflected Code A binary code which has the property that the codes following every 2n-1 codes

are the reflection of the first 2n-1 codes in all bits except the most significant. The MSB of the first 2n-1

is a zero, and the MSB of the next 2n-1 codes is a 1.

Example Binary Code Reflected Code

00 00

01 01

10 11

11 10

PURPOSE:

A method of realizing a function in either of the two standard forms such that the number and

complexity of terms in the function is minimal.

Construction:

Draw a square or rectangular figure allowing 2n squares (for a problem with n variables).

The table should be drawn so that there are 2x rows and 2y columns where x y n .

Label the rows and columns with reflected code from left to right and top to bottom.

Fill in each square of the map with its corresponding truth table function value.

Simplification:

Circle the largest contiguous binary (2w, w n ) group of 1’s for SOP (0’s for POS) which is rectangular or square

Consider all the edges of the map to be physically adjacent.

Each circled block of 1’s (0’s) corresponds to one SOP (POS) term. The term is extracted by observing which literals do not change for the block. These can be complemented to create the POS terms using 0’s. The literals are then AND’ed (OR’ed) together to realize the function.

Finally, each term, which corresponds to each rectangular block, is OR’ed (AND’ed) together to realize the function

Each 1 (0) in the map must be circled at least once to realize the function. (Sometimes there is more than one way to do it).

Additional Rule for Don’t Cares: It is not required that don’t cares be circled, but treat them as 1 (0) if it will help with minimization.

Page 264: Dr. Robert A. Durham, PhD, PE Dr. Marcus O. Durham, PhD, PE … Nutshell Master.pdf · Dr. Robert A. Durham, PhD, PE . Dr. Marcus O. Durham, PhD, PE . A Quick Reference . The book

Page | 6-9

6.3.1 Construction / Simplification of Karnaugh Maps:

Example 1:

( )( ')

AB A B

A B A B

Example 2:

A B

A B

Example 3:

( )( ' )

A B AB

A B A B

Example 4:

( )( )

A B C BC AC

A C A BC

Eliminate Races - Overlap

(SOP)F A C BC AB AC

A B F

0 0 0 0 1 1 1 0 1 1 1 0

B A

0 1

0 0 1

1 1 0

A B F

0 0 1 0 1 1 1 0 1 1 1 0

B A

0 1

0 1 1

1 1 0

A B F

0 0 1 0 1 0 1 0 0 1 1 1

B A

0 1

0 1 0

1 0 1

A B C F

0 0 0 0 0 0 1 1 0 1 0 0 0 1 1 1 1 0 0 1 1 0 1 0 1 1 0 1 1 1 1 1

C AB

0 1

00 0 1

01 0 1

11 1 1

10 1 0

C AB

0 1

00 0 1

01 0 1

11 1 1

10 1 0

Page 265: Dr. Robert A. Durham, PhD, PE Dr. Marcus O. Durham, PhD, PE … Nutshell Master.pdf · Dr. Robert A. Durham, PhD, PE . Dr. Marcus O. Durham, PhD, PE . A Quick Reference . The book

Page | 6-10

Example 5:

(0,1,3,4,6,7) (2,5)F (0,1,3,4,6,7)F (2,5)F

( )( ) ( )

(SOP)

F A B C A B C POS

F A B BC AC A C

(SOP)F B C A C AB

Eliminate Races

(SOP)F A B A C AB BC AC

C AB

0 1

00 1 1

01 0 1

11 1 1

10 1 0

A B C F

0 0 0 1 0 0 1 1 0 1 0 0 0 1 1 1 1 0 0 1 1 0 1 0 1 1 0 1 1 1 1 1

C AB

0 1

00 1 1

01 0 1

11 1 1

10 1 0

C AB

0 1

00 1 1

01 0 1

11 1 1

10 1 0

C AB

0 1

00 1 1

01 0 1

11 1 1

10 1 0

C AB

0 1

00 1 1

01 0 1

11 1 1

10 1 0

Page 266: Dr. Robert A. Durham, PhD, PE Dr. Marcus O. Durham, PhD, PE … Nutshell Master.pdf · Dr. Robert A. Durham, PhD, PE . Dr. Marcus O. Durham, PhD, PE . A Quick Reference . The book

Page | 6-11

6.4 Design w/ Multiplexer

Multiplexers are a mixed bag

Designers try to minimize gates good

Do clever things good

Results are not logical bad

Hard to troubleshoot bad

Use multiplexer to show sequence of states

3 level pattern

#1 – multiplexers that determine the next state of registers

#2 – Register that holds present binary state

#3 – Decoder that provides a separate output for each control state

Put “Select” line across top of Karnaugh map

With “C” Select With “A” Select

'

F A B AC BC

F A B AC B C

F A B B C AC

F A B B C AC

0

1

Select C

I AB AB AB

I A B

0

1

Select A

I B

I C

C AB

0 1

00 0 0

01 1 1

11 1 0

10 1 0

A BC

0 1

00 0 1

01 0 0

11 1 0

10 1 1

Page 267: Dr. Robert A. Durham, PhD, PE Dr. Marcus O. Durham, PhD, PE … Nutshell Master.pdf · Dr. Robert A. Durham, PhD, PE . Dr. Marcus O. Durham, PhD, PE . A Quick Reference . The book

Page | 6-12

Three different implementations of the same function.

A valve header requires the following logic to control its actions:

(2,3,4,6)

(0,1,5,7)

F

For 8:1 Multiplexer

I0

I1

I2

I3

I4

I5

I6

I7

EN

S2 S1 S0

Z

+5V

+5V+5V

+5V

A B C

F

constants

For 4:1 Multiplexer

Only need

“A” inputs

I0

I1

I2

I3

EN

S2 S1

Z

A

+5V

A’

B C

B & C are select

Do not cross areas of constant BC

A B C F

0 0 0 0 0 0 1 0 0 1 0 1 0 1 1 1 1 0 0 1 1 0 1 0 1 1 0 1 1 1 1 0

BC A

00 01 11 10

0 0 0 0 1 1 3 1 2

1 1 4 0 5 0 7 1 6

BC A

00 01 11 10

0 0 0 0 1 1 3 1 2

1 1 4 0 5 0 7 1 6

E 0 1 3 2

Page 268: Dr. Robert A. Durham, PhD, PE Dr. Marcus O. Durham, PhD, PE … Nutshell Master.pdf · Dr. Robert A. Durham, PhD, PE . Dr. Marcus O. Durham, PhD, PE . A Quick Reference . The book

Page | 6-13

For 2:1 Multiplexer

I0

I1

EN

S0

Z

ABA’B

C

F

Need A&B

Input

C=0 C=1 C is select

Do not cross areas of constant C Use I0 for C = 0 Area Use I1 for C = 1 Area

For 2:1 Multiplexer – different select

I0

I1

EN

S0

ZB

C’

A

F

Need B&C

Input

A=0 A=1 A is select

Do not cross areas of constant A Use I0 for A = 0 Area Use I1 for A = 1 Area

Choice of inputs to select lines determines how to partition K-Map

Select line – ABC, then each value is fixed, nothing changes

Select line = BC, then A value changes

Select line = C, then AB values change

BC A

00 01 11 10

0 0 0 0 1 1 3 1 2

1 1 4 0 5 0 7 1 6

B A

0 1

0 0 1

1 1 1

B A

0 1

0 0 1

1 0 0

BC A

00 01 11 10

0 0 0 0 1 1 3 1 2

1 1 4 0 5 0 7 1 6

C B

0 1

0 0 0

1 1 1

C B

0 1

0 1 0

1 1 0

Page 269: Dr. Robert A. Durham, PhD, PE Dr. Marcus O. Durham, PhD, PE … Nutshell Master.pdf · Dr. Robert A. Durham, PhD, PE . Dr. Marcus O. Durham, PhD, PE . A Quick Reference . The book

Page | 6-14

6.5 Decoder

Function

SOP = (minterms) (OR of minterms)

POS = Π(maxterms) (AND of maxterms)

Each output of decoder is a minterm

For function in SOP

Use Active Hi output for OR gate

If use Active Lo output – must use invert-OR = NAND

For function in POS

Active Lo output – use AND gate

If use Active Hi output – must use invert-AND = NOR

Decoder Example

SOP Form

SOP uses 1’s

F= (1,2,4,6,7)

S0

A

S1

B

CS2

O0

O1

O2

O3

O4

O5

O6

O7

EN

0

Active Hi

Sum (OR)

F

S0

A

S1

B

CS2

O0

O1

O2

O3

O4

O5

O6

O7

EN

0

Active Lo

NAND

F

POS Form

POS Uses 0’s

F= Π(0,3,5)

S0

A

S1

B

CS2

O0

O1

O2

O3

O4

O5

O6

O7

EN

0

Active Lo

AND

F

S0

A

S1

B

CS2

O0

O1

O2

O3

O4

O5

O6

O7

EN

0

Active Hi

NOR

F

Page 270: Dr. Robert A. Durham, PhD, PE Dr. Marcus O. Durham, PhD, PE … Nutshell Master.pdf · Dr. Robert A. Durham, PhD, PE . Dr. Marcus O. Durham, PhD, PE . A Quick Reference . The book

Page | 6-15

Note that the Functions are equivalent - F= (1,2,4,6,7) = Π(0,3,5)

6.5.1 74156 DECODER

The 74156 is a dual 2:4 decoder

The 2 input lines can be decoded into 4 output lines

An Active low device – when the input is selected low, the decoded output line will be low

Two separate 2:4 decoders may be individual, or connected as a 3:8

a11

O0

O1

O2

O3

7

6

5

4S1S0

0 0

Y4Y5Y6Y7

a11

O0

O1

O2

O3

9

10

11

12

S0

0 0

S1 Y0Y1Y2Y3

B(3)A(13)

Strobe W’ (2)

Data C (1)

Strobe W’ (14)

Data C’ (15)

The select lines are common

WCBA is sequence for complete address

W’ have write to output, this is enable line

C-C’ tie together – data

BA Address

RAM

Write – write line=0, then data (1 or 0) on data line is clocked into decoder

A1A0 – Same address connected to both decoders

DATA (C-C’)– Inverted on decoder b, direct on Decoder a, therefore output of

both each side of a FF will force the FF to change state

Page 271: Dr. Robert A. Durham, PhD, PE Dr. Marcus O. Durham, PhD, PE … Nutshell Master.pdf · Dr. Robert A. Durham, PhD, PE . Dr. Marcus O. Durham, PhD, PE . A Quick Reference . The book

Page | 6-16

6.6 Flip Flops / Latch When S=0, R=1, Qt+1 = 0 When S=1, R=0, Tt+1 = 1

QR Flip Flop Function – 1tQ S R Q

S R Q Qt+1

0 0 0 0 0 0 1 1 0 1 0 0 0 1 1 0 1 0 0 1 1 0 1 1 1 1 0 indeterminate

1 1 1 indeterminate

Q SR

0 1

00 0 1

01 0 0

11 IND IND

10 1 1

Page 272: Dr. Robert A. Durham, PhD, PE Dr. Marcus O. Durham, PhD, PE … Nutshell Master.pdf · Dr. Robert A. Durham, PhD, PE . Dr. Marcus O. Durham, PhD, PE . A Quick Reference . The book

Page | 6-17

6.6.1 General – Flip Flop Types

Characteristic Table Excitation Table

Characteristic Equation

Symbol

Circuit

Asynchronous (RS Flip Flop)

S R t dtQ

't dtQ

tQ t dtQ

S R

0 0 tQ tQ 0 0 0 -

0 1 1 0 0 1 1 0 1 0 0 1 1 0 0 1 1 1 0 0 1 1 - 0

0

t dt tQ S R Q

SR

Q

QSET

CLR

S

R

R

S

Q

Q`

S R

S R

0 0 1 1 0 0 1 - 0 1 1 0 0 1 0 1 1 0 0 1 1 0 1 0 1 1

1 1 - 1

1

t dt tQ S RQ

S R

Q

QSET

CLR

S

R

R

S

Q

Q`

Synchronous

S R

S R

0 0 0 0 0 - 0 1 1 0 0 1 1 0 1 0 0 1 1 0 0 1 1 1 0 0 1 1 - 0

1

0

t tQ S R Q

SR

Q

QSET

CLR

S

R

cp

S

R

cp

Q

QSET

CLR

S

R

J K

J K

0 0

0 0 0 -

0 1 0 1 0 1 1 - 1 0 1 0 1 0 - 1 1 1

1 1 - 0

1t t tQ JQ K Q

J

Q

Q

K

SET

CLR

cp

K

J

cp

Q

QSET

CLR

S

R

D

D

0 0 1 0 0 0 1 1 0 0 1 1 1 0 0 1 1 1

1tQ D

Q

QSET

CLR

D

cp

D

cp

Q

QSET

CLR

S

R

T

T

0

0 0 0

1

0 1 1

1 0 1 1 1 0

1t t tQ TQ T Q

T Q

Q`

Q

QSET

CLR

S

RT

cp

t dtQ 't dtQ tQ t dtQ

tQ tQ

1tQ 1'tQ tQ 1tQ

1tQ 1'tQ tQ 1tQ

tQ tQ

tQ tQ

1tQ 1'tQ tQ 1tQ

1tQ 1'tQ tQ 1tQ

tQ tQ

tQ tQ

Page 273: Dr. Robert A. Durham, PhD, PE Dr. Marcus O. Durham, PhD, PE … Nutshell Master.pdf · Dr. Robert A. Durham, PhD, PE . Dr. Marcus O. Durham, PhD, PE . A Quick Reference . The book

Page | 6-18

6.6.2 Counter

Sequential circuit that goes through a prescribed sequence of states on application of input pulse

Binary counter – counter that follows binary sequence\

n-bit counter has n flip flops

only input is count pulse, clock is implied

Output = present state of FF

Counter completely specified by a list of the count sequence – is the sequence of binary states it

undergoes

Page 274: Dr. Robert A. Durham, PhD, PE Dr. Marcus O. Durham, PhD, PE … Nutshell Master.pdf · Dr. Robert A. Durham, PhD, PE . Dr. Marcus O. Durham, PhD, PE . A Quick Reference . The book

Page | 6-19

Counter Design

Description – Specifications

Draw state diagram a. Used to show

progressive states of sequence events

b. Same as state table c. Present state in circle d. Vector shows next state e. Vector information

o First character indicates input that causes change of state. Use 1 for clock

o 2nd number(s) show output for present state. Only happens when arriving at next state, not during transition

Make State Table a. This contains the same information as the state diagram, and is

interchangeable.

Assign state variables a. Assign each desired output to a FF if have no conditional outputs

(no logic) b. Assign each state adjacent so that you do not change more than

one state variable for a single change in input – mirror code

c. n=# state variables → 2 #n states

i. Example 4 states = 2n → n = 2 state variables, which is the number of FFs

d. Identify 2 FFs as Q1Q2, QAQB, AB, PQ, depending on author e. Sometimes do not use abc, but numbers

Make transition table a. This is simply present state – next state with

state assignments b. Output = present state is use FF for output

Determine FF type & assign letters to each a. Make column for input to each FF b. From excitation table for the FF, what is the

input to the FF required to make transition from PS to NS. i.e. if PS Q1=1, NS Q1=0

c. FF input T1 must be 1 to cause toggle D FF = Next State

PS NS w=0

NS w=1

Out w=0

Out w=1

a a b 0 0 b c b 0 0 c d c 0 0 d d a 0 0

PS Assignment

a 00 AB b 01 AB c 11 AB d 10 AB

Inputs

w

Present State Q1Q2

Next State Q1Q2

Outputs

Z1

FF Input T1T2

0 00(a) 00 0 00 1 00(a) 01 0 01 0 01(b) 11 0 10 1 01(b) 01 0 00 0 11(c) 10 0 01 1 11(c) 11 1 00 0 10(d) 10 0 00 1 10(d) 00 0 10

a

/ 0w

b

/ 0w

c

d

/ 0w

/ 0w

/ 0w

/ 0w/1w

Use a 1

for

clock

Page 275: Dr. Robert A. Durham, PhD, PE Dr. Marcus O. Durham, PhD, PE … Nutshell Master.pdf · Dr. Robert A. Durham, PhD, PE . Dr. Marcus O. Durham, PhD, PE . A Quick Reference . The book

Page | 6-20

Draw K-map for FF input @ this present state. Where is the T FF input required from excitation table to get next state. d. Use PS & Input to yield FF input

1 1 2 1 2T wQ Q wQ Q 2 1 2 1 2T wQ Q wQ Q 1 2z wQ Q

Start w/ specification – end w/ Boolean expression. Can draw diagram and wire circuit from this.

Q1Q2

W 00 01 11 10

0 0 1 0 0

1 0 0 0 1

Q1Q2

W 00 01 11 10

0 0 0 1 0

1 1 0 0 0

Page 276: Dr. Robert A. Durham, PhD, PE Dr. Marcus O. Durham, PhD, PE … Nutshell Master.pdf · Dr. Robert A. Durham, PhD, PE . Dr. Marcus O. Durham, PhD, PE . A Quick Reference . The book

Page | 6-21

Example: 3 bit binary counter

State diagram

state Table (T Flip Flops)

Output = Present State = Q2Q1Q0

000

001

010

011

100

101

110

111

Karnaugh Maps

T2 T1 T0

2 1 0T Q Q 1 0T Q 0 1T

Draw Circuit

PS NS T2 T1 T0

000 001 0 0 1 001 010 0 1 1 010 011 0 0 1 011 100 1 1 1 100 101 0 0 1 101 110 0 1 1 110 111 0 0 1 111 000 1 1 1 000

Q1Q0

Q2 00 01 11 10

0 0 0 1 0

1 0 0 1 0

Q1Q0

Q2 00 01 11 10

0 0 1 1 0

1 0 1 1 0

Q1Q0

Q2 00 01 11 10

0 1 1 1 1

1 1 1 1 1

Page 277: Dr. Robert A. Durham, PhD, PE Dr. Marcus O. Durham, PhD, PE … Nutshell Master.pdf · Dr. Robert A. Durham, PhD, PE . Dr. Marcus O. Durham, PhD, PE . A Quick Reference . The book

Page | 6-22

T0 Q0

T1 Q1

T2 Q2

5V

Page 278: Dr. Robert A. Durham, PhD, PE Dr. Marcus O. Durham, PhD, PE … Nutshell Master.pdf · Dr. Robert A. Durham, PhD, PE . Dr. Marcus O. Durham, PhD, PE . A Quick Reference . The book

Page | 6-23

6.6.3 Sequence Detector (Random)

Sequence consists of n bits

number of states: Mealy = n, Moore = n+1

Make transition table with added columns of current sequence and desired sequence. Table columns are current sequence, desired sequence, input, present state, next state, output, FF input

Label current state & desired state

Write desired sequence under column for all rows

Place sequence up to this point under correct sequence

Draw line through as many as useable of current sequence to next state in desired sequence.

Same number of states must be crossed in both

Output = 1 when all states are crossed for Mealy. For Moore, add one more state when all states are crossed

Else, go to 7.

Note: it is easier to make input

Page 279: Dr. Robert A. Durham, PhD, PE Dr. Marcus O. Durham, PhD, PE … Nutshell Master.pdf · Dr. Robert A. Durham, PhD, PE . Dr. Marcus O. Durham, PhD, PE . A Quick Reference . The book

Page | 7-1

Chapter 7 - Economics – Time Value of Money

7.1 Introduction Does money have the same value today that it did 10 years ago? Would you rather have your money

now or later? If you can get it sooner, would you be willing to give a discount on the amount. If you are

willing to get it later, how much interest do you need?

In the process of creating projects, it is necessary to balance time, money, and quality. Other chapters

look at the time and quality issues. This section discusses the relative value of money. The study of the

value of money is called economics.

Economics is often divided into two segments: macro and micro. Macroeconomics deals with large-scale

money manipulation that is often controlled by government. It attempts to look at an economy as a

whole. Microeconomics is local scale money that is controlled by a project, corporation, or other entity.

This chapter will focus on project economics.

Economics has been called the science of allocating scare resources. That definition is limiting. It

assumes there is a fixed sized pie. If one group gets a larger slice, then someone else must necessarily

receive a smaller share. That is a negative feedback analysis, and conflicts with the fundamental premise

of positive feedback.

Positive feedback philosophy is the basis for developing new technology. It assumes something that has

little value can have value added through technology, thereby creating more value or wealth. This

comes without reducing the wealth of others.

Consider a common example. Sand has little value. The most common component of sand, however is

silicon oxide; it can be modified to create the silicon wafers used in microelectronics. A previously

limited value material now has great value.

Page 280: Dr. Robert A. Durham, PhD, PE Dr. Marcus O. Durham, PhD, PE … Nutshell Master.pdf · Dr. Robert A. Durham, PhD, PE . Dr. Marcus O. Durham, PhD, PE . A Quick Reference . The book

Page | 7-2

7.2 Time and Interest At the present time money has a known value, called the present value or purchase price, PV. At some

time in the future, the same money will have a different value called the future value, FV.

Two things relate the values. Interest rate, i, is the percentage of a sum of money charged for its use.

This is also called the discount rate. Interest is based on a period of time. In most cases it is one year, but

it can be any interval. The number of interest periods is n.

It is assumed that all money transfer is at the end of a period. If it is at the beginning, the present value

must be calculated from one period in the future.

The simplest calculation is for a single payment exemplified in the cashflow diagram. A value below the

line is negative or paid out. A value above the line is positive or received in. Hash marks without arrows

simply identify a time period.

FV

PV

Equations for calculating the values are shown below. In addition, many texts give an abbreviation for

the factor that relates the value to be calculated to the known value. In addition, we will show the

equivalent spreadsheet command, (Microsoft Excel 2003), since that is the most common way of

calculating value of money. [Excel] The Excel formulae start with an “=” sign

Find the amount of money that it would take in the future, FV, which has an equivalent value to the

present worth, PV.

*(1 )nFV PV i

( , ,0, ,)FV i n PV

Conversely, when the value in the future is known, determine the present value.

*(1 ) nPV FV i

( , ,0, ,)PV i n FV =PV(i,n,0,FV,)

Page 281: Dr. Robert A. Durham, PhD, PE Dr. Marcus O. Durham, PhD, PE … Nutshell Master.pdf · Dr. Robert A. Durham, PhD, PE . Dr. Marcus O. Durham, PhD, PE . A Quick Reference . The book

Page | 7-3

7.2.1 Uniform series

Rather than single payments, a much more common practice is to have progress payment amounts, A. If

these are at regular time intervals, the process is called a uniform series. When uniform amounts are

applied to the future value and present value, four new relationships are established. Some authors

treat these as equivalent uniform annual cost (EUAC).

Future value and uniform amounts are illustrated in a cashflow diagram.

FV

A A A A A A A

Series compound amount is the future value found from the uniform amount.

(1 ) 1ni

FV Ai

( , , ,,)FV i n A

Sinking fund is the uniform payment found from the future value.

(1 ) 1n

iA FV

i

( , ,0, ,)PMT i n FV

The cashflow diagram demonstrates the present worth and the uniform payment amounts.

PV

A A A A A A A

Series present worth is the present value found from the uniform payment.

(1 ) 1

(1 )

n

n

iPV A

i i

( , , ,,)PV i n A

Page 282: Dr. Robert A. Durham, PhD, PE Dr. Marcus O. Durham, PhD, PE … Nutshell Master.pdf · Dr. Robert A. Durham, PhD, PE . Dr. Marcus O. Durham, PhD, PE . A Quick Reference . The book

Page | 7-4

Capital recovery is the uniform payment found from the present value.

(1 )

(1 ) 1

n

n

i iA PV

i

( , , ,,)PMT i n PV

Capital recovery on salvage is a modification of the capital recovery relationship. The future value of the

salvage, S, is converted to a uniform payment and subtracted from the present value convert to a

uniform series. Alternately the salvage value is transferred through the present worth as developed

above.

(1 )

( )(1 ) 1

n

n

i iA PV S iS

i

( , , , )PMT i n PV S

Page 283: Dr. Robert A. Durham, PhD, PE Dr. Marcus O. Durham, PhD, PE … Nutshell Master.pdf · Dr. Robert A. Durham, PhD, PE . Dr. Marcus O. Durham, PhD, PE . A Quick Reference . The book

Page | 7-5

7.2.2 Gradient

A gradient is a value that increases or decreases at a rate for each time interval. An arithmetic gradient

is an integer multiple during each interval. It can have either a uniform amount or a present value, PV.

G

A A A A

PV

AA

2G

3G

4G

(n-1)G

Arithmetic gradient uniform series is calculated from the gradient value, G, after the first interval.

1

(1 )n

nA G

i i i

In a similar manner the present value is derived from the first gradient, G.

(1 ) 1

(1 )

n

n

i inPV G

i i i

A geometric gradient grows by a rate, g, during each interval. Therefore the change in amount varies

according to an exponential curve.

A1

PV

A2

A3

A4

An

The present value depends on two rates, the interest and the gradient, as well as the first amount in the

series.

1 (1 ) (1 )n ng i

PV Ai g

Frequently, the gradient rate is equal to the interest rate. Then the equation reduces significantly.

11 (1 )PV A n i

Page 284: Dr. Robert A. Durham, PhD, PE Dr. Marcus O. Durham, PhD, PE … Nutshell Master.pdf · Dr. Robert A. Durham, PhD, PE . Dr. Marcus O. Durham, PhD, PE . A Quick Reference . The book

Page | 7-6

7.2.3 Nominal Interest or APR

Nominal interest rate, r, is also called the annual percentage rate (APR). It is the reported rate that is

compounded multiple times M, per year. It may be stated as 18% APR or 1.5% per month. These are

equivalent. However, they are not the effective interest rate.

Effective annual interest rate, ia is also called annual effective yield (AYR). It is substantially greater than

the nominal rate.

1 1

m

a

Ri

M

The most insidious effect is continuous compounding at a nominal rate. This creates an exponential

growth.

For a single payment, the relationships are simple, but expensive.

( )rnFV PV e

( )rnPV FV e

Page 285: Dr. Robert A. Durham, PhD, PE Dr. Marcus O. Durham, PhD, PE … Nutshell Master.pdf · Dr. Robert A. Durham, PhD, PE . Dr. Marcus O. Durham, PhD, PE . A Quick Reference . The book

Page | 7-7

7.2.4 Perpetual and Rule of 72

Present value of perpetuities is a common way to determine the worth of an ongoing, perpetual uniform

payment. this is valuable in evaluating annuities, insurance, and lotteries.

A

PVi

Rule of 72 is a quick technique to determine how long it takes for the present value of money to double.

The number Of periods is determined from 72 divided by the interest rate.

72

ni

Page 286: Dr. Robert A. Durham, PhD, PE Dr. Marcus O. Durham, PhD, PE … Nutshell Master.pdf · Dr. Robert A. Durham, PhD, PE . Dr. Marcus O. Durham, PhD, PE . A Quick Reference . The book

Page | 7-8

7.3 Rate of Return

Project economics are based on recovery of money. A variation of present worth has been used in all

the calculations to this point. Projects typically can be implemented with a number of different

alternatives. Therefore, the economic analysis is a comparison of alternatives.

Internal rate of return (IRR) is the break-even interest rate i* at which the present worth of a project is

zero. With a present worth of zero, we are ambivalent and do not particularly care about the project.

( *) PV cas in - PV cash out =0PV i

The calculation of IRR is an iterative process. Begin with the number of time intervals, n, and the

present, future, or uniform amounts. Guess at an interest rate and calculate the present worth. If it is

positive increase the interest rate. If the present worth is negative decrease the interest rate. Continue

estimating interests until the present value is adequately close to zero.

It is not necessary to laboriously perform this calculation. Many calculators and spreadsheets have the i*

function built into the spreadsheet package (Microsoft Excel 2003).

( , )IRR values guess

Investors have a minimum acceptable rate of return, MARR. This is the interest that must be exceeded if

the project is to be funded. Compare the calculated rate of return with the MARR. If the IRR is less than

the MARR, then the project should not be done.

Typically the analyst will go back and tweak the costs associated with the project until an acceptable IRR

is obtained. This is a very important process since it forces the analyst to have a better handle on the

project, and, as a result, the project is more likely to perform as expected.

Page 287: Dr. Robert A. Durham, PhD, PE Dr. Marcus O. Durham, PhD, PE … Nutshell Master.pdf · Dr. Robert A. Durham, PhD, PE . Dr. Marcus O. Durham, PhD, PE . A Quick Reference . The book

Page | 7-9

7.3.1 Incremental Analysis

There is an anomaly with simple IRR analysis. For projects with a short term and low investment, the IRR

may be very high. However, that project may not have the greatest present worth. This is because

present worth is an absolute dollar measurement while IRR is a relative measurement.

Incremental investment analysis is used to compare the difference in investments for projects.

Begin with a table for the number of years. In year zero, place the present worth or purchase price

under each project. In year one, place the future value or uniform amount under each project. Continue

for all remaining years.

Next create a column that is the difference in the cashflow between the two projects at each time

interval. This gives a sequence of incremental values.

n Project1 Project2 2 –1

0 P10 P20 P20 – P10

1 F11 F21 F21 – F11

2 F12 F22 F22 – F21

IRR1 IRR2 IRR2-1

Next calculate the rate of return on the differences. If the rate of return on the difference exceeds the

MARR, then the incremental investment should be made to do the project.

Page 288: Dr. Robert A. Durham, PhD, PE Dr. Marcus O. Durham, PhD, PE … Nutshell Master.pdf · Dr. Robert A. Durham, PhD, PE . Dr. Marcus O. Durham, PhD, PE . A Quick Reference . The book

Page | 7-10

7.3.2 Payback

Payback is simply how long it takes for net receipts to equal investment outlays. Conventional payback is

calculated ignoring the time value of money.

Discounted payback includes the interest on the money. For a project with the same cash inflows each

year,

cost

Payback=uniform annual benefit

For projects with varying cash inflows, these inflows are added until the cost is equaled.

For payback analysis, all costs and all revenue prior to the payback are included without considering

differences in their timing. All consequences after the payback time are completely ignored.

This is obviously a simple technique. It also has some serious constraints. Nevertheless, it is still a

popular indicator since it gives the time to recover the investment. It is important to note again that the

simple payback ignores the time value of money, which may be very high for an expensive, or lengthy,

project.

Page 289: Dr. Robert A. Durham, PhD, PE Dr. Marcus O. Durham, PhD, PE … Nutshell Master.pdf · Dr. Robert A. Durham, PhD, PE . Dr. Marcus O. Durham, PhD, PE . A Quick Reference . The book

Page | 7-11

7.3.3 Benefit Cost Ratio

Benefit is simply another way a describing the positive cash flow or value. Cost describes the negative

cash flow or value. When comparing projects, these are calculated using incremental analysis. The ratio

is the benefit to cost ratio. If the ratio is greater than 1, the incremental project should be done.

(benefits)

(costs)

PVBC

PV

Page 290: Dr. Robert A. Durham, PhD, PE Dr. Marcus O. Durham, PhD, PE … Nutshell Master.pdf · Dr. Robert A. Durham, PhD, PE . Dr. Marcus O. Durham, PhD, PE . A Quick Reference . The book

Page | 7-12

7.3.4 Tax Implications

The taxes on income and investments will influence the value of the project. Calculate the value of the

project before income tax (BFIT). Financial types refer to this as EBITDA (Earnings Before Interest, Tax,

Depreciation and Amortization). Calculate the value of the project after income tax (AFIT). The process

can be used with all the techniques including present worth, incremental analysis IRR, and payback.

The value of the analysis is it reflects the real net worth, since different investments have alternative tax

implications. BFIT may be a better indicator of projects, but AFIT is the better indicator of value to the

organization. After all, the purpose of projects is not for the projects sake, but to increase the value to

the organization.

Page 291: Dr. Robert A. Durham, PhD, PE Dr. Marcus O. Durham, PhD, PE … Nutshell Master.pdf · Dr. Robert A. Durham, PhD, PE . Dr. Marcus O. Durham, PhD, PE . A Quick Reference . The book

Page | 7-13

7.4 Table of Terminology

Numerous symbols are used for economic comparisons. These are shown in a table.

Symbol Definition I Interest rate per interest period, often one year N Number of interest periods PV Present value of money FV Future value of money A Amount of money at end of period in a uniform series for n periods G Arithmetic gradient, uniform period change in amount of money G Geometric gradient, uniform rate of cash flow change. R Nominal interest rate per interest period M Number of compounding subperiods S Salvage value is cash recoverable at the end of a project. PV*, FV* Amount of money flowing continuously during one period

7.5 Commentary

Determining which project is the best choice is less than straightforward. Many issues enter the

evaluation, besides the pure economic calculations.

There are biases and preferences from the management and the project team. These are difficult to

eliminate, and the biases may even be desirable. With practice, there can come that ‘gut feel’ that this is

right. However, gut feel can also be wrong.

Most organizations place some criteria on selecting projects. Traditionally over many years, an IRR of

15% AFIT has been acceptable, assuming the project was well defined. For higher risk ventures, the

MARR often moves toward 30%.

Other companies place great emphasis on payback. Traditionally, a 3-year term has been acceptable. As

risk increases, the term is frequently shortened toward 1 year.

With these more stringent criteria, one must wonder if any projects will ever be built. That is not the

point. The objective is to compensate for higher than normal risk. There is no future in investing money

in ventures that do not increase the wealth of the stakeholders.

7.6 Review

Page 292: Dr. Robert A. Durham, PhD, PE Dr. Marcus O. Durham, PhD, PE … Nutshell Master.pdf · Dr. Robert A. Durham, PhD, PE . Dr. Marcus O. Durham, PhD, PE . A Quick Reference . The book

Page | 7-14

Money has different values, depending on the time it is obtained. The worth of the money is dependent

on present, future, and uniform amounts. There are multiple projects on which to invest money. The

preferred project is determined by incremental analysis that compares the projects. The methods are

investor rate of return, payback, and benefit to cost ratio. The return after taxes is a better indicator of

worth.

Page 293: Dr. Robert A. Durham, PhD, PE Dr. Marcus O. Durham, PhD, PE … Nutshell Master.pdf · Dr. Robert A. Durham, PhD, PE . Dr. Marcus O. Durham, PhD, PE . A Quick Reference . The book

Page | 7-15

Chapter 7 Problems

1. Retirement savings. Given: Uninflated growth of capital is 3% per year. You begin paying into a retirement program when you turn 25. After25 years, you begin collecting $50,000 annually. You collect for 40 years. a. What is the present value of the annuity? b. What are your annual contributions to have that amount in the account? c. How much will be in the account when you begin collecting? d. Is the annual contribution a reasonable amount to expect you can contribute after you pay taxes

on a $50,000 salary? e. Wait 35 years to begin collection and receive for 30 years, do you contribute more or less each

month? f. To retire at 50, what is the best approach?

2. Auto Purchase. Given: A new university graduate purchases a new Whizmobile for $25,000. She borrows the money at an interest of 1%/mo. Loan period is 60 months. a. Find, future value, if a single balloon payment is made at 60 months. b. Monthly payments if made at the end of each month. c. Total amount paid (cash-flow) d. Total Interest paid e. Amount unpaid after 12 months a. Assume the same monthly payments were deposited in savings program. The account pays 1%

per month. How many months would it before you had $25,000?

3. Rule of 72: Given: $1000 to invest and the interest is compounded 10% per year. a. How long will it take to double your money? b. What is the effective monthly interest rate?

4. House Purchase. Given: Engineer with a salary of $40,000 per year. Federal and state income tax of 25%. Mortgage company will lend 2.5 times the annual salary for a house. Interest rate is 7% per year. Loan period is 30 years. Use uniform series of annual payments procedure.

Create a spreadsheet with the following columns. Calculate the values for the end of each year.

a. Principal remaining at start of year b. Interest owed for year c. Annual payment d. Amount of payment that is principal e. Total payments to date

Page 294: Dr. Robert A. Durham, PhD, PE Dr. Marcus O. Durham, PhD, PE … Nutshell Master.pdf · Dr. Robert A. Durham, PhD, PE . Dr. Marcus O. Durham, PhD, PE . A Quick Reference . The book

Page | 7-16

f. Total interest to date g. Double the payment. h. Apply the extra amount to principal only. i. Tabulate the lower principal remaining. j. How long does it take to pay off the loan, with double payments?

Calculate the following items for a 15-year loan.

a. Total cash flow (amount paid). b. Total interest paid.

The next sections should be added to the spreadsheet after taxes.

c. Effect on income tax for interest paid d. Cash flow after taxes

Bibliography

Anthony Tarquin, Anthony Engineering Economy, McGraw-Hill Education, 1989

Excel 2003, Microsoft, Redmond, WA, 2003.

Page 295: Dr. Robert A. Durham, PhD, PE Dr. Marcus O. Durham, PhD, PE … Nutshell Master.pdf · Dr. Robert A. Durham, PhD, PE . Dr. Marcus O. Durham, PhD, PE . A Quick Reference . The book

Page | 8-1

Chapter 8 Business Ethics Ethics and professional behavior are critical components for the practice of any profession. The

expected professional conduct is outlined in the NCEES Model Rules [1]. These are excerpted for review

and consideration and credit is acknowledged.

240.15 Rules of Professional Conduct A. Licensee’s Obligation to Society

1. Licensees, in the performance of their services for clients, employers, and customers, shall be

cognizant that their first and foremost responsibility is to the public welfare.

2. Licensees shall approve and seal only those design documents and surveys that conform to accepted

engineering and surveying standards and safeguard the life, health, property, and welfare of the public.

3. Licensees shall notify their employer or client and such other authority as may be appropriate when

their professional judgment is overruled under circumstances where the life, health, property, or

welfare of the public is endangered.

4. Licensees shall be objective and truthful in professional reports, statements, or testimony. They shall

include all relevant and pertinent information in such reports, statements, or testimony.

5. Licensees shall express a professional opinion publicly only when it is founded upon an adequate

knowledge of the facts and a competent evaluation of the subject matter.

6. Licensees shall issue no statements, criticisms, or arguments on technical matters which are inspired

or paid for by interested parties, unless they explicitly identify the interested parties on whose behalf

they are speaking and reveal any interest they have in the matters.

7. Licensees shall not permit the use of their name or firm name by, nor associate in the business

ventures with, any person or firm which is engaging in fraudulent or dishonest business or professional

practices.

8. Licensees having knowledge of possible violations of any of these Rules of Professional Conduct shall

provide the board with the information and assistance necessary to make the final determination of

such violation. (Section 150, Disciplinary Action, NCEES Model Law)

B. Licensee’s Obligation to Employer and Clients

1. Licensees shall undertake assignments only when qualified by education or experience in the specific

technical fields of engineering or surveying involved.

2. Licensees shall not affix their signatures or seals to any plans or documents dealing

with subject matter in which they lack competence, nor to any such plan or document not prepared

under their direct control and personal supervision.

3. Licensees may accept assignments for coordination of an entire project, provided that each design

segment is signed and sealed by the licensee responsible for preparation of that design segment.

Page 296: Dr. Robert A. Durham, PhD, PE Dr. Marcus O. Durham, PhD, PE … Nutshell Master.pdf · Dr. Robert A. Durham, PhD, PE . Dr. Marcus O. Durham, PhD, PE . A Quick Reference . The book

Page | 8-2

4. Licensees shall not reveal facts, data, or information obtained in a professional capacity without the

prior consent of the client or employer except as authorized or required by law. Licensees shall not

solicit or accept gratuities, directly or indirectly, from contractors, their agents, or other parties in

connection with work for employers or clients.

5. Licensees shall make full prior disclosures to their employers or clients of potential conflicts of

interest or other circumstances which could influence or appear to influence their judgment or the

quality of their service.

6. Licensees shall not accept compensation, financial or otherwise, from more than one party for

services pertaining to the same project, unless the circumstances are fully disclosed and agreed to by all

interested parties.

7. Licensees shall not solicit or accept a professional contract from a governmental body on which a

principal or officer of their organization serves as a member. Conversely, licensees serving as members,

advisors, or employees of a government body or department, who are the principals or employees of a

private concern, shall not participate in decisions with respect to professional services offered or

provided by said concern to the governmental body which they serve. (Section 150, Disciplinary Action,

NCEES Model Law)

C. Licensee’s Obligation to Other Licensees

1. Licensees shall not falsify or permit misrepresentation of their, or their associates’, academic or

professional qualifications. They shall not misrepresent or exaggerate their degree of responsibility in

prior assignments nor the complexity of said assignments. Presentations incident to the solicitation of

employment or business shall not misrepresent pertinent facts concerning employers, employees,

associates, joint ventures, or past accomplishments.

2. Licensees shall not offer, give, solicit, or receive, either directly or indirectly, any commission, or gift,

or other valuable consideration in order to secure work, and shall not make any political contribution

with the intent to influence the award of a contract by public authority.

3. Licensees shall not attempt to injure, maliciously or falsely, directly or indirectly, the professional

reputation, prospects, practice, or employment of other licensees, nor indiscriminately criticize other

licensees’ work. (Section 150, Disciplinary Action, NCEES Model Law)

Model Rules, National Council of Examiners for Engineering and Surveying, Clemson, SC,

http://www.ncees.org/introduction/about_ncees/ncees_model_rules.pdf, 2005

Page 297: Dr. Robert A. Durham, PhD, PE Dr. Marcus O. Durham, PhD, PE … Nutshell Master.pdf · Dr. Robert A. Durham, PhD, PE . Dr. Marcus O. Durham, PhD, PE . A Quick Reference . The book

Page | 9-1

Chapter 9 Codes and Standards

9.1 Introduction

There are minimum performance requirements that should be incorporated in any design. The actual

installation should meet or exceed these.

Standards are consensus documents typically developed by professional and technical societies. These

typically become an American National Standards Institute (ANSI) standard.

Codes are standards that have been incorporated as law.

Industry practices are common techniques used by knowledgeable individuals.

9.2 NEC Synopsis The National Electrical Code (NEC) is the standard for practical safeguarding of persons and equipment.

The areas that involve calculations are the components that are most frequently on the professional

exams. Articles are used as broad categories from NEC 2005. The specific section & table numbers may

change between versions of the NEC.

General: Article 100

Definitions, workmanship, and responsibility are discussed in this section.

Identification of grounded conductors: Article 200

Grounding is bare, green, or green with yellow stripe.

Grounded, or neutral, is white or gray.

High-leg or wild-leg of a three-phase delta system with a center-tapped winding that is grounded must be orange. (Article 110)

Branch circuits: Article 210

Branch circuits are everything but motors.

General lighting can be on 15 A breakers.

The minimum number of 20 A branch circuits in a residence is 2 for small appliances, 1 for bath, and 1 for laundry.

Ground-fault protection is required in bathrooms, kitchens, garages, outside, and any other place that contact with a ground is possible.

Arc-fault breakers are required for bedrooms.

Feeders: Article 215

Ampacity = (1.25 * largest Full Load Amps) + sum of all other current loads.

Page 298: Dr. Robert A. Durham, PhD, PE Dr. Marcus O. Durham, PhD, PE … Nutshell Master.pdf · Dr. Robert A. Durham, PhD, PE . Dr. Marcus O. Durham, PhD, PE . A Quick Reference . The book

Page | 9-2

Branch-circuit, feeder, and service calculations: Article 220

Demand factors and calculation requirements are covered.

Start in Part III and step through the applicable sections.

Overcurrent protection: Article 240

Overcurrent protection for specific equipment: Table 240.3

Protection of conductors shall be in accordance with ampacity from Table 310.16 ff

Grounding: Article 250

Sizing of grounding electrode conductor: Table 250.66

Sizing of equipment grounding conductor: Table 250.122

The maximum resistance of a grounding electrode according to Art 250.56: 25 Ω,

Conductor to be grounded - neutral or common: Art 250.26

Conductors for general wiring: Table 310.16 ff

Maximum capacity is shown in table at 30C

* AWG 14, 12, & 10 have maximum rating based on Art 240.4D, which is 5 A less than Table 310.16

Ampacity is rerated for different temperatures by the correction factors.

NM cable (Romex) is sized as 60C, regardless of insulation type

Alternate ampacity can be used under engineering supervision using Art 310.60D

Motors: Article 430

Motor full load current rating: Table 430.247 ff

Short circuit current = 6* full load current

Overload current = 1.5 * full load current

Maximum setting of motor branch circuit protection & ground fault protection: Table 430.52

Maximum locked-rotor current for selection of disconnecting means: Table 430.251

Locked rotor code letters for KVA per horsepower: Table 430.7

Environmental selection of motor controller enclosures: Table 430.91

Capacitor added to motor terminal: reduces total current from overload, so reduce size of overload only

M

Tables: Chapter 9

Conduit dimensions and fill

Insulated conductors nominal dimensions for AWG, diameter, area

Page 299: Dr. Robert A. Durham, PhD, PE Dr. Marcus O. Durham, PhD, PE … Nutshell Master.pdf · Dr. Robert A. Durham, PhD, PE . Dr. Marcus O. Durham, PhD, PE . A Quick Reference . The book

Page | 9-3

Conductor properties including AWG, area, Ohms/kft

Cable resistance and reactance in conduit

Power source limitations

Examples: Annex D

Single family dwelling with appliance, heating, air conditioning

Store building

Industrial feeders

Multifamily dwelling with demand factors on single-phase and three-phase

Generator field control

Mobile home

Page 300: Dr. Robert A. Durham, PhD, PE Dr. Marcus O. Durham, PhD, PE … Nutshell Master.pdf · Dr. Robert A. Durham, PhD, PE . Dr. Marcus O. Durham, PhD, PE . A Quick Reference . The book

Page | 9-4

9.3 Motor Installation Tables This section lists excerpts from industry, national, and international standards. The excerpts are for

illustration and educational purposes. There are often several related tables and information for other

configurations and applications. In addition, the standards have added detail and information that

applies to all these excerpts. Therefore, any application should refer to the standard, rather than the

excerpts.

Standard Organization Application

National Electrical Code (NEC) NFPA 70-2005

National Fire Protection Association, Batterymarch Park, Quincy, MA

Electrical installations in occupancies

National Electrical Safety Code (NESC) IEEE C2-2002

Institute of Electrical & Electronics Engineers, New York, NewYork

Electrical supply stations, overhead, and underground lines.

Motors and Generators (MG1) NEMA MG1-2003

National Electrical Manufacturers Association

Performance of motors and generators

NEC 240.6(A) Standard Ampere Ratings for Fuses & Circuit Breakers

Amperes

15 20 25 30 35 40 45 50 60 70 80 90 100 110 125 150 175 200 225 250 300 350 400 450 500 - - - 600 700 800 1000 1200 1600 2000 2500 3000 4000 5000 6000 - -

Page 301: Dr. Robert A. Durham, PhD, PE Dr. Marcus O. Durham, PhD, PE … Nutshell Master.pdf · Dr. Robert A. Durham, PhD, PE . Dr. Marcus O. Durham, PhD, PE . A Quick Reference . The book

Page | 9-5

Table 310.16 Allowable Ampacities of Insulated Conductors Rated 0 Through 2000 Volts, 60°C

Through 90°C (140°F Through 194°F), Not More Than Three Current-Carrying Conductors in Raceway,

Cable, or Earth (Directly Buried), Based on Ambient Temperature of 30°C (86°F)

Temperature Rating of Conductor (See Table 310.13.)

60°C (140°F)

75°C (167°F)

90°C (194°F)

60°C (140°F)

75°C (167°F)

90°C (194°F)

Size AWG or kcmil

Types TW, UF

Types RHW, THHW, THW,

THWN, XHHW, USE, ZW

Types TBS, SA, SIS, FEP, FEPB, MI, RHH, RHW-2, THHN, THHW, THW-2, THWN-2, USE-2, XHH,

XHHW, XHHW-2, ZW-2

Types TW, UF

Types RHW, THHW, THW,

THWN, XHHW, USE

Types TBS, SA, SIS, THHN, THHW, THW-2, THWN-2, RHH, RHW-2,

USE-2, XHH, XHHW, XHHW-2, ZW-2

Size AWG or kcmil

COPPER ALUMINUM OR COPPER-CLAD ALUM

18 — — 14 — — — — 16 — — 18 — — — —

14* 20 20 25 — — — — 12* 25 25 30 20 20 25 12* 10* 30 35 40 25 30 35 10*

8 40 50 55 30 40 45 8

6 55 65 75 40 50 60 6 4 70 85 95 55 65 75 4 3 85 100 110 65 75 85 3 2 95 115 130 75 90 100 2 1 110 130 150 85 100 115 1

1/0 125 150 170 100 120 135 1/0 2/0 145 175 195 115 135 150 2/0 3/0 165 200 225 130 155 175 3/0 4/0 195 230 260 150 180 205 4/0

250 215 255 290 170 205 230 250 300 240 285 320 190 230 255 300 350 260 310 350 210 250 280 350 400 280 335 380 225 270 305 400 500 320 380 430 260 310 350 500

600 355 420 475 285 340 385 600 700 385 460 520 310 375 420 700 750 400 475 535 320 385 435 750 800 410 490 555 330 395 450 800 900 435 520 585 355 425 480 900

1000 455 545 615 375 445 500 1000 1250 495 590 665 405 485 545 1250 1500 520 625 705 435 520 585 1500 1750 545 650 735 455 545 615 1750 2000 560 665 750 470 560 630 2000

CORRECTION FACTORS

Ambient Temp. (°C)

For ambient temperatures other than 30°C (86°F), multiply the allowable ampacities shown above by the appropriate factor shown below.

Ambient Temp. (°F)

21–25 1.08 1.05 1.04 1.08 1.05 1.04 70–77

26–30 1.00 1.00 1.00 1.00 1.00 1.00 78–86

31–35 0.91 0.94 0.96 0.91 0.94 0.96 87–95

36–40 0.82 0.88 0.91 0.82 0.88 0.91 96–104

41–45 0.71 0.82 0.87 0.71 0.82 0.87 105–113

46–50 0.58 0.75 0.82 0.58 0.75 0.82 114–122

51–55 0.41 0.67 0.76 0.41 0.67 0.76 123–131

56–60 — 0.58 0.71 — 0.58 0.71 132–140

61–70 — 0.33 0.58 — 0.33 0.58 141–158

71–80 — — 0.41 — — 0.41 159–176

* See 240.4(D).

Page 302: Dr. Robert A. Durham, PhD, PE Dr. Marcus O. Durham, PhD, PE … Nutshell Master.pdf · Dr. Robert A. Durham, PhD, PE . Dr. Marcus O. Durham, PhD, PE . A Quick Reference . The book

Page | 9-6

Table 310.15(B)(2)(a) Adjustment

Factors for More Than Three Current-

Carrying Conductors in a Raceway or

Cable

Number of

Current-

Carrying

Conductors

Percent of Values in Tables

310.16 through 310.19 as

Adjusted for Ambient

Temperature if Necessary

4–6 80

7–9 70

10–20 50

21–30 45

31–40 40

41 and above 35

Table 310.15(B)(6) Conductor Types and Sizes

for 120/240-Volt, 3-Wire, Single-Phase Dwelling

Services and Feeders.

Conductor Types RHH, RHW, RHW-2, THHN, THHW, THW,

THW-2, THWN, THWN-2, XHHW, XHHW-2, SE, USE, USE-2

Conductor (AWG or kcmil)

Copper Aluminum or Copper-Clad Aluminum

Service or Feeder Rating (Amperes)

4 2 100 3 1 110 2 1/0 125 1 2/0 150

1/0 3/0 175 2/0 4/0 200 3/0 250 225 4/0 300 250 250 350 300 350 500 350 400 600 400

Page 303: Dr. Robert A. Durham, PhD, PE Dr. Marcus O. Durham, PhD, PE … Nutshell Master.pdf · Dr. Robert A. Durham, PhD, PE . Dr. Marcus O. Durham, PhD, PE . A Quick Reference . The book

Page | 9-7

Figure 430.1 Article 430 Contents

Page 304: Dr. Robert A. Durham, PhD, PE Dr. Marcus O. Durham, PhD, PE … Nutshell Master.pdf · Dr. Robert A. Durham, PhD, PE . Dr. Marcus O. Durham, PhD, PE . A Quick Reference . The book

Page | 9-8

Table 430.7(B) Locked-Rotor Indicating Code Letters

Code Letter

Kilovolt-Amperes per Horsepower

with Locked Rotor

A 0–3.14 B 3.15–3.54 C 3.55–3.99 D 4.0–4.49 E 4.5–4.99 F 5.0–5.59 G 5.6–6.29 H 6.3–7.09 J 7.1–7.99 K 8.0–8.99

Code Letter

Kilovolt-Amperes per Horsepower

with Locked Rotor

L 9.0–9.99 M 10.0–11.19 N 11.2–12.49 P 12.5–13.99 R 14.0–15.99 S 16.0–17.99 T 18.0–19.99 U 20.0–22.39 V 22.4 and up

Table 430.52 Maximum Rating or Setting of Motor Branch-Circuit Short-

Circuit and Ground-Fault Protective Devices

Percentage of Full-Load Current Type of Motor Non-time

Delay Fuse Dual Element (Time-Delay) Fuse

Instantaneous Trip Breaker

Inverse Time Breaker

Single-phase motors

300 175 800 250

AC polyphase motors other than wound-rotor Squirrel cage other than Design B energy-efficient

300 175 800 250

Design B energy-efficient

300 175 1100 250

Synchronous 300 175 800 250 Wound rotor 150 150 800 150 Direct current (constant voltage)

150 150 250 150

Page 305: Dr. Robert A. Durham, PhD, PE Dr. Marcus O. Durham, PhD, PE … Nutshell Master.pdf · Dr. Robert A. Durham, PhD, PE . Dr. Marcus O. Durham, PhD, PE . A Quick Reference . The book

Page | 9-9

Table 430.91 Motor Controller Enclosure Selection

For Outdoor Use

Enclosure Type Number1

Provides a Degree of Protection Against the Following Environmental Conditions

3 3R 3S 3X 3RX 3SX 4 4X 6 6P

Incidental contact with the enclosed equipment

X X X X X X X X X X

Rain, snow, and sleet X X X X X X X X X X

Sleet2 — — X — — X — — — —

Windblown dust X — X X — X X X X X

Hosedown — — — — — — X X X X

Corrosive agents — — — X X X — X — X

Temporary submersion — — — — — — — — X X

Prolonged submersion — — — — — — — — — X

For Indoor Use

Enclosure Type Number1

Provides a Degree of Protection Against the Following Environmental Conditions

1 2 4 4X 5 6 6P 12 12K 13

Incidental contact with the enclosed equipment

X X X X X X X X X X

Falling dirt X X X X X X X X X X

Falling liquids and light splashing

— X X X X X X X X X

Circulating dust, lint, fibers, and flyings

— — X X — X X X X X

Settling airborne dust, lint, fibers, and flyings

— — X X X X X X X X

Hosedown and splashing water

— — X X — X X — — —

Oil and coolant seepage — — — — — — — X X X

Oil or coolant spraying and splashing

— — — — — — — — — X

Corrosive agents — — — X — — X — — —

Temporary submersion — — — — — X X — — —

Prolonged submersion — — — — — — X — — —

Page 306: Dr. Robert A. Durham, PhD, PE Dr. Marcus O. Durham, PhD, PE … Nutshell Master.pdf · Dr. Robert A. Durham, PhD, PE . Dr. Marcus O. Durham, PhD, PE . A Quick Reference . The book

Page | 9-10

1Enclosure type number shall be marked on the motor controller enclosure. 2Mechanism shall be operable when ice covered. FPN: The term raintight is typically used in conjunction with Enclosure Types 3, 3S, 3SX, 3X, 4, 4X, 6, 6P. The term rainproof is typically used in conjunction with Enclosure Type 3R, 3RX. The term watertight is typically used in conjunction with Enclosure Types 4, 4X, 6, 6P. The term driptight is typically used in conjunction with Enclosure Types 2, 5, 12, 12K, 13. The term dusttight is typically used in conjunction with Enclosure Types 3, 3S, 3SX, 3X, 5, 12, 12K, 13.

Page 307: Dr. Robert A. Durham, PhD, PE Dr. Marcus O. Durham, PhD, PE … Nutshell Master.pdf · Dr. Robert A. Durham, PhD, PE . Dr. Marcus O. Durham, PhD, PE . A Quick Reference . The book

Page | 9-11

Table 430.248 Full-Load

Currents in Amperes, Single-Phase

Alternating-Current Motors

Horse-power

115 Volts

200 Volts

208 Volts

230 Volts

4.4 2.5 2.4 2.2

¼ 5.8 3.3 3.2 2.9

7.2 4.1 4.0 3.6

½ 9.8 5.6 5.4 4.9 ¾ 13.8 7.9 7.6 6.9 1 16 9.2 8.8 8.0 1½ 20 11.5 11.0 10 2 24 13.8 13.2 12 3 34 19.6 18.7 17 5 56 32.2 30.8 28 7½ 80 46.0 44.0 40 10 100 57.5 55.0 50

Page 308: Dr. Robert A. Durham, PhD, PE Dr. Marcus O. Durham, PhD, PE … Nutshell Master.pdf · Dr. Robert A. Durham, PhD, PE . Dr. Marcus O. Durham, PhD, PE . A Quick Reference . The book

Page | 9-12

Table 430.250 Full-Load Current, Three-Phase Alternating-Current Motors

Induction-Type Squirrel Cage and Wound Rotor (Amperes)

Synchronous-Type Unity Power Factor* (Amperes)

Horse-power

115 Volts

200 Volts

208 Volts

230 Volts

460 Volts

575 Volts 2300 Volts

230 Volts

460 Volts

575 Volts

2300 Volts

½ 4.4 2.5 2.4 2.2 1.1 0.9 — — — — — ¾ 6.4 3.7 3.5 3.2 1.6 1.3 — — — — — 1 8.4 4.8 4.6 4.2 2.1 1.7 — — — — — 1½ 12.0 6.9 6.6 6.0 3.0 2.4 — — — — — 2 13.6 7.8 7.5 6.8 3.4 2.7 — — — — —

3 — 11.0 10.6 9.6 4.8 3.9 — — — — — 5 — 17.5 16.7 15.2 7.6 6.1 — — — — — 7½ — 25.3 24.2 22 11 9 — — — — — 10 — 32.2 30.8 28 14 11 — — — — — 15 — 48.3 46.2 42 21 17 — — — — —

20 — 62.1 59.4 54 27 22 — — — — — 25 — 78.2 74.8 68 34 27 — 53 26 21 — 30 — 92 88 80 40 32 — 63 32 26 — 40 — 120 114 104 52 41 — 83 41 33 — 50 — 150 143 130 65 52 — 104 52 42 —

60 — 177 169 154 77 62 16 123 61 49 12 75 — 221 211 192 96 77 20 155 78 62 15 100 — 285 273 248 124 99 26 202 101 81 20 125 — 359 343 312 156 125 31 253 126 101 25

150 — 414 396 360 180 144 37 302 151 121 30 200 552 528 480 240 192 49 400 201 161 40 250 — — — — 302 242 60 — — — — 300 — — — — 361 289 72 — — — — 350 — — — — 414 336 83 — — — —

400 — — — — 477 382 95 — — — — 450 — — — — 515 412 103 — — — — 500 — — — — 590 472 118 — — — —

*For 90 and 80 percent power factor, the figures shall be multiplied by 1.1 and 1.25, respectively.

Page 309: Dr. Robert A. Durham, PhD, PE Dr. Marcus O. Durham, PhD, PE … Nutshell Master.pdf · Dr. Robert A. Durham, PhD, PE . Dr. Marcus O. Durham, PhD, PE . A Quick Reference . The book

Page | 9-13

Table 430.251(A) Conversion Table of Single-Phase

Locked- Rotor Currents for Selection of Disconnecting

Means and Controllers as Determined from Horsepower

and Voltage Rating

Rated Horsepower

Maximum Locked-Rotor Current in Amperes, Single Phase

115 Volts 208 Volts 230 Volts

½ 58.8 32.5 29.4 ¾ 82.8 45.8 41.4 1 96 53 48 1 ½ 120 66 60 2 144 80 72 3 204 113 102 5 336 186 168 7 ½ 480 265 240 10 600 332 300

Page 310: Dr. Robert A. Durham, PhD, PE Dr. Marcus O. Durham, PhD, PE … Nutshell Master.pdf · Dr. Robert A. Durham, PhD, PE . Dr. Marcus O. Durham, PhD, PE . A Quick Reference . The book

Page | 9-14

Table 430.251(B) Conversion Table of Polyphase Design B, C, and D Maximum Locked-Rotor

Currents for Selection of Disconnecting Means and Controllers as Determined from Horsepower

and Voltage Rating and Design Letter

Rated Maximum Motor Locked-Rotor Current in Amperes, Two- and Three-Phase, Design B, C, and D*

Horsepower 115 Volts 200 Volts 208 Volts 230 Volts 460 Volts 575 Volts

B, C, D B, C, D B, C, D B, C, D B, C, D B, C, D

½ 40 23 22.1 20 10 8 ¾ 50 28.8 27.6 25 12.5 10 1 60 34.5 33 30 15 12 1½ 80 46 44 40 20 16 2 100 57.5 55 50 25 20

3 — 73.6 71 64 32 25.6 5 — 105.8 102 92 46 36.8 7½ — 146 140 127 63.5 50.8 10 — 186.3 179 162 81 64.8 15 — 267 257 232 116 93

20 — 334 321 290 145 116 25 — 420 404 365 183 146 30 — 500 481 435 218 174 40 — 667 641 580 290 232 50 — 834 802 725 363 290

60 — 1001 962 870 435 348 75 — 1248 1200 1085 543 434 100 — 1668 1603 1450 725 580 125 — 2087 2007 1815 908 726 150 — 2496 2400 2170 1085 868

200 — 3335 3207 2900 1450 1160 250 — — — — 1825 1460 300 — — — — 2200 1760 350 — — — — 2550 2040 400 — — — — 2900 2320

450 — — — — 3250 2600 500 — — — — 3625 2900

*Design A motors are not limited to a maximum starting current or locked rotor current.

These tables for use only with 430.110, 440.12, 440.41 and 455.8(C).

Page 311: Dr. Robert A. Durham, PhD, PE Dr. Marcus O. Durham, PhD, PE … Nutshell Master.pdf · Dr. Robert A. Durham, PhD, PE . Dr. Marcus O. Durham, PhD, PE . A Quick Reference . The book

Page | 9-15

Table 8 Conductor Properties Conductors Direct-Current Resistance at 75°C (167°F)

Size Stranding Overall Copper Aluminum

(AWG Area QTY Diameter Diameter Area Uncoated Coated

or kcmil)

mm2 Circular mils

mm in. mm in. mm2 in.2 ohm/ km

ohm/ kFT

ohm/ km

ohm/ kFT

ohm/ km

ohm/ kFT

18 0.823 1620 1 — — 1.02 0.040 0.823 0.001 25.5 7.77 26.5 8.08 42.0 12.8 18 0.823 1620 7 0.39 0.015 1.16 0.046 1.06 0.002 26.1 7.95 27.7 8.45 42.8 13.1

16 1.31 2580 1 — — 1.29 0.051 1.31 0.002 16.0 4.89 16.7 5.08 26.4 8.05 16 1.31 2580 7 0.49 0.019 1.46 0.058 1.68 0.003 16.4 4.99 17.3 5.29 26.9 8.21

14 2.08 4110 1 — — 1.63 0.064 2.08 0.003 10.1 3.07 10.4 3.19 16.6 5.06 14 2.08 4110 7 0.62 0.024 1.85 0.073 2.68 0.004 10.3 3.14 10.7 3.26 16.9 5.17

12 3.31 6530 1 — — 2.05 0.081 3.31 0.005 6.34 1.93 6.57 2.01 10.45 3.18 12 3.31 6530 7 0.78 0.030 2.32 0.092 4.25 0.006 6.50 1.98 6.73 2.05 10.69 3.25

10 5.261 10380 1 — — 2.588 0.102 5.26 0.008 3.984 1.21 4.148 1.26 6.561 2.00 10 5.261 10380 7 0.98 0.038 2.95 0.116 6.76 0.011 4.070 1.24 4.226 1.29 6.679 2.04

8 8.367 16510 1 — — 3.264 0.128 8.37 0.013 2.506 0.764 2.579 0.786 4.125 1.26 8 8.367 16510 7 1.23 0.049 3.71 0.146 10.76 0.017 2.551 0.778 2.653 0.809 4.204 1.28

6 13.30 26240 7 1.56 0.061 4.67 0.184 17.09 0.027 1.608 0.491 1.671 0.510 2.652 0.808 4 21.15 41740 7 1.96 0.077 5.89 0.232 27.19 0.042 1.010 0.308 1.053 0.321 1.666 0.508 3 26.67 52620 7 2.20 0.087 6.60 0.260 34.28 0.053 0.802 0.245 0.833 0.254 1.320 0.403 2 33.62 66360 7 2.47 0.097 7.42 0.292 43.23 0.067 0.634 0.194 0.661 0.201 1.045 0.319

1 42.41 83690 19 1.69 0.066 8.43 0.332 55.80 0.087 0.505 0.154 0.524 0.160 0.829 0.253 1/0 53.49 105600 19 1.89 0.074 9.45 0.372 70.41 0.109 0.399 0.122 0.415 0.127 0.660 0.201 2/0 67.43 133100 19 2.13 0.084 10.62 0.418 88.74 0.137 0.3170 0.0967 0.329 0.101 0.523 0.159 3/0 85.01 167800 19 2.39 0.094 11.94 0.470 111.9 0.173 0.2512 0.0766 0.2610 0.0797 0.413 0.126 4/0 107.2 211600 19 2.68 0.106 13.41 0.528 141.1 0.219 0.1996 0.0608 0.2050 0.0626 0.328 0.100

250 127 — 37 2.09 0.082 14.61 0.575 168 0.260 0.1687 0.0515 0.1753 0.0535 0.2778 0.0847 300 152 — 37 2.29 0.090 16.00 0.630 201 0.312 0.1409 0.0429 0.1463 0.0446 0.2318 0.0707 350 177 — 37 2.47 0.097 17.30 0.681 235 0.364 0.1205 0.0367 0.1252 0.0382 0.1984 0.0605 400 203 — 37 2.64 0.104 18.49 0.728 268 0.416 0.1053 0.0321 0.1084 0.0331 0.1737 0.0529 500 253 — 37 2.95 0.116 20.65 0.813 336 0.519 0.0845 0.0258 0.0869 0.0265 0.1391 0.0424

600 304 — 61 2.52 0.099 22.68 0.893 404 0.626 0.0704 0.0214 0.0732 0.0223 0.1159 0.0353 700 355 — 61 2.72 0.107 24.49 0.964 471 0.730 0.0603 0.0184 0.0622 0.0189 0.0994 0.0303 750 380 — 61 2.82 0.111 25.35 0.998 505 0.782 0.0563 0.0171 0.0579 0.0176 0.0927 0.0282 800 405 — 61 2.91 0.114 26.16 1.030 538 0.834 0.0528 0.0161 0.0544 0.0166 0.0868 0.0265 900 456 — 61 3.09 0.122 27.79 1.094 606 0.940 0.0470 0.0143 0.0481 0.0147 0.0770 0.0235 1000 507 — 61 3.25 0.128 29.26 1.152 673 1.042 0.0423 0.0129 0.0434 0.0132 0.0695 0.0212

1250 633 — 91 2.98 0.117 32.74 1.289 842 1.305 0.0338 0.0103 0.0347 0.0106 0.0554 0.0169 1500 760 — 91 3.26 0.128 35.86 1.412 1011 1.566 0.02814 0.00858 0.02814 0.00883 0.0464 0.0141 1750 887 — 127 2.98 0.117 38.76 1.526 1180 1.829 0.02410 0.00735 0.02410 0.00756 0.0397 0.0121 2000 1013 — 127 3.19 0.126 41.45 1.632 1349 2.092 0.02109 0.00643 0.02109 0.00662 0.0348 0.0106

Notes: 1. These resistance values are valid only for the parameters as given. Using conductors having coated strands, different stranding type, and, especially, other temperatures changes the resistance. 2. Formula for temperature change: R2 = R1 [1 + α (T 2 - 75)] where α cu = 0.00323, α AL = 0.00330 at 75°C. 3. Conductors with compact and compressed stranding have about 9 percent and 3 percent, respectively, smaller bare conductor diameters than those shown. See Table 5A for actual compact cable dimensions. 4. The IACS conductivities used: bare copper = 100%, aluminum = 61%. 5. Class B stranding is listed as well as solid for some sizes. Its overall diameter and area is that of its circumscribing circle.

Page 312: Dr. Robert A. Durham, PhD, PE Dr. Marcus O. Durham, PhD, PE … Nutshell Master.pdf · Dr. Robert A. Durham, PhD, PE . Dr. Marcus O. Durham, PhD, PE . A Quick Reference . The book

Page | 9-16

Table 9 Alternating-Current Resistance and Reactance for 600-Volt Cables, 3-Phase, 60 Hz, 75°C (167°F)

— Three Single Conductors in Conduit

Ohms to Neutral per Kilometer Ohms to Neutral per 1000 Feet6

Size

XL (Reactance) for All Wires

Alternating-Current Resistance for Uncoated Copper Wires

Alternating-Current Resistance for Aluminum Wires

Effective Z at 0.85 PF for Uncoated Copper Wires

Effective Z at 0.85 PF for Aluminum Wires

Size

(AWG or kcmil)

PVC, Aluminum Conduits

Steel Conduit

PVC Conduit

Aluminum Conduit

Steel Conduit

PVC Conduit

Aluminum Conduit

Steel Conduit

PVC Conduit

Aluminum Conduit

Steel Conduit

PVC Conduit

Aluminum Conduit

Steel Conduit

(AWG or kcmil)

14 0.190 0.240 10.2 10.2 10.2 — — — 8.9 8.9 8.9 — — — 14

0.058 0.073 3.1 3.1 3.1 — — — 2.7 2.7 2.7 — — —

12 0.177 0.223 6.6 6.6 6.6 10.5 10.5 10.5 5.6 5.6 5.6 9.2 9.2 9.2 12

0.054 0.068 2.0 2.0 2.0 3.2 3.2 3.2 1.7 1.7 1.7 2.8 2.8 2.8

10 0.164 0.207 3.9 3.9 3.9 6.6 6.6 6.6 3.6 3.6 3.6 5.9 5.9 5.9 10

0.050 0.063 1.2 1.2 1.2 2.0 2.0 2.0 1.1 1.1 1.1 1.8 1.8 1.8

8 0.171 0.213 2.56 2.56 2.56 4.3 4.3 4.3 2.26 2.26 2.30 3.6 3.6 3.6 8

0.052 0.065 0.78 0.78 0.78 1.3 1.3 1.3 0.69 0.69 0.70 1.1 1.1 1.1

6 0.167 0.210 1.61 1.61 1.61 2.66 2.66 2.66 1.44 1.48 1.48 2.33 2.36 2.36 6

0.051 0.064 0.49 0.49 0.49 0.81 0.81 0.81 0.44 0.45 0.45 0.71 0.72 0.72

4 0.157 0.197 1.02 1.02 1.02 1.67 1.67 1.67 0.95 0.95 0.98 1.51 1.51 1.51 4

0.048 0.060 0.31 0.31 0.31 0.51 0.51 0.51 0.29 0.29 0.30 0.46 0.46 0.46

3 0.154 0.194 0.82 0.82 0.82 1.31 1.35 1.31 0.75 0.79 0.79 1.21 1.21 1.21 3

0.047 0.059 0.25 0.25 0.25 0.40 0.41 0.40 0.23 0.24 0.24 0.37 0.37 0.37

2 0.148 0.187 0.62 0.66 0.66 1.05 1.05 1.05 0.62 0.62 0.66 0.98 0.98 0.98 2

0.045 0.057 0.19 0.20 0.20 0.32 0.32 0.32 0.19 0.19 0.20 0.30 0.30 0.30

1 0.151 0.187 0.49 0.52 0.52 0.82 0.85 0.82 0.52 0.52 0.52 0.79 0.79 0.82 1

0.046 0.057 0.15 0.16 0.16 0.25 0.26 0.25 0.16 0.16 0.16 0.24 0.24 0.25

1/0 0.144 0.180 0.39 0.43 0.39 0.66 0.69 0.66 0.43 0.43 0.43 0.62 0.66 0.66 1/0

0.044 0.055 0.12 0.13 0.12 0.20 0.21 0.20 0.13 0.13 0.13 0.19 0.20 0.20

2/0 0.141 0.177 0.33 0.33 0.33 0.52 0.52 0.52 0.36 0.36 0.36 0.52 0.52 0.52 2/0

0.043 0.054 0.10 0.10 0.10 0.16 0.16 0.16 0.11 0.11 0.11 0.16 0.16 0.16

3/0 0.138 0.171 0.253 0.269 0.259 0.43 0.43 0.43 0.289 0.302 0.308 0.43 0.43 0.46 3/0

0.042 0.052 0.077 0.082 0.079 0.13 0.13 0.13 0.088 0.092 0.094 0.13 0.13 0.14

4/0 0.135 0.167 0.203 0.220 0.207 0.33 0.36 0.33 0.243 0.256 0.262 0.36 0.36 0.36 4/0

0.041 0.051 0.062 0.067 0.063 0.10 0.11 0.10 0.074 0.078 0.080 0.11 0.11 0.11

250 0.135 0.171 0.171 0.187 0.177 0.279 0.295 0.282 0.217 0.230 0.240 0.308 0.322 0.33 250

0.041 0.052 0.052 0.057 0.054 0.085 0.090 0.086 0.066 0.070 0.073 0.094 0.098 0.10

300 0.135 0.167 0.144 0.161 0.148 0.233 0.249 0.236 0.194 0.207 0.213 0.269 0.282 0.289 300

0.041 0.051 0.044 0.049 0.045 0.071 0.076 0.072 0.059 0.063 0.065 0.082 0.086 0.088

350 0.131 0.164 0.125 0.141 0.128 0.200 0.217 0.207 0.174 0.190 0.197 0.240 0.253 0.262 350

0.040 0.050 0.038 0.043 0.039 0.061 0.066 0.063 0.053 0.058 0.060 0.073 0.077 0.080

400 0.131 0.161 0.108 0.125 0.115 0.177 0.194 0.180 0.161 0.174 0.184 0.217 0.233 0.240 400

0.040 0.049 0.033 0.038 0.035 0.054 0.059 0.055 0.049 0.053 0.056 0.066 0.071 0.073

500 0.128 0.157 0.089 0.105 0.095 0.141 0.157 0.148 0.141 0.157 0.164 0.187 0.200 0.210 500

0.039 0.048 0.027 0.032 0.029 0.043 0.048 0.045 0.043 0.048 0.050 0.057 0.061 0.064

600 0.128 0.157 0.075 0.092 0.082 0.118 0.135 0.125 0.131 0.144 0.154 0.167 0.180 0.190 600

0.039 0.048 0.023 0.028 0.025 0.036 0.041 0.038 0.040 0.044 0.047 0.051 0.055 0.058

750 0.125 0.157 0.062 0.079 0.069 0.095 0.112 0.102 0.118 0.131 0.141 0.148 0.161 0.171 750

0.038 0.048 0.019 0.024 0.021 0.029 0.034 0.031 0.036 0.040 0.043 0.045 0.049 0.052

1000 0.121 0.151 0.049 0.062 0.059 0.075 0.089 0.082 0.105 0.118 0.131 0.128 0.138 0.151 1000

0.037 0.046 0.015 0.019 0.018 0.023 0.027 0.025 0.032 0.036 0.040 0.039 0.042 0.046

Page 313: Dr. Robert A. Durham, PhD, PE Dr. Marcus O. Durham, PhD, PE … Nutshell Master.pdf · Dr. Robert A. Durham, PhD, PE . Dr. Marcus O. Durham, PhD, PE . A Quick Reference . The book

Page | 9-17

Notes: 1. These values are based on the following constants: UL-Type RHH wires with Class B stranding, in cradled configuration. Wire conductivities are 100 percent IACS copper and 61 percent IACS aluminum, and aluminum conduit is 45 percent IACS. Capacitive reactance is ignored, since it is negligible at these voltages. These resistance values are valid only at 75°C (167°F) and for the parameters as given, but are representative for 600-volt wire types operating at 60 Hz. 2. Effective Z is defined as R cos(θ) + X sin(θ), where θ is the power factor angle of the circuit. Multiplying current by effective impedance gives a good approximation for line-to-neutral voltage drop. Effective impedance values shown in this table are valid only at 0.85 power factor. For another circuit power factor (PF), effective impedance (Ze) can be calculated from R and XL values given in this table as follows: Ze = R × PF + XL sin[arccos(PF)].

Page 314: Dr. Robert A. Durham, PhD, PE Dr. Marcus O. Durham, PhD, PE … Nutshell Master.pdf · Dr. Robert A. Durham, PhD, PE . Dr. Marcus O. Durham, PhD, PE . A Quick Reference . The book

Page | 9-18

Table C.4 Maximum Number of Conductors or Fixture Wires in Intermediate Metal Conduit

(IMC) (Based on Table 1, Chapter 9)

CONDUCTORS

Size Metric Designator (Trade Size)

Type (AWG/ kcmil) 16 (½)

21 (¾)

27 (1)

35 (1¼)

41 (1½)

53 (2)

63 (2½)

78 (3)

91 (3½)

103 (4)

RHH, RHW, RHW-2

14 4 8 13 22 30 49 70 108 144 186 12 4 6 11 18 25 41 58 89 120 154

10 3 5 8 15 20 33 47 72 97 124 8 1 3 4 8 10 17 24 38 50 65 6 1 1 3 6 8 14 19 30 40 52

4 1 1 3 5 6 11 15 23 31 41 3 1 1 2 4 6 9 13 21 28 36 2 1 1 1 3 5 8 11 18 24 31 1 0 1 1 2 3 5 7 12 16 20

1/0 0 1 1 1 3 4 6 10 14 18 2/0 0 1 1 1 2 4 6 9 12 15 3/0 0 0 1 1 1 3 5 7 10 13 4/0 0 0 1 1 1 3 4 6 9 11

250 0 0 1 1 1 1 3 5 6 8 300 0 0 0 1 1 1 3 4 6 7 350 0 0 0 1 1 1 2 4 5 7 400 0 0 0 1 1 1 2 3 5 6 500 0 0 0 1 1 1 1 3 4 5

600 0 0 0 0 1 1 1 2 3 4 700 0 0 0 0 1 1 1 2 3 4 750 0 0 0 0 1 1 1 1 3 4 800 0 0 0 0 0 1 1 1 3 3 900 0 0 0 0 0 1 1 1 2 3

1000 0 0 0 0 0 1 1 1 2 3 1250 0 0 0 0 0 1 1 1 1 2 1500 0 0 0 0 0 0 1 1 1 1 1750 0 0 0 0 0 0 1 1 1 1 2000 0 0 0 0 0 0 1 1 1 1

TW 14 10 17 27 47 64 104 147 228 304 392 12 7 13 21 36 49 80 113 175 234 301 10 5 9 15 27 36 59 84 130 174 224 8 3 5 8 15 20 33 47 72 97 124

RHH*, RHW*, RHW-2*, THHW, THW, THW-2

14 6 11 18 31 42 69 98 151 202 261 12 5 9 14 25 34 56 79 122 163 209 10 4 7 11 19 26 43 61 95 127 163 8 2 4 7 12 16 26 37 57 76 98

6 1 3 5 9 12 20 28 43 58 75

4 1 2 4 6 9 15 21 32 43 56 3 1 1 3 6 8 13 18 28 37 48 2 1 1 3 5 6 11 15 23 31 41 1 1 1 1 3 4 7 11 16 22 28

1/0 1 1 1 3 4 6 9 14 19 24 2/0 0 1 1 2 3 5 8 12 16 20 3/0 0 1 1 1 3 4 6 10 13 17 4/0 0 1 1 1 2 4 5 8 11 14

250 0 0 1 1 1 3 4 7 9 12 300 0 0 1 1 1 2 4 6 8 10 350 0 0 1 1 1 2 3 5 7 9 400 0 0 0 1 1 1 3 4 6 8 500 0 0 0 1 1 1 2 4 5 7

600 0 0 0 1 1 1 1 3 4 5 700 0 0 0 0 1 1 1 3 4 5 750 0 0 0 0 1 1 1 2 3 4

Page 315: Dr. Robert A. Durham, PhD, PE Dr. Marcus O. Durham, PhD, PE … Nutshell Master.pdf · Dr. Robert A. Durham, PhD, PE . Dr. Marcus O. Durham, PhD, PE . A Quick Reference . The book

Page | 9-19

800 0 0 0 0 1 1 1 2 3 4 900 0 0 0 0 1 1 1 2 3 4

1000 0 0 0 0 0 1 1 1 3 3 1250 0 0 0 0 0 1 1 1 1 3 1500 0 0 0 0 0 1 1 1 1 2 1750 0 0 0 0 0 0 1 1 1 1 2000 0 0 0 0 0 0 1 1 1 1

THHN, THWN, THWN-2

14 14 24 39 68 91 149 211 326 436 562 12 10 17 29 49 67 109 154 238 318 410

10 6 11 18 31 42 68 97 150 200 258 8 3 6 10 18 24 39 56 86 115 149 6 2 4 7 13 17 28 40 62 83 107

4 1 3 4 8 10 17 25 38 51 66 3 1 2 4 6 9 15 21 32 43 56 2 1 1 3 5 7 12 17 27 36 47 1 1 1 2 4 5 9 13 20 27 35

1/0 1 1 1 3 4 8 11 17 23 29 2/0 1 1 1 3 4 6 9 14 19 24 3/0 0 1 1 2 3 5 7 12 16 20 4/0 0 1 1 1 2 4 6 9 13 17

250 0 0 1 1 1 3 5 8 10 13 300 0 0 1 1 1 3 4 7 9 12 350 0 0 1 1 1 2 4 6 8 10 400 0 0 1 1 1 2 3 5 7 9 500 0 0 0 1 1 1 3 4 6 7

600 0 0 0 1 1 1 2 3 5 6 700 0 0 0 1 1 1 1 3 4 5 750 0 0 0 1 1 1 1 3 4 5 800 0 0 0 0 1 1 1 3 4 5 900 0 0 0 0 1 1 1 2 3 4 1000 0 0 0 0 1 1 1 2 3 4

FEP, FEPB, PFA, PFAH, TFE

14 13 23 38 66 89 145 205 317 423 545 12 10 17 28 48 65 106 150 231 309 398 10 7 12 20 34 46 76 107 166 221 285

8 4 7 11 19 26 43 61 95 127 163 6 3 5 8 14 19 31 44 67 90 116

4 1 3 5 10 13 21 30 47 63 81 3 1 3 4 8 11 18 25 39 52 68 2 1 2 4 6 9 15 21 32 43 56

PFA, PFAH, TFE 1 1 1 2 4 6 10 14 22 30 39 1/0 1 1 1 4 5 8 12 19 25 32

2/0 1 1 1 3 4 7 10 15 21 27 3/0 0 1 1 2 3 6 8 13 17 22 4/0 0 1 1 1 3 5 7 10 14 18

Z 14 16 28 46 79 107 175 247 381 510 657 12 11 20 32 56 76 124 175 271 362 466 10 7 12 20 34 46 76 107 166 221 285 8 4 7 12 21 29 48 68 105 140 180 6 3 5 9 15 20 33 47 73 98 127

4 1 3 6 10 14 23 33 50 67 87 3 1 2 4 7 10 17 24 37 49 63 2 1 1 3 6 8 14 20 30 41 53 1 1 1 3 5 7 11 16 25 33 43

XHH, XHHW, XHHW-2, ZW

14 10 17 27 47 64 104 147 228 304 392 12 7 13 21 36 49 80 113 175 234 301

10 5 9 15 27 36 59 84 130 174 224 8 3 5 8 15 20 33 47 72 97 124 6 1 4 6 11 15 24 35 53 71 92

4 1 3 4 8 11 18 25 39 52 67 3 1 2 4 7 9 15 21 33 44 56 2 1 1 3 5 7 12 18 27 37 47

Page 316: Dr. Robert A. Durham, PhD, PE Dr. Marcus O. Durham, PhD, PE … Nutshell Master.pdf · Dr. Robert A. Durham, PhD, PE . Dr. Marcus O. Durham, PhD, PE . A Quick Reference . The book

Page | 9-20

1 1 1 2 4 5 9 13 20 27 35

1/0 1 1 1 3 5 8 11 17 23 30 2/0 1 1 1 3 4 6 9 14 19 25 3/0 0 1 1 2 3 5 7 12 16 20 4/0 0 1 1 1 2 4 6 10 13 17

250 0 0 1 1 1 3 5 8 11 14 300 0 0 1 1 1 3 4 7 9 12 350 0 0 1 1 1 3 4 6 8 10 400 0 0 1 1 1 2 3 5 7 9 500 0 0 0 1 1 1 3 4 6 8

600 0 0 0 1 1 1 2 3 5 6 700 0 0 0 1 1 1 1 3 4 5 750 0 0 0 1 1 1 1 3 4 5 800 0 0 0 0 1 1 1 3 4 5 900 0 0 0 0 1 1 1 2 3 4

1000 0 0 0 0 1 1 1 2 3 4 1250 0 0 0 0 0 1 1 1 2 3 1500 0 0 0 0 0 1 1 1 1 2 1750 0 0 0 0 0 1 1 1 1 2 2000 0 0 0 0 0 0 1 1 1 1

Page 317: Dr. Robert A. Durham, PhD, PE Dr. Marcus O. Durham, PhD, PE … Nutshell Master.pdf · Dr. Robert A. Durham, PhD, PE . Dr. Marcus O. Durham, PhD, PE . A Quick Reference . The book

Page | 9-21

NEMA Controller Size for Motors, Transformers, & Capacitors

Load Voltage

Continuous Current

Service Limit Current

Motor Maximum

Motor Maximum

Transformer Primary Switching

Transformer Primary Switching

Capacitor Switching

Circuit Closing Maximum

Non-plugging and Non-jogging Duty

Plugging and Jogging Duty

Inrush Current < = 20 times Continuous Amp

Inrush Current = 20 to 40 times Continuous Amp

Inrush Current Peak Including Offset

NEMA

V Amp Amp HP HP HP HP kVA kVA kVA kVA kVAR Amp

Size 1φ 3 φ 1 φ 3 φ 1 φ 3 φ 1 φ 3 φ 3 φ 3 φ

00 115 200 230 380 460 575

9 11 1/3 — 1 — — —

— 1-1/2 1-1/2 1-1/2 2 2

1/4 — 1/2 — — —

— 1 1 1 1-1/2 1-1/2

— — — — — —

— — — — — —

— — — — — —

— — — — — —

— — — — — —

87

0 115 200 230 380 460 575

18 21 1 — 2 — — —

— 3 3 5 5 5

1/2 — 1 — — —

— 1-1/2 1-1/2 1-1/2 2 2

0.6 — 1.2 — 2.4 3

— 1.8 2.1 — 4.2 5.2

0.3 — 0.6 — 1.2 1.5

— 0.9 1 — 2.1 2.6

— — — — — —

140

1 115 200 230 380 460 575

27 32 2 — 3 — — —

— 7-1/2 7-1/2 10 10 10

1 — 2 — — —

— 3 3 5 5 5

1.2 — 2.4 — 4.9 6.2

— 3.6 4.3 — 8.5 11

0.6 — 1.2 — 2.5 3.1

— 1.8 2.1 — 4.3 5.3

— — 6 — 13.5 17

288

1P 115 230

36 42 3 5

— —

1-1/2 3

— —

— —

— —

— —

— —

— —

— —

2 115 200 230 380 460 575

45 52 3 — 7-1/2 — — —

— 10 15 25 25 25

2 — 5 — — —

— 7-1/2 10 15 15 15

2.1 — 4.1 — 8.3 10

— 6.3 7.2 — 14 18

1 — 2.1 — 4.2 5.2

— 3.1 3.6 — 7.2 8.9

— — 12 — 25 31

483

3 115 200 230 380 460 575

90 104 7-1/2 — 15 — — —

— 25 30 50 50 50

7-1/2 — 15 — — —

— 15 20 30 30 30

4.1 — 8.1 — 16 20

— 12 14 — 28 35

2 — 4.1 — 8.1 10

— 6.1 7.0 — 14 18

— — 27 — 53 67

947

4 115 200 230 380 460 575

135 156 — — — — — —

— 40 50 75 100 100

— — — — — —

— 25 30 50 60 60

6.8 — 14 — 27 34

— 20 23 — 47 59

3.4 — 6.8 — 14 17

— 10 12 — 23 29

— — 40 — 80 100

1581

5 115 200 230 380 460 575

270 311 — — — — — —

— 75 100 150 200 200

— — — — — —

— 60 75 125 150 150

14 — 27 — 54 68

— 41 47 — 94 117

6.8 — 14 — 27 34

— 20 24 — 47 59

— — 80 — 160 200

3163

6 115 200

540 621 — —

— 150

— —

— 125

27 —

— 81

14 —

— 41

— —

6326

Page 318: Dr. Robert A. Durham, PhD, PE Dr. Marcus O. Durham, PhD, PE … Nutshell Master.pdf · Dr. Robert A. Durham, PhD, PE . Dr. Marcus O. Durham, PhD, PE . A Quick Reference . The book

Page | 9-22

230 380 460 575

— — — —

200 300 400 400

— — — —

150 250 300 300

54 — 108 135

94 — 188 234

27 — 54 68

47 — 94 117

160 — 320 400

7 230 460 575

810 932 — — —

300 600 600

— — —

— — —

— — —

— — —

— — —

— — —

240 480 600

9470

8 230 460 575

1215 1400 — — —

450 900 900

— — —

— — —

— — —

— — —

— — —

— — —

360 720 900

14205

9 230 460 575

2250 2590 — — —

800 1600 1600

— — —

— — —

— — —

— — —

— — —

— — —

665 1325 1670

25380

Service-Limit Current Ratings - The service-limit current ratings shown represent the maximum rms current, in amperes, which the controller

shall be permitted to carry for protracted periods in normal service. At service-limit current ratings, temperature rises shall be permitted to exceed

those obtained by testing the controller at its continuous current rating. The current rating of overload relays or the trip current of other motor

protective devices used shall not exceed the service-limit current rating of the controller.

Plugging or Jogging Service - The listed horsepower ratings are recommended for those applications requiring repeated interruption of stalled

motor current encountered in rapid motor reversal in excess of five openings or closings per minute and shall not be more than ten in a ten minute

period.

Capacitor terminals - If maximum available current is greater than 3,000 amperes, consult NEMA ICS-2 Standard.

Page 319: Dr. Robert A. Durham, PhD, PE Dr. Marcus O. Durham, PhD, PE … Nutshell Master.pdf · Dr. Robert A. Durham, PhD, PE . Dr. Marcus O. Durham, PhD, PE . A Quick Reference . The book

Page | 9-23

NEMA Table 11 Typical Characteristics and Applications of Fixed Frequency Small and Medium AC

Squirrel-Cage Induction Motors

Design Letter Locked Rotor Torque

Pull- up Torque

Break-down Torque

Locked Rotor Current

Slip Typical Applications Relative Efficiency

Polyphase Characteristics

Percent Rated Load Torque*

Percent Rated Load Torque

Percent Rated Load Torque

Percent Rated Load Current

Percent Sync Speed

Design A

High locked rotor torque High locked rotor current

70-275 65-190 175-300 Not defined

0.5-5% Fans, blowers, centrifugal pumps and compressors, motor-generator sets, etc., where starting torque requirements are relatively low.

Medium or high

Design B

Normal locked rotor torque Normal locked rotor current

70-275 65-190 175-300 600-700 0.5-5% Fans, blowers, centrifugal pumps and compressors, motor-generator sets, etc., where starting torque requirements are relatively low.

Medium or high

Design C

High locked rotor torque Normal locked rotor current

200-285 140-195 190-225 600-700 1-5% Conveyors, crushers, stirring motors, agitators, reciprocating pumps and compressors, etc., where starting under load is required

Medium

Design D

High locked rotor torque High slip

275 NA 275 600-700 5-8% High peak loads with or without flywheels such as punch presses, shears, elevators, extractors, winches, hoists, oil-well pumping and wire-drawing motors

Low

Design N

Small motor

- NA - - - Centrifugal loads where starting torque requirements are relatively low.

Low

Design 0

Small motor - NA - - NA

Design L

Medium motor

- 100% - - NA Fans, blowers, centrifugal pumps and compressors, motor-generator sets, etc., where starting torque requirements are relatively low.

Medium or low

Design M

Medium motor

- 100% - - NA Fans, blowers, centrifugal pumps and compressors, motor-generator sets, etc., where starting torque requirements are relatively low.

Medium or low

*Higher values are for motors having lower horsepower ratings.

Page 320: Dr. Robert A. Durham, PhD, PE Dr. Marcus O. Durham, PhD, PE … Nutshell Master.pdf · Dr. Robert A. Durham, PhD, PE . Dr. Marcus O. Durham, PhD, PE . A Quick Reference . The book

Page | 9-24

Frame D E 2F H N* O* P* U V AA AB* AH AJ AK BA BB BD* TAP 42 2-5/8 1-3/4 1-11/16 9/32 slot 1-1/2 5 4-11/16 3/8 1-1/8 3/8 4-1/32 1-5/16 3-3/4 3 2-1/16 1/8 4-5/8 1/4-20 48 3 2-1/8 2-3/4 11/32 slot 1-7/8 5-7/8 5-11/16 1/2 1-1/2 1/2 4-3/8 1-11/16 3-3/4 3 2-1/2 1/8 5-5/8 1/4-20 56 3-1/2 2-7/16 3 11/32 slot 2-7/16 6-7/8 6-5/8 5/8 1-7/8 1/2 5 2-1/16 5-7/8 4-1/2 2-3/4 1/8 6-1/2 3/8-16 56H 5 2-1/8 143T 3-1/2 2-3/4 4 11/32 2-1/2 6-7/8 6-5/8 7/8 2-1/4 3/4 5-1/4 2-1/8 5-7/8 4-1/2 2-1/4 1/8 6-1/2 3/8-16

145T 5 182 4-1/2 2-11/16 7/8 2-1/4 2-1/8 5-7/8 4-1/2 1/8 6-1/2 3/8-16 184 4-1/2 3-3/4 5-1/2 13/32 2-11/16 8-11/16 7-7/8 7/8 2-1/4 3/4 5-7/8 2-1/8 5-7/8 4-1/2 2-3/4 1/8 6-1/2 3/8-16 182T 4-1/2 3-9/16 1-1/8 2-3/4 2-5/8 7-1/4 8-1/2 1/4 9 1/2-13 184T 5-1/2 3-9/16 1-1/8 2-3/4 2-5/8 7-1/4 8-1/2 1/4 9 1/2-13

213 5-1/2 3-1/2 1-1/8 3 2-3/4 215 5-1/4 4-1/4 7 13/32 3-1/2 10-1/4 9-9/16 1-1/8 3 3/4 7-3/8 2-3/4 7-1/4 8-1/2 3-1/2 1/4 9 1/2-13 213T 5-1/2 3-7/8 1-3/8 3-3/8 3-1/8 215T 7 3-7/8 1-3/8 3-3/8 3-1/8 254U 8-1/4 4-1/16 1-3/8 3-3/4 3-1/2

256U 6-1/4 5 10 17/32 4-1/16 12-7/8 12-15/16 1-3/8 3-3/4 1 9-5/8 3-1/2 7-1/4 8-1/2 4-1/4 1/4 10 1/2-13 254T 8-1/4 4-5/16 1-5/8 4 3-3/4 256T 10 4-5/16 1-5/8 4 3-3/4 284U 9-1/2 5-1/8 1-5/8 4-7/8 4-5/8 286U 11 5-1/8 1-5/8 4-7/8 4-5/8

284T 7 5-1/2 9-1/2 17/32 4-7/8 14-5/8 14-5/8 1-7/8 4-5/8 1-1/2 13-1/8 4-3/8 9 10-1/2 4-3/4 1/4 11-1/4 1/2-13 286T 11 4-7/8 1-7/8 4-5/8 4-3/8 284TS 9-1/2 3-3/8 1-5/8 3-1/4 3 286TS 11 3-3/8 1-5/8 3-1/4 3 324U 10-1/2 5-7/8 1-7/8 5-5/8 5-3/8

326U 12 5-7/8 1-7/8 5-5/8 5-3/8 324T 8 6-1/4 10-1/2 21/32 5-1/2 2-1/8 5-1/4 2 14-1/8 5 11 12-1/2 5-1/4 1/4 13-3/8 5/8-11 326T 12 5-1/2 16-1/2 16-1/2 2-1/8 5-1/4 5 324TS 10-1/2 3-15/16 1-7/8 3-3/4 3-1/2 326TS 12 3-15/16 1-7/8 3-3/4 3-1/2

364U 11-1/4 6-3/4 2-1/8 6-3/8 6-1/8 365U 12-1/4 6-3/4 2-1/8 6-3/8 6-1/8 364T 9 7 11-1/4 21/32 6-1/4 18-1/2 18-1/4 2-3/8 5-7/8 2-1/2 15-1/16 5-5/8 11 12-1/2 5-7/8 1/4 13-3/8 5/8-11 365T 12-1/4 6-1/4 2-3/8 5-7/8 5-5/8 364TS 11-1/4 4 1-7/8 3-3/4 3-1/2

365TS 12-1/4 4 1-7/8 3-3/4 3-1/2 404U 12-1/4 7-3/16 2-3/8 7-1/8 6-7/8 405U 13-3/4 7-3/16 2-3/8 7-1/8 6-7/8 404T 10 8 12-1/4 13/16 7-5/16 20-5/16 20-1/8 2-7/8 7-1/4 3 18 7 11 12-1/2 6-5/8 1/4 13-7/8 5/8-11 405T 13-3/4 7-5/16 2-7/8 7-1/4 7

404TS 12-1/4 4-1/2 2-1/8 4-1/4 4 405TS 13-3/4 4-1/2 2-1/8 4-1/4 4

NEMA MOTOR DIMENSIONS

Shaft – Key Dimensions Frame Dimensions

NEMA Shaft (U)

Key (R) Dimen (S)

3/8 21/64 flat

1/2 29/64 flat 5/8 33/64 3/16

7/8 49/64 3/16 1-1/8 63/64 1/4 1-3/8 1-13/64 5/16

1-5/8 1-13/32 3/8 1-7/8 1-19/32 1/2

2-1/8 1-27/32 1/2 2-3/8 2-1/64 5/8

2-1/2 2-3/16 5/8 2-7/8 2-29/64 3/4

3-3/8 2-7/8 7/8 3-7/8 3-5/16 1

Page 321: Dr. Robert A. Durham, PhD, PE Dr. Marcus O. Durham, PhD, PE … Nutshell Master.pdf · Dr. Robert A. Durham, PhD, PE . Dr. Marcus O. Durham, PhD, PE . A Quick Reference . The book

Page | 9-25

444U 14-1/2 8-5/8 22-7/8 22-3/8 2-7/8 8-5/8 19-9/16 8-3/8 445U 16-1/2 8-5/8 22-7/8 22-3/8 2-7/8 8-5/8 19-9/16 8-3/8 444T 14-1/2 8-1/2 22-7/8 22-3/8 3-3/8 8-1/2 19-9/16 8-1/4

445T 11 9 16-1/2 13/16 8-1/2 22-7/8 22-3/8 3-3/8 8-1/2 19-9/16 8-1/4 447T 20 8-15/16 22-15/16 22-3/4 3-3/8 8-1/2 3 21-11/16 8-1/4 14 16 7-1/2 1/4 16-3/4 5/8-11 449T 25 8-15/16 22-15/16 22-3/4 3-3/8 8-1/2 21-11/16 8-1/4 444TS 14-1/2 5-3/16 22-7/8 22-3/8 2-3/8 4-3/4 19-9/16 4-1/2 445TS 16-1/2 5-3/16 22-7/8 22-3/8 2-3/8 4-3/4 19-9/16 4-1/2

447TS 20 4-15/16 22-15/16 22-3/4 2-3/8 4-3/4 4NPT 21-11/16 4-1/2 449TS 25 4-15/16 22-15/16 22-3/4 2-3/8 4-3/4 4NPT 21-11/16 4-1/2

NEMA MOTOR DIMENSIONS – 2

Frame Size Information Suffix letters after the NEMA frame size indicates that the frame differs in some way from the standard frame. Below is a list of suffixes that may be found after the frame size and their definition.

A DC Motor or Generator

C ―C‖ flange mounting on drive end **

D ―D‖ flange mounting on drive end **

E Shaft dimensions for elevator motors in frames larger than the 326U frame

H Frame with an ―F‖ dimension larger than a frame without (small framed motors)

J Jet pump motors

JM ―C‖-face mounted close coupling pump with mechanical seal

JP ―C‖-face mounted close coupling pump -packed pump

K Sump pump motor LP & LPH ―P‖ flange mounting vertical solid shaft pump

P & PH ―P‖ flange mounting vertical hollow shaft pump

S Standard short shaft T Included as part of a frame number-standard dimension

U Included as part of a frame number-standard dimension V Vertical mounting

Y Special mounting dimensions -manufactured specified Z Special shaft dimensions -manufactured specified

** If the face mounting is on the end opposite the drive, the suffix will be as follows: ―FC‖ or ―FD‖

Assembly Position F-1 VS. F-2

F-1 F-2

Notes From Front: -When a ―C‖ flange has been added to a NEMA motor the ―BA‖ dimensions are:

Frame: ―BA‖:

143-5TC 2-3/4 182-4TC 3-1/2 213-5TC 4-1/4 254-6TC 4-3 4

-Dimensions: C, N, O, P, & AB are specific to each manufacturer. Please call for exact dimensions.

Page 322: Dr. Robert A. Durham, PhD, PE Dr. Marcus O. Durham, PhD, PE … Nutshell Master.pdf · Dr. Robert A. Durham, PhD, PE . Dr. Marcus O. Durham, PhD, PE . A Quick Reference . The book

Page | 9-26

Pre-NEMA Dimensions

FRAME U V D 2F E

203 3/4 2 5 5-1/2 4 204 3/4 2 5 6-1/2 4 224 1 2-3/4 5-1/2 6-3/4 4-1/2 225 1 2-3/4 5-1/2 7-1/2 4-1/2 254 1-1/8 3-1/8 6-1/4 8-

1/14 5

284 1-1/4 3-3/4 7 9-1/2 5-1/2 324 1-5/8 4-5/8 8 10-

1/2 6-1/4

324S 1-5/8 3 8 10-1/2

6-1/4

326 1-5/8 4-5/8 8 12 6-1/4 326S 1-5/8 3 8 12 6-1/4

364 1-7/8 5-3/8 9 11-1/4

7

364S 1-7/8 3-1/2 9 11-1/4

7

356 1-7/8 5-3/8 9 12-1/4

7

365S 1-5/8 3-1/2 9 12-1/4

7

FRAME U V D 2F E

404 2-1/8

6-1/8 10 12-1/4 8

404S 1-7/8

3-1/2 10 12-1/4 8

405 2-1/8

6-1/8 10 13-3/4 8

405S 1-7/8

3-/2 10 13-3/4 8

444 2-3/8

6-7/8 11 14-1/2 9

444S 2-1/8

4 11 14-1/2 9

445 2-3/8

6-7/8 11 16-1/2 9

445S 2-1/8

4 11 16-1/2 9

504 2-5/8

7-5/8 12-1/2

16 8

504S 2-1/8

4 12-1/2

16 8

505 2-7/8

8-3/8 12-1/2

18 9

505S 2-1/8

4 12-1/2

18 9

584 2-7/8

11-5/8

14-1/2

18 9

584S 2-3/8

4-3/4 14-1/2

18 9

Page 323: Dr. Robert A. Durham, PhD, PE Dr. Marcus O. Durham, PhD, PE … Nutshell Master.pdf · Dr. Robert A. Durham, PhD, PE . Dr. Marcus O. Durham, PhD, PE . A Quick Reference . The book

Page | 9-27

Standard NEMA Frames vs Horsepower 900 RPM 1200 RPM 1800 RPM 3600 RPM

HP T-Frame U-Frame Original T-Frame U-Frame Original T-Frame U-Frame Original T-Frame U-Frame Original 1964-

Present 1952-Present

NEMA 1964-Present

1952-Present

NEMA 1964- Present

1952- Present

NEMA 1964- Present

1952- Present

NEMA

1 182T 213 225 145T 184 204 143T 182 203 . . . 1.5 184T 213 254 182T 184 224 145T 184 204 143T 182 203 2 213T 215 254 184T 213 225 145T 184 224 145T 184 204 3 5 7.5 10 15 20 25 30 40 50 60 75 100 125 150 200

215T 254T 256T 284T 286T 324T 326T 364T 365T 404T 405T 444T 445T . . .

254U 256U 284U 286U 326U 364U 365U 404U 405U 444U 445U . . . . .

284 324 326 364 365 404 405 444 445 504 505 . . . . .

213T 215T 254T 256T 284T 286T 324T 324T 364T 365T 404T 405T 444T 445T . .

215 254U 256U 284U 324U 326U 364U 365U 404U 405U 444U 445U . . . .

254 284 324 326 364 365 404 405 444 445 504 505 . . . .

182T 184T 213T 215T 254T 256T 284T 286T 324T 326T 364TS 365TS 404TS/405TS 405TS/444TS 444TS/445TS 455TS

213 215 254U 256U 284U 286U 324U 326U 364U 365US 404US/405US 405US/444US 444US/445US 445US . .

225 254 284 324 326 364 364/365 365/404 404/405 405S/444 444S/445S 445S/504S 504S/505S 505S . .

145T/182T 182T/184T 184T/213T 213T/215T 215T/254T 254T/256T 256T/284TS 284TS/286TS 286TS/324TS 324TS/326TS 326TS/364TS 364TS/365TS 365TS/405TS 404TS/444TS 405TS/445TS 444TS

184 213 215 254U 256U 284U/286U 286U/324U 324S/326S 326S/364US 364US 365US/405US 404US/444US 405US/445US 444US 445US .

224 225 254 284 324 326 364S/365S 364S/404S 365S/405S 404S/444S 405S/445S 444S/504S 445S/505S 504S 505S .

250 . . . . . . . . . 445TS . .

These charts are provided for reference use only. We are not responsible for any printing errors.

Page 324: Dr. Robert A. Durham, PhD, PE Dr. Marcus O. Durham, PhD, PE … Nutshell Master.pdf · Dr. Robert A. Durham, PhD, PE . Dr. Marcus O. Durham, PhD, PE . A Quick Reference . The book

Page | 9-28

NEMA Motor Enclosure Type

Type Abbreviation

Description Designed for use in

Open Drip Proof ODG Open Drip-Proof, Guarded non-hazardous, relatively clean areas, most common type, ODG-FV Open Drip-Proof, Force Ventilated

ODG-SV Open Drip-Proof, Separately Ventilated ODP Open Drip-Proof

Totally Enclosed TEAO Totally-Enclosed, Air-Over extremely wet, dirty, or dusty areas TEBC Totally-Enclosed, Blower-Cooled

TECACA Totally-Enclosed, Closed Circuit,, Air to Air

TEDC-A/A Totally-Enclosed, Dual Cooled, Air to Air TEDC-A/W Totally-Enclosed, Dual Cooled, Air to

Water

TEFC Totally-Enclosed, Fan-Cooled TENV Totally-Enclosed Non-Ventilated TETC Totally-Enclosed, Tube Cooled TEWAC Totally-Enclosed, Water/Air Cooled TEXP Totally-Enclosed, Explosion-Proof

Weather Protected

WPI Weather Protected, Type I adverse outdoor conditions

WPII Weather Protected Type II

Special XE Premium Efficient improved efficiency XL Extra Life XP Explosion-Proof withstanding an explosion of a

specified dust, gas, or vapor XT Extra Tough Dust ignition proof preventing the ignition of a dust,

gas, or vapor surrounding the motor

IEC IP-22 Open Drip-Proof representative IEC designations IP-44 Totally-Enclosed IP-54 Splash Proof IP-55 Washdown

Page 325: Dr. Robert A. Durham, PhD, PE Dr. Marcus O. Durham, PhD, PE … Nutshell Master.pdf · Dr. Robert A. Durham, PhD, PE . Dr. Marcus O. Durham, PhD, PE . A Quick Reference . The book

Page | 9-29

Electrical Power System Design Example

Pump: 20 Hp, 300 RPM, 18‖ sheave Motor: 3Φ, 460 V Power: 3Φ, 7200 LN Environment: ambient 98F, outdoor

Pump: Motor: Power: Environment:

# Measure Parameter Standard Table or Reference

Example Factor

Example Result

Problem Factor

Problem Result

1 Motor horsepower NEC 430.250 - 20 2 Full load Amps - FLA NEC 430.250 - 27 3 Lock letter code & kVA/hp NEC 430.7(B) F 5.59 112 4 Lock rotor amp calculate kVA*1000/1.732 V 112,000/1.732*460 141 5 Lock rotor amp for disconnect NEC 430.251(B) - 145 6 Wire rating:1.25*largest+other NEC 430.24 1.25*27 + 0 34 7 Insulation type NEC 310-16 - THHN 8 Insulation temperature NEC 310-16 - 90C 9 AWG / kcmil NEC 310-16 - 10 AWG 10 Temperature correction amp NEC 310-16 0.91 36 11 Max breaker rating & type NEC 430.52 800 instant 216 12 Actual breaker size NEC 240.6(A) - 200 13 Controller enclosure NEC 430.91 - 3R 14 Controller size NEMA Controller - 2 15 Controller max closing amp NEMA Controller - 483 16 Overload setting % - Amp - 105 28.4 17 Motor enclosure NEMA Enclosure TEFC 18 Motor NEMA design NEMA 11 - B 19 Motor sync speed 120 * freq / poles 120*60/4 1800 20 Motor slip - shaft speed NEMA 11 2% 1764 21 Motor frame NEMA Dimension 2 - 256T 22 Shaft diameter NEMA Dimensions U 1-5/8 23 Sheave diameter P(n*dia) = M(n*dia) 300*18/1800 3‖ 24 # wires - 3Φ + N 4 25 Conduit type & size NEC C4 et al 1/2 3/4 26 Total kVA 1.732 V * I / 1000 1.732*480*27/1000 22.5 26 Transformer kVA size NEMA - 3 – 7.5 28 Secondary volt & Y-Δ - 277 / 480 Y 29 Primary volt & Y-Δ - 12470 Δ 30 Transformers taps two 2-1/2 + 31 Transformer impedance PU